You are on page 1of 130

 

  
  

Note for my Dear Students


- By CA Sanchit Grover

 This Booklet contains compilation of all the MCQs that have been provided by ICAI
in its various publications till now – Sample questions as per 70:30 scheme hosted
on ICAI website, Mock test papers and Revision Test Papers.

 Solutions to the questions have been provided in accordance with provisions of


Finance Act 2022 and have been fully updated for May 23 CA-Inter exams.
Although Students are not required to give any explanation or calculations for MCQ
questions in exam, but we have incorporated detailed explanations with each
solution for reference of students.

 Questions have been divided as per various chapters given in AVJ Study module
for ease of reference.

 This booklet contains over 350 questions that will ensure complete practice for CA-
Inter Students appearing in May 2023 exams.

 Please feel to reach me at below mentioned Instagram ID in case of any doubts or


queries.

All the Best ..!!!


CA Sanchit Grover
   

INDEX

S.No. Chapter Name Pate No.

1. Chapter 1 : BASICS AND CALCULATION OF TAX LIABILITY 1-6

2. Chapter 2 : RESIDENTIAL STATUS 7-14

3. Chapter 3 : AGRICULTURAL INCOME 15-17

4. Chapter 4 : INCOMES EXEMPT FROM TAX 18-18

5. Chapter 5 : DEDUCTIONS UNDER CHAPTER VI-A 19-21

6. Chapter 6 : INCOME UNDER THE HEAD SALARY 22-25

7. Chapter 7 : INCOME UNDER THE HEAD ‘HOUSE PROPERTY 26-28

8. Chapter 8 : INCOME UNDER THE HEAD ‘OTHER SOURCES’ 29-31

9. Chapter 9 : INCOME UNDER THE HEAD PGBP 32-39

10. Chapter 10 : INCOME UNDER THE HEAD CAPITAL GAINS 40-46

11. Chapter 11 : TAXATION OF GIFTS 47-49

12. Chapter 12 : CLUBBING OF INCOME 50-53

13. Chapter 13 : CARRY FORWARD AND SET OFF OF LOSSES 54-57

14. Chapter 14 : TDS AND TCS 58-64

15. Chapter 15 : ADVANCE TAX 65-67

16. Chapter 16 : FILING OF RETURN OF INCOME 68-71

17. Chapter 17 : MISCELLANEOUS TOPICS 72-73

18. Chapter 18 : CASE SCENARIO BASED MCQS 74-126

  
  
  Multiple Choice Questions
  for CA Inter Taxation For May 2023 Exams

CHAPTER 1 BASICS AND CALCULATION OF TAX LIABILITY


Q1) The tax liability of Mr. Saral, a resident, who was What shall be the tax payable by him on the
born on 01.04.1963 and does not opt for the value of such investments considered to be
provisions of section 115BAC for the P.Y. 2022- deemed income as per section 69?
23, on the total income of ₹ 5,60,000, (a) ₹ 2,18,400
comprising of salary income and interest on (b) ₹ 55,000
fixed deposits would be:- (c) ₹ 5,46,000
(a) ₹ 9,880 (d) ₹ 54,600
(b) ₹ 22,880 Q6) Mr. Arjun, aged 53 years, has a total income of
(c) ₹ 25,480 ₹51 lakhs for A.Y.2023-24. His total income
(d) Nil comprises of salary, income from house
property and interest on savings bank account.
Q2) The tax liability of Nirlep Co-operative Society His tax liability for A.Y.2023-24 would be –
(does not opt to pay tax under section 115BAD) (a) ₹13,96,200
on the total income of ₹ 90,000 for P.Y. 2022-23 (b) ₹14,82,000
is – (c) ₹15,35,820
(a) ₹ 24,000 (d) ₹14,69,000
(b) ₹ 28,080 Q7) Mr. Ramanan, a resident aged 40 years, has a
(c) Nil total income of ₹3,25,000 for A.Y.2023-24,
(d) ₹ 24,960 comprising of his salary income and income
Q3) What is the amount of marginal relief available from house property. His tax liability for A.Y.
to Sadvichar Ltd., a domestic company, on the 2023-24 would be –
total income of ₹ 10,03,50,000 for P.Y. 2022-23 (a) ₹3,900
(comprising only of business income) whose (b) ₹1,300
turnover in P.Y. 2020-21 is ₹ 450 crore, paying (c) ₹2,600
tax as per regular provisions of Income-tax Act? (d) Nil
Assume that the company does not exercise Q8) Unexhausted basic exemption limit of a resident
option under section 115BAA. individual can be adjusted against –
(a) ₹ 9,98,000 (a) only LTCG taxable @20% u/s 112
(b) ₹ 12,67,600 (b) only STCG taxable @15% u/s 111A
(c) ₹ 3,50,000 (c) both (a) and (b)
(d) ₹ 13,32,304 (d) casual income taxable @30% u/s 115BB
Q4) The tax payable by Dharma LLP on total income Q9) Unexhausted basic exemption limit, if any, of a
of ₹ 1,01,00,000 for P.Y. 2022-23 is – non-resident for A.Y. 2023-24 can be adjusted
(a) ₹ 35,29,340 against –
(b) ₹ 32,24,000 (a) Only LTCG taxable @20%
(c) ₹ 33,21,500 (b) Only STCG taxable @15%
(d) ₹ 31,51,200 (c) Both (a) and (b)
Q5) Mr. Raman, aged 64 years, was not able to (d) Neither (a) nor (b)
provide satisfactory explanation to the Q10) Mr. Ajay is a recently qualified doctor. He joined
Assessing Officer for the investments of ₹ 7 a reputed hospital in Delhi on\ 01.01.2023. He
lakhs not recorded in the books of accounts. earned total income of ₹ 3,40,000 till
  
Prepared by : CA Sanchit Grover 1
   CHAPTER 1 : BASICS AND CALCULATION OF TAX LIABILITY
31.03.2023. His employer advised him to claim crore, since total income exceeds ₹ 2
rebate u/s 87A while filing return of income for crore.
A.Y. 2023-24. He approached his father to (b) Surcharge @15% is leviable on income-
enquire regarding what is rebate u/s 87A of the tax computed on total income of ₹ 2.95
Act. His father told him: crore.
(i) An individual who is resident in India and (c) Surcharge @15% is leviable in respect of
whose total income does not exceed ₹ income-tax computed on capital gains of
500,000 is entitled to claim rebate under ₹ 2.05 crore; in respect of business
section 87A. income, surcharge is leviable @25% on
(ii) An individual who is resident in India and income-tax, since total income exceeds ₹
whose Gross total income does not 2 crore.
exceed ₹ 5,00,000 is entitled to claim (d) Surcharge @15% is leviable in respect of
rebate under section 87A. income-tax computed on capital gains of
(iii) Maximum rebate allowable under section ₹ 2.05 crore; surcharge @10% is leviable
87A is ₹ 5,000. on income-tax computed on business
(iv) Rebate under section 87A is available in income, since the same exceeds ₹ 50
the form of exemption from total income. lakhs but is less than ₹ 1 crore.
(v) Maximum rebate allowable under section (RTP May 2020)
87A is ₹ 12,500. Q13) Mr. Nekinsaan, aged 43 years, provides the
(vi) Rebate under section 87A is available in following income details for P.Y. 2022-23 as
the form of deduction from tax liability. follows:
As a tax expert, do you agree with the Particulars ₹ in lakhs
explanation given by Mr. Ajay’s father? Capital Gains u/s 112A 120
Choose the correct option from the Capital Gains u/s 111A 110
following: Other Income 520
(a) (ii), (iii), (vi) What shall be the tax liability of Mr. Nekinsaan
(b) (i), (v), (vi) as per regular provisions of the Income-tax Act,
(c) (ii), (iii), (iv) 1961 for A.Y. 2023-24
(d) (i), (iv), (v) (a) ₹ 260.06 lakhs
Q11) If Mr. Y’s total income for A.Y. 2023-24 is ₹ 52 (b) ₹ 253.68 lakhs
Lakhs, surcharge is payable at the rate of – (c) ₹ 256.52 lakhs
(a) 15% (d) ₹ 253.56 lakhs
(b) 12% Q14) Continuing Q. 72, what shall be tax liability of
(c) 10% Mr. Nekinsaan as per regular provisions of the
(d) 2% Income-tax Act, 1961 for A.Y. 2023-24, if the
Q12) During the P.Y.2022-23, Mr. Ranjit has short- Other Income is ₹ 480 lakhs?
term capital gains of ₹ 95 lakhs taxable under (a) ₹ 218.20 lakhs
section 111A, long-term capital gains of ₹ 110 (b) ₹ 221.03 lakhs
lakhs taxable under section 112A and business (c) ₹ 218.73 lakhs
income of ₹ 90 lakhs. Which of the following (d) ₹ 242.25 lakhs
statements is correct? Q15) Mr. Ashutosh, aged 65 years and a resident in
(a) Surcharge @25% is leviable on income- India, has a total income of ₹ 3,20,00,000,
tax computed on total income of ₹ 2.95 comprising long term capital gain taxable under
 
2 Prepared by : CA Sanchit Grover
  Multiple Choice Questions
  for CA Inter Taxation For May 2023 Exams

section 112 of ₹ 57,00,000, long term capital (Securities Transaction Tax


gain taxable under section 112A of ₹ 65,00,000 has been paid on acquisition
and other income of ₹ 1,98,00,000. What would and transfer of the said
be his tax liability for A.Y. 2023-24. Assume that shares)
Mr. Ashutosh has not opted for the provisions Other income ₹2,75,000
of section 115BAC. (MTP Oct 2021) Calculate the tax liability of Miss Nisha for
(a) ₹ 90,05,880 Assessment Year 2023-24. (MTP Nov 21)
(b) ₹ 97,25,690 (a) Nil
(c) ₹ 97,34,400 (b) ₹5,670
(d) ₹ 97,22,440 (c) ₹5,720
Q16) Under the provisions of the Income-tax Act, (d) ₹8,320
1961, the term “Person” would not include: Q20) Miss Nisha (68 years) is a resident individual.
For the Assessment Year 2023-24, she has
a) A body corporate incorporated in a following income:
country outside India Long-term capital gain on 1,80,000
b) A Limited Liability Partnership (LLP) transfer of equity shares
c) Indian branch of a foreign company ((Securities Transaction Tax has
d) A local authority (MTP April 2019) been paid on acquisition and
Q17) XYZ LLP falls under which category of person? transfer of the said shares))
Other Income 2,75,000
(a) Firm Calculate the tax liability of Miss Nisha for
(b) Company Assessment Year 2023-24.
(c) Association of persons (a) Nil
(d) Artificial judicial person (b) ₹ 5,670
(MTP May 2020) (c) ₹ 5,720
Q18) Mr. Mahesh is found to be the owner of two (d) ₹ 8,320
gold chains of 50 gms each (value of which is Q21) The Gupta HUF in Maharashtra comprises of
₹ 1,45,000 each) during the financial year Mr. Harsh Gupta, his wife Mrs. Nidhi Gupta, his
ending 31.3.2022 which are not recorded in his son Mr. Deepak Gupta, his daughter-in-law Mrs.
books of account and he could not offer Deepti Gupta, his daughter Miss Preeti Gupta
satisfactory explanation for the amount spent and his unmarried brother Mr. Gautam Gupta.
on acquiring these gold chains. As per section Which of the members of the HUF are eligible
115BBE, Mr. Mahesh would be liable to pay tax for coparcenary rights?
of:- (a) Only Mr. Harsh Gupta, Mr. Gautam Gupta
(a) ₹ 1,80,960 and Mr. Deepak Gupta
(b) ₹2,26,200 (b) Only Mr. Harsh Gupta, Mr. Gautam Gupta,
(c) ₹ 90,480 Mr. Deepak Gupta and Miss Preeti Gupta
(d) ₹ 1,23,958 (c) Only Mr. Harsh Gupta, Mr. Gautam Gupta, Mr.
Q19 Miss Nisha (68 years) is a resident individual. Deepak Gupta, Mrs. Nidhi Gupta and Mrs.
For the Assessment Year 2023-24, she has Deepti Gupta
following income: (d) All the members are co-parceners
Long-term capital gain on ₹ 1,80,000
transfer of equity shares

  
Prepared by : CA Sanchit Grover 3
   CHAPTER 1 : BASICS AND CALCULATION OF TAX LIABILITY

Answers to Chapter No. 1

Ans. 1) Option (b) is correct Add :- HEC @4% 1,24,000


Tax on first ₹ 3,00,000 Nil Tax after HEC 32,24,00
(As per CBDT circular, if person was born
on 1st April 1963, he is deemed to have Ans. 5) Option (c) is correct
attained age of 60 years on 31st March Tax on ₹ 7,00,000 @60% as per Sec 11BBE 4,20,000
st
2023 & not 1 April 2023) Add:- Mandatory surcharge of 25% 1,05,000
Tax on next 2,00,000 @ 5% 10,000 Tax before HEC 5,25,000
Tax on remaining 60,000 @ 20% 12,000 Add:- HEC @4% 21,000
Tax before HEC 22,000 Tax Liability 5,46,000
HEC @4% 880
Total Tax liability 22,880 Ans. 6) Option (d) is correct
Tax on first ₹ 51,00,000 14,76,750
Ans. 2) Option (d) is correct (12,500 + 1,00,000 + 12,30,000) + 10%
Tax on first 10,000 @ 10% 1000 Surcharge
Tax on next 10,000 @ 20% 2000 Tax on ₹ 50,00,000 13,12,500
Tax on remaining 70,000 @ 30% 21,000 (12,500 + 1,00,000 + 12,00,000)
Tax before HEC 24,000 Incremental tax 1,64,250
HEC @4% 960 Incremental income 1,00,000
Total Tax liability 24,960 Marginal relief 64,250
Tax after marginal relief 14,12,500
Ans. 3) Option (b) is correct HEC @4% 56,500
Tax on 10,03,50,000 @ 30% 3,01,05,000 Total Tax liability 14,69,000
(Since Turnover in PY 20-21 is more than
₹ 400 Cr., applicable tax rate would be Ans. 7) Option (d) is correct
30%) Tax on ₹ 325,000 income 3,750
Surcharge @12% 36,12,600 Less:- Rebate u/s 87-A being ₹ 3,750 or ₹ (3,750)
Tax before HEC 3,37,17,600 12,500 whichever is lower
Tax before HEC on ₹ 10,00,00,000 3,21,00,000 Tax Liability after rebate Nil
income
Incremental tax 16,17,600 Ans.8) Option (c) is correct. In case of resident, the benefit
Incremental income 3,50,000 of adjusting unutilized exemption limit is available only
Marginal relief 12,67,600 against LTCG or STCG and not against casual income.

Ans. 4) Option (b) is correct Ans. 9) Option (d) is correct. In case of non-resident, the
Tax on ₹ 1,01,00,000 @ 30% 30,30,000 benefit of adjusting unutilized exemption limit against LTCG
Surcharge @12% 3,63,600 or STCG is not available
Tax before HEC (A) 33,93,600
Tax on ₹ 1,00,00,000 before HEC 30,00,000 Ans. 10) Option (b) is correct.
Incremental tax 3,93,600
Incremental income 1,00,000 Ans. 11) Option (c) is correct.
Marginal Relief (B) 2,93,600 Where Total Income exceeds ₹ 50L but is not more than ₹
1 Cr., surcharge rate applicable is 10%
Tax after marginal relief (A-B) 31,00,000

 
4 Prepared by : CA Sanchit Grover
  Multiple Choice Questions
  for CA Inter Taxation For May 2023 Exams
Ans. 12) Option (b) is correct surcharge rate on tax computed on
Here Total Income is exceeding 2 Crore, hence surcharge remaining income shall also be 15%)
on STCG u/s 111A and LTCG u/s 112A shall be capped at Tax before HEC 86,59,500
15%. Further, since remaining income is up to 2 Cr., HEC @4% 3,46,380
surcharge of 15% is applicable on remaining income of ₹ Tax after HEC 90,05,880
90L
Ans. 16) Option (C) is correct
Ans. 13) Option (d) is correct. Under Income Tax Act, a branch of Company is not treated
Tax u/s 112A (120L – 1L) x 10% 11,90,000 as a distinct person (unlike GST law where specific deeming
Tax u/s 111A (110L x 15%) 16,50,000 fiction has been created for this purpose)
Tax on normal income 1,54,12,500
(12,500 + 1,00,000 + 1,53,00,000) Ans. 17) Option (A) is correct
Tax before surcharge 1,82,52,500 As per Sec 2 of Income Tax Act, the term ‘firm’ shall include
Surcharge @15% on (11,90,000 + 4,26,000 LLP also for the purposes of Income tax.
16,50,000)
Surcharge @37% on 1,54,12,500 57,02,625 Ans. 18) Option (b) is correct
Tax before HEC 2,43,81,125 Tax on ₹ 2,90,000 @60% as per Sec 11BBE 1,74,000
HEC @4% 9,75,245 Add:- Mandatory surcharge of 25% 43,500
Tax after HEC 2,53,56,370 Tax before HEC 2,17,500
Add:- HEC @4% 8,700
Ans. 14) Option (c) is correct. Tax Liability 2,26,200
Tax u/s 112A (120L – 1L) x 10% 11,90,000
Tax u/s 111A (110L x 15%) 16,50,000 Ans. 19) Option (C) is correct.
Tax on normal income 1,42,12,500 Tax on Other Income of ₹ 2,75,000 Nil
(12,500 + 1,00,000 + 1,41,00,000) (Since she is a senior citizen, exemption
Tax before surcharge 1,70,52,500 limit of ₹ 3L shall be applicable)
Surcharge @15% on (11,90,000 + 4,26,000 LTCG u/s 112A 1,80,000 5,500
16,50,000) Less:- Unutilized
Surcharge @25% on 1,42,12,500 35,53,125 Exemption limit (25,000)
Tax before HEC 2,10,31,625 Tax @10% on (1,55,000 – 1,00,000)
HEC @4% 8,41,265 Tax before Rebate u/s 87A 5,500
Tax after HEC 2,18,72,890 Less:- Rebate u/s 87-A Nil
(Not available against LTCG u/s 112A)
Ans. 15) Option (a) is correct. Tax before HEC 5,500
Tax u/s 112A (65L – 1L) x 10% 6,40,000 Add:- HEC @4% 220
Tax u/s 112 (57L x 20%) 11,40,000 Final Tax liability 5,720
Tax on normal income 57,50,000
(10,000 + 1,00,000 + 56,40,000) Ans. 20) Option (c) is correct
Tax before surcharge 75,30,000 Tax computed on normal income of ₹ Nil
Surcharge @15% on ₹ 75,30,000 11,29,500 2,75,000 (since Nisha is a senior citizen,
(Rate of surcharge on LTCG u/s 112 & applicable exemption limit would be ₹
LTCG u/s 112A shall be capped at 15%. 2,50,000)
Further, since remaining income is also LTCG u/s 112A 1,80,000 5,500
not more than ₹ 2 crore, hence Less:- Unutilized exemption
Limit (3,00,000 – 2,75,000) (25,000)

  
Prepared by : CA Sanchit Grover 5
   CHAPTER 1 : BASICS AND CALCULATION OF TAX LIABILITY
Less:- Amount taxable (1,00,000) Ans. 21) Option (B) is correct
Remaining amt. taxable 55,000 In case of an HUF, Wife of the karta doesn’t have
Less:- Rebate u/s 87A (although Total Nil coparcenary rights (i.e. she cannot demand partition). While
income < ₹ 5L but still rebate would not be children of Karta (son or daughter) have coparcenary rights,
allowed against tax computed u/s 112A) their spouses (i.e. daughter in law or son in law) don’t have
Add:- HEC @4% 220 coparcenary rights in HUF

   

 
6 Prepared by : CA Sanchit Grover
  Multiple Choice Questions
  for CA Inter Taxation For May 2023 Exams

CHAPTER 2 RESIDENTIAL STATUS


Q1) If Anirudh, (whose PGBP income from Textile as interest in saving A/c from State Bank of
business in India is ₹ 18,00,000), a citizen of India for the previous year 2022-23. Assuming
India, has stayed in India in the P.Y. 2022-23 for that he has no other income, what will be
181 days, and he is non-resident in 9 out of 10 amount of income chargeable to tax in his
years immediately preceding the current hands in India for A.Y. 2023-24?
previous year and he has stayed in India for 365 (a) ₹ 2,55,000
days in all in the 4 years immediately preceding (b) ₹ 12,65,000
the current previous year and 420 days in all in (c) ₹ 12,50,000
the 7 years immediately preceding the current (d) ₹ 12,55,000
previous year, his residential status for the A.Y. Q4) Determine residential status of Sundaram (HUF)
2023-24 would be – which carries out its transactions in Malaysia.
(a) Resident and ordinarily resident Its affairs are partly controlled from India. The
(b) Resident but not ordinarily resident Karta of HUF, Mr. Sundaram who is from
(c) Non-resident Chennai visits India on 01.06.2022 and leaves
(d) Deemed resident but not ordinarily to Malaysia on 10.02.2023. He has not visited
resident u/s 6(1A) India for the past 11 years.
Q2) Raman, a citizen of India, was employed in (a) Non-resident
Hindustan Lever Ltd. He resigned on (b) Resident but not ordinarily resident
27.09.2022. He received a salary of ₹ 40,000 (c) Deemed resident
p.m. from 1.4.2022 to 27.9.2022 from Hindustan (d) Resident and ordinarily resident
Lever Ltd. Thereafter he left for Dubai for the first (MTP Nov 21)
time on 1.10.2022 and got salary of rupee Q5) Aashish earns the following income during the
equivalent of ₹ 80,000 p.m. from 1.10.2022 to P.Y. 2022-23:
31.3.2023 in Dubai. His salary for October to • Interest on U.K. Development Bonds
December 2022 was credited in his Dubai bank (1/4th being received in India): ₹ 4,00,000
account and the salary for January to March • Capital gain on sale of a building in India
2023 was credited in his Mumbai account but received in Holland: ₹ 6,00,000
directly. He is liable to tax in respect of If Aashish is a resident but not ordinarily
(a) income received in India from Hindustan resident in India, then what will be
Lever Ltd. amount of income chargeable to tax in
(b) income received in India and in Dubai. India for A.Y. 2023-24?
(c) income received in India from Hindustan (a) ₹ 7,00,000
Lever Ltd. and income directly credited in (b) ₹ 10,00,000
India. (c) ₹ 6,00,000
(d) income received in Dubai (d) ₹ 1,00,000
Q3) Mr. Suhaan (aged 35 years), a non-resident, Q6) Mr. Sumit is an Indian citizen and a member of
earned dividend income of ₹ 12,50,000 from an the crew of an America bound Indian ship
Indian company which was declared on engaged in carriage of freight in international
30.09.2022 and credited directly to his bank traffic departing from Kochi on 25th April, 2022.
account on 05.10.2022 in France and ₹ 15,000 From the following details for the P.Y. 2022-23,
  
Prepared by : CA Sanchit Grover 7
   CHAPTER 2 : RESIDENTIAL STATUS
determine the residential status of Mr. Sumit for Q8) Mr. Rajesh, aged 53 years, and his wife, Mrs.
A.Y. 2023-24, assuming that his stay in India in Sowmya, aged 50 years, are citizens of Country
the last 4 previous years preceding P.Y. 2022-23 X. They are living in Country X since birth. They
is 365 days and last seven previous years are not liable to tax in Country X. Both of them
preceding P.Y. 2022-123 is 730 days: have keen interest in Indian Culture. Mr.
Date entered in the Continuous Discharge Rajesh’s parents and grandparents were born in
Certificate in respect of joining the ship by Mr. Country X. Mrs. Sowmya visits India along with
Sumit: 25th April, 2022 Mr. Rajesh for four months every year to be with
Date entered in the Continuous Discharge her parents, who were born in Delhi and have
Certificate in respect of signing off the ship by always lived in Delhi. During their stay in India,
Mr. Sumit: 24th October, 2022 they organize Cultural Programme in Delhi-
Mr. Sumit has been filing his income tax return NCR. Income of Mr. Rajesh and Mrs. Sowmya
in India as a Resident for previous 2 years. from the Indian sources for the P.Y. 2022-23 is
What is his residential status for A.Y. 2023-24: ₹ 18 lakhs and ₹ 16 lakhs, respectively.
(a) Resident and ordinarily resident
(b) Resident but not-ordinarily resident What is the residential status of Mr. Rajesh and
(c) Non-resident Mrs. Sowmya for A.Y.2023 -24?
(d) Non-resident till 24.10.2022 and resident a) Both are resident and ordinarily resident
till 31.03.2023 in India
Q7) Mr. Square, an Indian citizen, currently resides b) Both are non-resident in India
in Dubai. He came to India on a visit and his c) Mr. Rajesh is resident but not ordinarily
total stay in India during the F.Y. 2022-23 was resident in India and Mrs. Sowmya is
135 days. He is not liable to pay any tax in non- resident
Dubai. Following is his details of stay in India in d) Mrs. Sowmya is resident but not ordinarily
the preceding previous years: resident in India and Mr. Rajesh is
FY Days Stay in India Resident and ordinarily resident
2021-22 100 (RTP May 2022)
2020-21 125 Q9) Dividend income from Australian company
2019-20 106 received in Australia in the year 2021, brought
2018-19 83 to India during the P.Y. 2022-23 is taxable in the
2017-18 78 A.Y.2023-24 in the case of –
2016-17 37 (a) resident and ordinarily resident only
2015-16 40 (b) both resident and ordinarily resident and
resident but not ordinarily resident
What shall be his residential status for the P.Y.
(c) non-resident
2022-23 if his total income (other than income
(d) None of the above
from foreign sources) is ₹ 10 lakhs?
Q10) Mr. Ramesh, a citizen of India, is employed in
(a) Resident but not ordinary resident
the Indian embassy in Australia. He is a non-
(b) Resident and ordinary resident
resident for A.Y. 2023-24. He received salary and
(c) Non-resident
allowances in Australia from the Government of
(d) Deemed resident but not ordinarily
India for the year ended 31.03.2023 for services
resident (MTP Oct 2020)
rendered by him in Australia. In addition, he was
 
8 Prepared by : CA Sanchit Grover
  Multiple Choice Questions
  for CA Inter Taxation For May 2023 Exams

allowed perquisites by the Government. Which (b) Yes; ₹ 1,00,000 was taxable in India
of the following statements are correct? during the previous year 2021-22.
(a) Salary, allowances and perquisites (c) Yes; ₹ 70,000 was taxable in India during
received outside India are not taxable in the previous year 2022-23.
the hands of Mr. Ramesh, since he is non- (d) No; such rent is not taxable in India either
resident. during the previous year 2021-22 or
(b) Salary, allowances and perquisites during the previous year 2022-23.
received outside India by Mr. Ramesh are (RTP Nov 2020)
taxable in India since they are deemed to Q13) Who among the following will qualify as non-
accrue or arise in India. resident for the previous year 2022-23?
(c) Salary received by Mr. Ramesh is taxable - Mr. Joey, an Italian designer, came on visit
in India but allowances and perquisites to India to explore Indian handloom on
are exempt. 03.09.2022 and left on 15.12.2022. For
(d) Salary received by Mr. Ramesh is exempt past four years, he visited India for
in India but allowances and perquisites fashion shows and stayed in India for 100
are taxable. (MTP May 2020) days each year.
Q11) Mr. Mango, an Indian citizen, lives in New York, - Mr. Sanjay born and settled in Canada,
USA since the last 10 years. He has a penthouse visits India each year for three months to
in Mumbai, given on rent @2,00,000 per month. meet his parents and grandparents, born
During the year 2022-23, he came to India for in India in 1946, living in Mumbai. His
152 days in aggregate. His total stay in India in Indian income is ₹ 15,20,000.
the immediately preceding 4 previous years is - Mr. Chang, a Korean scientist, left India to
366 days. You are, being the tax consultant of his home country for fixed employment
Mr. Mango, advise him about his residential there. He stayed in India for study and
status for the A.Y. 2023-24. research in medicines from 01.01.2018
(a) Non Resident till 01.07.2022.
(b) Resident but not ordinary resident Choose the correct answer:
(c) Resident and ordinary resident (a) Mr. Joey and Mr. Chang
(d) Deemed resident (MTP April 2021) (b) Mr. Sanjay
Q12) Mr. Nishant, a resident but not ordinarily (c) Mr. Sanjay and Mr. Chang
resident for the previous year 2021-22 and (d) Mr. Chang
resident and ordinarily resident for the previous Q14) Which of the following incomes is not deemed
year 2022-23, has received rent from property in to accrue or arise in India under section 9(1)(i)
Canada amounting to ₹ 1,00,000 during the P.Y. of the Income-tax Act, 1961?
2021-22 in a bank in Canada. During the (a) Income from any business connection in
financial year 2022-23, he remitted this amount India
to India through approved banking channels. Is (b) Income through or from any property in
such rent taxable in India, and if so, how much India
and in which year? (c) Income arising from transfer of a capital
(a) Yes; ₹ 70,000 was taxable in India during asset situated in India
the previous year 2021-22.

  
Prepared by : CA Sanchit Grover 9
   CHAPTER 2 : RESIDENTIAL STATUS
(d) Income relating to operations which are which Mr. Happy should depart from India after
confined to purchase of goods in India for completing the assignment so as to qualify as
the purpose of expor non-resident for P.Y. 2022-23? (Assume that he
Q15) During the P.Y. 2022-23, Mr. Samar, a non- shall not be visiting India again during the year)
resident, received ₹75,00,000 on account of sale
of agricultural land in Mauritius. The money was (a) 29-05-2022
first received in Mauritius and then remitted to (b) 30-05-2022
his Indian bank account. Is the sum taxable in (c) 31-05-2022
India? (d) 28-09-2022 (MTP March 2019)
(a) No, as agricultural income is exempt u/s Q18) On 31.08.2022, Mr. Kashyap moved to Japan for
10(1). employment. His family accompanied him,
(b) No, as the income has accrued and arisen owing to long term nature of employment. Mrs.
outside India and is also received outside Kashyap is also planning to start a fashion
India. boutique in Japan soon, once she gets settled.
(c) Yes, since it is remitted to India in the Both Mr. & Mrs. Kashyap are Indian citizens and
same year. have been working in India for more than a
(d) Yes, as agricultural income earned decade now. Comment on their residential
outside India is not exempted in India in status for A.Y. 2023-24, assuming they did not
the hands of a non-resident. visit India after August 2019 -
(MTP April 2019) (a) Mr. & Mrs. Kashyap will qualify to be non-
Q16) Lister Internationals Inc., a non resident, resident
engaged in business of selling “Good Z” (b) Mr. Kashyap will qualify to be non-
appoints Mr. Risky as an agent in India for resident and Mrs. Kashyap will be
selling such product. Mr. Risky works as an resident but not ordinarily resident
agent for several other persons also including (c) Mr. Kashyap will qualify to be non-
non residents aiding them in selling their resident and Mrs. Kashyap will be
products. The appointment of Mr. Risky will – resident and ordinarily resident
(a) lead to business connection in India as (d) Mr. & Mrs. Kashyap will qualify to be
he is not independent agent resident but not ordinarily resident
(b) lead to business connection in India as (MTP April 2019)
he is an independent agent Q19) Mr. Sushant is a person of Indian origin,
(c) not lead to business connection in India residing in Canada. During P.Y. 2022-23, he
as he is not independent agent visited India on several occasions and his
(d) not lead to business connection in India period of stay, in total, amounted to 129 days
as he is an independent agent during P.Y. 2022-23 and his period of stay in
(MTP April 2021) India during P.Y. 2021-22, P.Y. 2020-21, P.Y.
Q17) Mr. Happy, a US citizen, came to India for an 2019-20 and P.Y. 2018-19 was 135 days, 115
assignment from 11.01.2019 to 09.10.2019 and days, 95 days and 125 days, respectively. He
went back to his home country on completion earned the following incomes during the P.Y.
of the same. He thereafter, visited India on 2022-23:
05.07.2021 again for an assignment, which
ended on 26.05.2022. What is the latest date by
 
10 Prepared by : CA Sanchit Grover
  Multiple Choice Questions
  for CA Inter Taxation For May 2023 Exams

Source of Income Amount (c) No income shall be taxable since Mr.


Income received or deemed to 2,50,000 William is a non-resident in India for P.Y.
be received in India 2022-23
Income accruing or arising or 3,75,000 (d) Salary and interest income of fixed
which is deemed to accrue or deposits with SBI (MTP April 2019)
arise in India Q21) Mr. Harry, an Indian citizen, is a marketing
Income accruing or arising 5,50,000 consultant who provides consultancy to various
and received outside India countries around the globe. Due to his
from business controlled from profession, he is required to travel across
India various countries throughout the year. His
Income accruing or arising 6,50,000 marketing project does not last for more than
and received outside India 40 days and therefore his stay in any country
from business controlled including India usually never exceeds 40 days
outside India during a year. His income is ₹ 80 lakhs across
What is the residential status of Mr. Sushant the globe which is not liable to tax in any
for A.Y. 2023-24 and his income liable to tax country. During the P.Y. 2022- 23, an Indian
in India during A.Y. 2023-24? company provides him a marketing project in
(a) Non-Resident; ₹ 6,25,000 is liable to tax India. His stay in India for the project is expected
in India to be only 25 days and his income from that
(b) Resident and ordinary resident; ₹ project would be ₹ 30 lakhs. Being a highly
18,25,000 is liable to tax in India qualified professional, he consults you about
(c) Resident but not ordinarily resident; ₹ the tax regime on his income and his residential
11,75,000 is liable to tax in India status in India.
(d) Non-Resident; ₹ 11,75,000 is liable to (a) He shall be treated as resident but not
tax in India (MTP April 2022) ordinarily resident and shall be liable to
Q20) Mr. William, an Indian citizen and a pay tax on ₹ 30 lakhs.
Government employee, left India for the first (b) He shall be treated as resident and
time on 28.02.2022 on account of his transfer ordinarily resident and shall be liable to
to High Commission in United Kingdom. During pay tax on ₹ 80 lakhs.
P.Y. 2022-23, he visited India only for a week on (c) He shall be treated as non-resident and
occasion of his brother marriage. During F.Y. shall not be liable to any tax.
2022-23, his income composition includes (d) He shall be treated as resident but not
salary, foreign allowances, rent from property in ordinarily resident and shall be liable to
Singapore and interest earned from fixed pay tax on his entire income of ₹ 80
deposits maintained with SBI. His taxable lakhs earned across the globe
income for P.Y. 2022-23 will include: (MTP March 2021)
(a) All of them, since Mr. William is a resident Q22) Which of the following statements is true for
in India, hence his global income will be companies in the context of the Income-tax
taxable Act, 1961?
(b) Only interest earned from fixed deposits (a) Residential status of a company has an
maintained in India impact on the tax rate of company

  
Prepared by : CA Sanchit Grover 11
   CHAPTER 2 : RESIDENTIAL STATUS
(b) Tax Rate of a company depends upon the (c) Resident but not ordinarily Resident and
place of incorporation ₹ 18 lakhs from Indian retail trade
(c) Residential status of a company helps to business would only be taxable.
classify the company as domestic (d) Deemed Resident as per Sec 6(1A) and ₹
company and foreign company 18 lakhs from Indian retail trade business
(d) Residential status of company helps would only be taxable
classification of closely held company Q24) Who among the following will qualify as non-
and widely held company. resident for the P.Y. 2022-23?
(MTP March 2019) - Mr. Bob, an Italian dancer, came on visit
Q23) Mr. Tejas, an Indian Citizen, left India to India to explore Indian dance on
permanently with his wife and two children, for 15.09.2022 and left on 25.12.2022. For
extending his retail trade business of toys in past four years, he visited India for dance
Canada in the year 2017. From Canada, he is competition and stayed in India for 120
managing his retail business of toys in India. days each year.
For the purpose of his Indian business, he visits - Mr. Samrat born and settled in USA, visits
India every year from 1st September to 31st India each year for 100 days to meet his
January. His business income is ₹ 23.50 lakhs parents and grandparents, born in India
and ₹ 18 lakhs from retail trade business in in 1946, living in Delhi. His Indian income
Canada and in India, respectively for the F.Y. is ₹ 15,20,000.
2022-23. He has no other income during the P.Y. - Mr. Joseph, an American scientist, left
2022-23. Determine his residential status and India to his home country for fixed
income taxable in his hands for the A.Y. 2023- employment there. He stayed in India for
24. study and research in medicines from
(a) Resident and ordinarily resident in India 01.01.2017 till 01.07.2022.
and income of ₹ 18 lakhs and ₹ 23.50 Choose the correct answer
lakhs would be taxable. (a) Mr. Bob and Mr. Joseph
(b) Non-Resident and ₹ 18 lakhs from Indian (b) Mr. Samrat
retail trade business would only be (c) Mr. Bob, Mr. Samrat and Mr. Joseph
taxable. (d) None of the three (RTP May 2021)

Answers to Chapter 2
Ans. 1) Option (b) is correct. he is ROR and hence Indian as well as Foreign Income shall
Since Anirudh is an Indian citizen who usually resides be taxable in his hands.
outside India & comes on a visit to India + His Total Income
(excluding foreign sources) is < ₹ 15L, 2nd basic condition Ans. 3) Option (d) is correct.
in modified form shall be applicable. Since He is staying for Dividend from Indian Company received in 12,50,000
atleast 120 days, he shall be deemed RNOR France (It is deemed to accrue in India as
per Sec 9)
Ans. 2) Option (b) is correct. Interest on Saving Deposits (since the 15,000
No of Days stay during PY 22-23 = 182 days. Further since sources of Income is in India, it shall be
he left India for the first time only, his 1st Basic as well as deemed to accrue in India)
Both Additional conditions shall be satisfied. Accordingly, Gross Total Income 12,65,000

 
12 Prepared by : CA Sanchit Grover
  Multiple Choice Questions
  for CA Inter Taxation For May 2023 Exams

Less:- Deduction u/s 80TTA (available to (10,000) Untaxed profits of past years brought into India during
resident as well as non-resident) relevant PY shall not be taxable, irrespective of residential
Total Income 12,55,000 status

Ans. 4) Option (b) is correct. Ans. 10) Option (c) is correct.


Since control & management of HUF is partially located in Salary received by Indian citizen from Indian Government
India, it shall be resident. Further, since Karta doesn’t satisfy for services outside India shall be deemed to accrue in India
both additional conditions, HUF shall be RNOR but allowances & perquisites received by such Indian
citizen shall be exempt u/s 10(7).
Ans. 5) Option (a) is correct.
Interest on UK developments bonds (only 1,00,000 Ans. 11) Option (b) is correct.
1/4th received in India shall be taxable in Total Income (excluding foreign sources) = ₹ 24L – 30% =
hands of RNOR) ₹ 16,80,000
Capital gains on sale of building (since it 6,00,000 Here Mr. Mango is an Indian citizen who usually resides
is located in India, cap gains will be outside India and comes on visit to India + Total Income
deemed to accrue in India as per as Sec 9) (excluding foreign sources) > ₹ 15L, hence 2nd Basic
Gross Total Income 7,00,000 condition (in modified form) shall be applicable. Since No.
of days stay during PY 22-23 > 120 days and aggregate stay
during 4 preceding PYs > 365 days, he shall be deemed to
Ans. 6) Option (a) is correct.
be RNOR as per Sec 6(1) and 6(6)
No of Days stay during PY 22-23 = 365 – (25th April to 24th
October) = 182 days. Further since Both Additional
Ans. 12) Option (d) is correct.
conditions are also satisfied in this case, he is ROR
Since he was RNOR in PY 21-22, Rent from Property in
Canada shall not be taxable in his hands since it is a
Ans. 7) Option (c) is correct.
foreign income. Further, when such income was brought to
Since Total Income (excluding foreign sources) is less than
India in PY 22-23, past untaxed profits can’t be taxed in PY
15L, hence 2nd basic condition shall not be applicable in
22-23
this case. Accordingly, although stay in PY 22-23 is more
than 60 days and aggregate stay is more than 365 days,
Ans. 13) Option (b) is correct.
still he shall be non-resident.
Mr. Joey has stayed in PY 22-23 for more than 60 days and
his aggregate stay during 4 preceding PYs is also more than
Ans. 8) Option (d) is correct
365 days, hence resident. Similar is the case with Mr. Chang.
Here Both of them visit India every year for more than 120
However, Mr. Sanjay is a person of Indian origin who
days but less than 182 days. Since Mr. Rajesh is neither
usually resides outside India and comes on a visit to India.
India citizen nor person of Indian origin, both 1st and 2nd
+ His Total Income excl. foreign sources > 15L. Since his
Basic condition shall be applicable on him. Since his stay is
stay is less than 120 days, he shall be non-resident for PY
more than 60 days in PY 22-23 and aggregate stay is more
22-23
than 365 days in 4 Preceding PYs, he shall be resident.
Further, since he comes to India for 4 months every year, he
Ans. 14) Option (d) is correct.
is satisfying both additional conditions and hence he will
Sec 9 has clarified that no business connection shall be
be ROR.
deemed to have been established in this case.
Mrs. Sowmya is a person of Indian origin whose Total
Income (excluding foreign sources) is more than 15L. Since
Ans. 15) Option (b) is correct.
she is staying for more than 120 days during PY 22-23 and
Any capital gains arising from transfer of an agricultural
her aggregate stay during 4 preceding PYs is more than 365
land is not covered in definition of Agricultural income and
days, she shall be Deemed RNOR (irrespective of whether
hence exemption u/s 10(1) shall not be applicable.
she is satisfying the additional conditions or not)
However, since the land is located outside India, such
capital gain would be considered as accrued as well as
Ans. 9) Option (d) is correct.
received outside India which can’t be taxed in hands of NR.

  
Prepared by : CA Sanchit Grover 13
   CHAPTER 2 : RESIDENTIAL STATUS
Ans. 16) Option (d) is correct. 6(1). However, since his Total income (excluding foreign
In the given case, Agent Mr. Risky is not working mainly or sources) > ₹ 15L and he is not liable to tax in any other
exclusively for Lister Internal Inc., hence he can be country, he shall be deemed as RNOR as per Sec 6(1A).
considered to be agent of independent status and Accordingly, he shall be liable to pay tax on ₹ 30L only
accordingly won’t be covered in the definition of business being income accrued in India.
connection as per Sec 9.
Ans. 22) Option (B) is correct
Ans. 17) Option (a) is correct. In order to determine tax rate applicable on Company, we
Since Mr. Happy is not citizen of India, 2nd Basic condition check whether such Company is Domestic Company or
shall also be applicable in his case. Since the aggregate Foreign Company (residential status has no impact on tax
stay in 4 preceding PYs is more than 365 days, in order to rate). However, while determining residential status of
be non-resident he must not stay in India for 60 or more Company as per Sec 6, if the place of incorporation of
days. Accordingly, 59 days from 1st April comes out to be Company is in India, it is always treated as resident
29th May 2022 (date of departure is also considered as stay irrespective of where its control & management lies
in India)
Ans. 23) Option (c) is correct.
Ans. 18) Option (c) is correct. No of Days stay of both during PY 22-23 = 153 days and
No of Days stay of both during PY 22-23 = 153 days and Aggregate stay during 4 years is more than 365 days. Since
Aggregate stay during 4 years is more than 365 days. Since Mr. Tejas is an Indian citizen who usually resides outside
Mr. Kashyap left India for the purpose of employment, 2nd India & only comes on a visit to India and his Total Income
Basic condition is not applicable on him and hence he (excluding Income from foreign sources) > 15L, 2nd Modified
becomes non-resident. However, Mrs. Kashyap did not leave basic condition shall be applicable. Since it is getting
India for employment and hence 2nd Basic condition is also satisfied, he shall be RNOR (irrespective of whether
applicable on here. Accordingly, she becomes ROR (2nd additional conditions are getting satisfied or not). Further,
Basic condition + Both Additional conditions) since he is RNOR, only income from business in India would
be taxable in his hands, not income from business in
Ans. 19) Option (a) is correct. Canada (since that business is not being controlled from
Total Income (excluding foreign sources) = ₹ 2, 50,000 + ₹ India)
3, 75,000 + ₹ 5, 50,000 = ₹ 11, 75,000 (i.e. < ₹ 15L).
Accordingly, in this case, only 1st Basic Condition shall be Ans. 24) Option (b) is correct.
applicable and since it is not getting satisfied, Mr. Sushant - In case of Mr. Bob, since he is neither Indian citizen
shall be a non-resident. nor person of Indian origin, 2nd basic condition is also
Total Income (only Indian incomes taxable) = ₹ 2, 50,000 + applicable. Since he stays in India for > 60 days in PY
₹ 3, 75,000 = ₹ 6, 25,000 22-23 and aggregate is more than 365 in preceding 4
PYs, he shall be resident
Ans. 20) Option (d) is correct. - Mr. Samrat is a person of Indian origin who usually
Since he visited India only for a week, he shall be non- resides outside India and comes on a visit to India
resident. Salary received by him from Indian Government having Total Income (excluding Income from foreign
shall be deemed to accrue in India and hence taxable but sources > ₹ 15L). Since his stay is less than 120 days
allowances & perquisites shall be exempt u/s 10(7). Further, in PY 22-23, he shall be non-resident.
Interest from FD in India shall also be deemed to accrue in - Mr. Joseph is neither India citizen nor person of Indian
India as per Sec 9. origin, hence 2nd Basic condition also applicable. His
stay during PY 22-23 is more than 60 days &
Ans. 21) Option (a) is correct. aggregate in preceding 4 PYs is more than 365 days,
In the given case, since Mr. Harry is not satisfying any of accordingly he shall be resident.
the basic conditions, he shall be non-resident as per Sec

 
14 Prepared by : CA Sanchit Grover
  Multiple Choice Questions
  for CA Inter Taxation For May 2023 Exams

CHAPTER 3 AGRICULTURAL INCOME


Q1) Mr. Harini earned income of ₹ 4,00,000 from the company has 80% of income exempt as
sale of tea grown and manufactured in Shimla. agricultural income and 20% is taxable as
Income from sapling and seedling grown in business income. During the P.Y. 2022-23, Jenny
nursery at Cochin is ₹ 80,000. The basic derived ₹ 5,000 as interest income from the
operations were not carried out by her on land. above investments. Which of the following
Her agricultural income is statements are correct on taxability:
(a) ₹4,80,000 (a) Interest will be exempt from tax to the
(b) ₹4,00,000 extent of 80% , since Hariyali Pvt. Ltd has
(c) ₹2,40,000 80% exempted income.
(d) ₹3,20,000 (b) Interest will be exempt from tax to the
Q2) Mr. Prem earned income of ₹ 22 lakhs from extent of 20% , since Hariyali Pvt. Ltd has
manufacture and sale of coffee grown, cured, claimed 80% of income as exempt.
roasted and grounded by him in India. The (c) Interest will be fully taxable
business income chargeable to tax in his hands (d) Interest will be fully exempt
would be – Q5) Which of the following statements is/are true in
(a) ₹8,80,000 respect of taxability of agricultural income
(b) ₹5,50,000 under the Income-tax Act, 1961?
(c) ₹13,20,000 (i) Any income derived from saplings or
(d) ₹16,50,000 seedlings grown in a nursery is
Q3) Mr. Devansh has agricultural income of agricultural income exempt from tax u/s
₹2,30,000 and business income of ₹2,45,000. 10(1).
Which of the following statements are correct? (ii) 60% of dividend received from shares
(a) Agricultural income has to be aggregated held in a tea company is agricultural
with business income for tax rate income exempt from tax u/s 10(1).
purposes. (iii) While computing income tax liability of
(b) No aggregation is required since an Assessee aged 50 years, agricultural
agricultural income is less than basic income is required to be added to total
exemption limit. income only if net agricultural income for
(c) No aggregation is required since the P.Y. exceeds ₹5,000 and the total
business income is less than basic income (including net agricultural
exemption limit. income) exceeds ₹2,50,000.
(d) Agricultural income is exempt under (iv) While computing income tax liability of
section 10(1) but the same has to be an Assessee aged 50 years, agricultural
aggregated with business income, since income is required to be added to total
it exceeds ₹5,000. (MTP March 2019) income only if net agricultural income for
Q4) Jenny has invested in debt securities of Haryali the P.Y. exceeds ₹5,000 and the total
Pvt. Ltd., a company deriving its main source of income (excluding net agricultural
income from business of growing and income) exceeds ₹2,50,000.
processing organic vegetables and fruits. Thus, Choose from the following options:

  
Prepared by : CA Sanchit Grover 15
   CHAPTER 3 : AGRICULTURAL INCOME
(a) (i) and (iii) Q8) For A.Y.2023-34, Mr. Hari, a resident Indian,
(b) (ii) and (iii) earns income of ₹ 10 lakhs from sale of rubber
(c) (i) and (iv) manufactured from latex obtained from rubber
(d) (i), (ii) and (iv) (MTP Oct 2019) plants grown by him in India and ₹ 15 lakhs
Q6) Sham Singh spends ₹ 1,00,000 on cultivation from sale of rubber manufactured from latex
and harvesting of his agricultural produce. 50% obtained from rubber plants grown by him in
of the production is sold for ₹ 1,10,000 and rest Malaysia. What would be his business income
is stored for self-consumption. What is the chargeable to tax in India, assuming he has no
amount of the agricultural income? other business?
(a) ₹ 60,000 (a) ₹ 3,50,000
(b) ₹ 1,10,000 (b) ₹ 4,00,000
(c) ₹ 1,20,000 (c) ₹ 8,75,000
(d) ₹ 1,00,000 (MTP Oct 2020) (d) ₹ 18,50,000
Q7) Income derived from farm building situated in Q9) Ms. Sowmya has three farm buildings situated
the immediate vicinity of an agricultural land in the immediate vicinity of a rural agricultural
(not assessed to land revenue) would be treated land. In the P.Y.2022-23, she earned
as agricultural income if such land is situated - ₹ 3 lakh from letting out her farm
in – building 1 for storage of food grains,
(a) an area at a distance of 3 kms from the - ₹ 10 lakh from letting out her farm
local limits of a municipality and has a building 2 for storage of dairy products
population of 80,000 as per last census and
(b) an area within 1.5 kms from the local - ₹ 15 lakh from letting out her farm
limits of a municipality and has a building 3 for residential purposes of Mr.
population of 12,000 as per last census Sumanth, whose food grain produce is
(c) an area within 2 kms from the local limits stored in farm building 1.
of a municipality and has a population of What is the amount of agricultural
11,00,000 as per last census income exempt from income-tax?
(d) an area within 8 kms from the local limits (a) Nil
of a municipality and has a population of (b) ₹ 3,00,000
10,50,000 as per last census. (c) ₹ 13,00,000
(d) ₹ 18,00,000

Answers to Chapter 3
Ans. 1) Option (d) is correct. In respect of composite business of coffee grown, cured,
roasted & grounded, 40% of 22L shall be considered as
Agricultural income in respect of Tea 2,40,000
business income.
business (60% of 4,00,000)
Income from sapling & seedling 80,000
Ans. 3) Option (c) is correct.
Total Agricultural income 3,20,000

Ans. 4) Option (c) is correct.


Ans. 2) Option (a) is correct.
Even if Income of the Company is partially agricultural
income, any Interest paid by such company on its
 
16 Prepared by : CA Sanchit Grover
  Multiple Choice Questions
  for CA Inter Taxation For May 2023 Exams
debentures would not be treated as agricultural income in would qualify as rural area and accordingly Income from
hands of investor. farm building would get qualified as agricultural income

Ans. 5) Option (c) is correct. Ans. 8) Option (D) is correct.


Business Income from sale of rubber in 15,00,000
Ans. 6) Option (a) is correct. Malaysia (entirely it will be categorized
Since only 50% of the sale produce of agricultural goods as Business Income)
has been sold, hence agricultural income shall be Business income from sale of rubber 3,50,000
computed as under:- plants in India (35% of 10,00,000)
Sale Price of agricultural produce 1,10,000 Total Business Income 18,50,000
Less:- Cost of cultivating this 50% of (50,000)
agricultural produce
Ans. 9) Option (B) is correct.
(50% of ₹ 1,00,000)
As per Sec 2(1A), income from farm building located in rural
Agricultural Income 60,000
area shall be considered as agricultural income if such
building is, by reason of connection with the land, used as
Ans. 7) Option (A) is correct.
a dwelling house or a store house or an out-house by the
In case of any municipality having population of more than
cultivator or receiver of rent in kind. In the given case,
10,000 but less than 1,00,000, only area within 2 km shall
Building 1 is covered in above purposes but Building 2 and
be considered to be urban area. Hence area beyond that
3 don’t satisfy the above condition.

  
Prepared by : CA Sanchit Grover 17
   CHAPTER 4 : INCOMES EXEMPT FROM TAX

CHAPTER 4 INCOMES EXEMPT FROM TAX


Q1) Which of the following income would be exempt scholarship of ₹5,00,000 from his employer to
in the hands of a Sikkimese Individual? meet the cost of education of his children. X
(a) only income from any source in the State actually spent an amount of ₹4,50,000 on
of Sikkim education of his children. What will be the
(b) only income by way of dividend amount of income exempt in the hands of X?
(c) only income from interest on securities (a) Nil
(d) All the above (b) ₹4,50,000
Q2) Mr. Joe, a foreign national, working with Mint (c) ₹50,000
Inc., a USA company, came India during the P.Y. (d) ₹5,00,000
2022-23 for rendering services on behalf of the Q4) XYZ Ltd. has two units, one unit at Special
employer. He wishes to claim his salary income Economic Zone (SEZ) and other unit at
earned during his stay in India as exempt. Domestic Tariff Area (DTA). The unit in SEZ was
Which of the following conditions are NOT set up and started manufacturing from
necessary to be fulfilled to claim such 12.3.2016 and unit in DTA from 15.6.2019. Total
remuneration as exempt income turnover of XYZ Ltd. and Unit in DTA is ₹
(a) Mint Inc. Is not engaged in a business 8,50,00,000 and ₹ 3,25,00,000, respectively.
activity in India. Export sales of unit in SEZ and DTA is ₹
(b) Mr. Joe should be an overseas citizen of 2,50,00,000 and ₹ 1,25,00,000, respectively and
India net profit of Unit in SEZ and DTA is ₹ 80,00,000
(c) Mr. Joe stay in India should not exceed 90 and ₹ 45,00,000, respectively. XYZ Ltd. would be
days during the P.Y. 2022-23 eligible for deduction under section 10AA in
(d) Remuneration received by Mr. Joe is not respect of PY 22-23 for -
liable to be deducted from Mint Inc.'s (a) ₹ 38,09,524
income chargeable to tax under the Act (b) ₹ 19,04,762
Q3) Mr. X, a resident employee of Hindustan (c) ₹ 23,52,941
Company established in India, received a (d) ₹ 11,76,471

Answers to Chapter 4

Ans. 1) Option (d) is correct Sec 10(6) while giving exemption in respect of Salary earned
As per Sec 10(26AAA), any income derived by Sikkimese by Foreign National working in a foreign entity doesn’t
individual from Sikkim + Any interest & dividend income prescribe any such condition that such individual should be
shall be exempt. Overseas Citizen of India

Ans. 2) Option (b) is correct Ans. 4) Option (b) is correct


Sec 10(6) while giving exemption in respect of Salary earned Export profits of SEZ unit = 80L x 250L / 525L
by Foreign National working in a foreign entity doesn’t = ₹ 38,09,524
prescribe any such condition that such individual should be Since it is 7th year of operation, only 50% of export profits
Overseas Citizen of India shall be exempt u/s 10AA. Hence Exemption u/s 10AA shall
be ₹ 19,04,762/-
Ans. 3) Option (b) is correct

 
18 Prepared by : CA Sanchit Grover
  Multiple Choice Questions
  for CA Inter Taxation For May 2023 Exams

CHAPTER 5 DEDUCTIONS UNDER CHAPTER VI-A


Q1) Mr. Srivastav, aged 72 years, paid medical maintenance. For A.Y.2023-24, Rajan would be
insurance premium of ₹ 52,000 by cheque and eligible for deduction under section 80DD of an
₹ 4,000 by cash during May, 2022 under a amount equal to –
Medical Insurance Scheme of the General (a) ₹ 15,000
Insurance Corporation. The above sum was paid (b) ₹ 35,000
for insurance of his own health. He would be (c) ₹ 75,000
entitled to a deduction under section 80D of a (d) ₹ 1,25,000
sum of – Q6) Mr. Shiva made a donation of ₹ 50,000 to PM
(a) ₹ 30,000 Cares Fund and ₹ 20,000 to Rajiv Gandhi
(b) ₹ 50,000 Foundation by cheque. He made a cash
(c) ₹ 52,000 donation of ₹ 10,000 to a public charitable trust
(d) ₹ 56,000 registered under section 80G. The deduction
Q2) Mr. Ramesh pays a rent of ₹ 5,000 per month. allowable to him under section 80G for A.Y.
His total income is ₹ 2,80,000 (i.e., Gross Total 2023-24 is –
Income as reduced by deductions under (a) ₹ 80,000
Chapter VI-A except section 80GG). He is also in (b) ₹ 70,000
receipt of HRA. He would be eligible for a (c) ₹ 60,000
deduction under section 80GG of an amount of- (d) ₹ 35,000
(a) ₹ 60,000 Q7) Mr. Anuj is a businessman whose total income
(b) ₹ 32,000 (after allowing deduction under Chapter VI-A
(c) ₹ 70,000 except under section 80GG) for A.Y. 2023-24 is
(d) Nil ₹ 5,95,000. He does not own any house property
Q3) An individual has paid life insurance premium and is staying in a rented accommodation in
of ₹ 25,000 during the previous year for a policy Patna for a monthly rent of ₹ 9,000. Deduction
of ₹ 1,00,000 taken on 1.4.2018. He shall – under section 80GG for A.Y. 2023-24 is –
(a) not be allowed deduction u/s 80C (a) ₹ 48,500
(b) be allowed deduction of ₹ 20,000 u/s 80C (b) ₹ 1,48,750
(c) be allowed deduction of ₹ 25,000 u/s 80C (c) ₹ 60,000
(d) be allowed deduction of ₹ 10,000 u/s 80C (d) ₹ 1,08,000
Q4) The maximum amount which can be donated in Q8) Mr. Shaleen, a businessman, whose total
cash for claiming deduction under section 80G income (before allowing deduction under
for the P.Y. 2022-23 is – section 80GG) for A.Y. 2023-24 is ₹ 4,60,000,
(a) ₹ 5,000 paid house rent at ₹12,000 p.m. in respect of
(b) ₹ 10,000 residential accommodation occupied by him at
(c) ₹ 1,000 Chennai. The deduction allowable to him under
(d) ₹ 2,000 section 80GG for A.Y. 2023-24 is
Q5) Rajan, a resident Indian, has incurred ₹ 15,000 (a) ₹ 98,000
for medical treatment of his dependent brother, (b) ₹ 1,15,000
who is a person with severe disability and has (c) ₹ 60,000
deposited ₹ 20,000 with LIC for his (d) ₹ 24,000
  
Prepared by : CA Sanchit Grover 19
   CHAPTER 5 : DEDUCTIONS UNDER CHAPTER VI-A
Q9) Gross total income of Arpita for P.Y. 2022-23 is (b) No, as what is done after income is
₹ 6,00,000. She had taken a loan of ₹ 7,20,000 earned by Mr. Krishna will not give him
in the financial year 2019-20 from a bank for any tax exemption. His total income shall
her husband who is pursuing MBA course from be ₹ 10,00,000.
IIM, Kolkata. On 02.04.2022, she paid the first (c) His gross total income and total income
installment of loan of ₹ 45,000 and interest of ₹ are ₹ 10 lakhs, since this is a case of
65,000. Compute her total income for A.Y. 2023- application of income and donation
24. made in cash will not qualify for
(a) ₹ 6,00,000 deduction under section 80G.
(b) ₹ 5,35,000 (d) Yes, as ₹ 8,00,000 is exempt from tax, the
(c) ₹ 4,90,000 gross total income as well as total income
(d) ₹ 5,55,000 of Mr. Krishna shall be ₹ 2,00,000 only.
Q10) Soumil, aged 47 years, paid medical insurance (MTP March 2019)
premium of ₹ 15,000 and ₹ 20,000 to insure Q12) Mr. Krishna, a resident Indian aged 61 years,
health of himself and his spouse, respectively. maintains a saving account with a co-operative
He also paid medical insurance premium of ₹ land development bank and he earns ₹ 20,000
43,000 to insure health of his father, aged 69 as interest on saving account for the Financial
years, not dependent on him. He had also Year 2021-22. Mr. Krishna also maintains a
incurred ₹ 4,000 in cash on preventive health fixed deposit and recurring deposit account with
check up of his father. Total deduction Mani Finance (A Non-Banking Finance
admissible under section 80D to Mr. Soumil is: Company) and earns ₹ 25,000 and ₹ 10,000 as
(a) ₹55,000 interest on fixed deposit and recurring deposit,
(b) ₹29,000 respectively. What would be the deduction
(c) ₹68,000 allowable to Mr. Krishna under Chapter VI-A if
(d) ₹72,000 he does not opt for the section 115BAC for the
Q11) Mr. Krishna is a philanthropic person. During A.Y. 2023-24?
the P.Y. 2022-23, out of his total receipts, he (a) ₹ 55,000
gave away ₹ 8,00,000 in cash to Prime (b) ₹ 10,000
Minister‟s National Relief Fund and was left (c) ₹ 20,000
with only ₹ 2,00,000 which is just enough (d) ₹ 50,000 (MTP April 2021)
money to meet his personal requirements. On Q13) Which of the following statements is/are correct
these facts, Mr. Krishna is of the view that as ₹ in respect of deduction allowed to an assessee
2,00,000 is below the maximum amount not in respect of certain donations for scientific
chargeable to tax, no income of him is research or rural development u/s 80GGA?
chargeable to tax during the previous year. He (i) Deduction is not allowed to an assessee
approaches you to file his income tax return having income from business.
showing ₹ 2,00,000 as his gross total income. (ii) The maximum amount of deduction
Do you agree with the view of Mr. Krishna? Also, allowed is ₹ 2,000.
compute the amount of his total income. (iii) 100% deduction is allowed if amount in
(a) Yes, as income actually left in Mr. excess of ₹ 2,000 donated is paid by any
Krishna‟s hands is ₹ 2,00,000 only. His mode other than cash.
total income shall be ₹ 2,00,000.
 
20 Prepared by : CA Sanchit Grover
  Multiple Choice Questions
  for CA Inter Taxation For May 2023 Exams

(iv) Deduction is not allowed to an assessee (vi) Any sum paid to a notified Urban
having income from salaries. Development Fund is allowed.
(v) Any sum paid to a University to be used (a) (i), (iii), (iv), (v), (vi)
for scientific research is allowed if such (b) (ii), (iii), (v)
University is approved u/s 35(1)(ii). (c) (i), (ii)
(d) (i), (iii), (v) (MTP Oct 2019)

Answers to Chapter 5

Ans. 1) Option (b) is correct a) ₹ 5,000 x 12m = ₹ 60,000


Premium paid by way of cash is not eligible for 80D b) 25% of Adjusted GTI = ₹ 1,48,750
deduction. In respect of ₹ 52,000 premium paid through c) Rent Paid – 10% of Adjusted GTI
cheque, maximum deduction allowable u/s 80D shall be ₹ = 1,08,000 – 59,500 = ₹ 48,500
50,000 since assessee is a senior citizen
Ans. 8) Option (c) is correct
Deduction u/s 80GG shall be lease of following 3:-
Ans. 2) Option (d) is correct a) ₹ 5,000 x 12m = ₹ 60,000
Deduction u/s 80GG is allowed only to an assessee who is b) 25% of Adjusted GTI = ₹ 1,15,000
not in receipt of HRA or concessional accommodation from c) Rent Paid – 10% of Adjusted GTI
his employer. = 1,44,000 – 46,000 = ₹ 98,000

Ans. 3) Option (d) is correct Ans. 9) Option (b) is correct


Since education loan has been taken higher education of
In respect of a life insurance policy taken on or after 01-04-
spouse, interest paid during PY 22-23 shall be eligible for
2012, maximum deduction allowed u/s 80C in any PY shall
deduction u/s 80E. Hence Total Income = ₹ 6,00,000 - ₹
be 10% of Sum assured.
65,000 = ₹ 5,35,000

Ans.4) Option (d) is correct Ans. 10) Option (d) is correct


As per Sec 80G, any donation above ₹ 2,000 shall be Premium paid for Self and Spouse 25,000
eligible for deduction u/s 80G only if it is paid by any (restricted to max. of ₹ 25,000)
medium other than cash. Premium for father (since he is a senior 43,000
citizen, max limit shall be ₹ 50,000)
Ans. 5) Option (d) is correct Preventive health care checkup 4,000
Sec 80DD allows ad-hoc deduction of ₹ 1,25,000 Total deduction u/s 80D 72,000
(irrespective of actual expenditure incurred) in case of
Ans. 11) Option (c) is correct
dependent disabled having severe disability. As per Sec 80G, any donation above ₹ 2,000 shall be
eligible for deduction u/s 80G only if it is paid by any
Ans. 6) Option (c) is correct medium other than cash. Hence, in the given case, no
Donation to PM Cares Fund (100% 50,000 deduction shall be allowed in respect of ₹ 8L donation
deduction allowed) made by Mr. Krishna
Donation to Rajiv Gandhi Foundation 10,000
Ans. 12) Option (c) is correct
(50% deduction allowed)
Sec 80TTB allows deduction to any resident senior citizen in
Donation to trust in cash (no deduction Nil respect of Interest income on any deposits with any
is allowed if amount > 2000 is paid in bank/cooperative bank or Post office. No deduction is
cash) allowed in respect of Interest received from NBFC. Hence, in
Total eligible deduction 60,000 this case, Mr. Krishna shall be eligible for deduction only in
respect of ₹ 20,000 interest received from cooperative bank.
Ans. 7) Option (a) is correct
Deduction u/s 80GG shall be lease of following 3:- Ans. 13) Option (d) is correct

  
Prepared by : CA Sanchit Grover 21
   CHAPTER 6 : INCOME UNDER THE HEAD SALARY

CHAPTER 6 INCOME UNDER THE HEAD SALARY


Q1) For the purposes of computing exemption under Q3) X is an employee of Z Ltd who receives
section 10(10), in case of Mr. Anand, an ₹1,25,000 as gratuity (he is covered under the
employee of ABC Ltd., who is covered by the Payment of Gratuity Act, 1972). He retires on
Payment of Gratuity Act, 1972, "salary" includes– 31.01.2023 after service of 29 years and 8
(a) only basic pay months. At the time of retirement, X drew
(b) basic pay and dearness allowance, if monthly salary of ₹ 5,200 and monthly bonus of
provided in the terms of employment ₹ 2,000. Compute the amount of gratuity
(c) basic pay and dearness allowance exempt from tax in the instant case u/s 10(10)
(d) basic pay, dearness allowance and of the Income-tax Act, 1961.
commission as a fixed percentage of a) ₹ 90,000
turnover b) ₹ 1,25,000
Q2) Mr. Jagat is an employee in accounts c) ₹ 78,000
department of Bharat Ltd., a cellular company d) ₹ 87,000 (MTP March 2019)
operating in the regions of eastern India. It is Q4) Which of the following incomes are exempt
engaged in manufacturing of cellular devices. incomes as per the provisions of Income-tax Act,
During F.Y. 2022-23, following transactions were 1961?
undertaken by Mr. Jagat: (i) Allowance paid by Government to a
(i) He attended a seminar on “Perquisite citizen of India for rendering services
Valuation”. Seminar fees of ₹ 12,500 was outside India
paid by Bharat Ltd. (ii) Death-cum-retirement gratuity received
(ii) Tuition fees of Mr. Himanshu (son of Mr. by a government employee
Jagat) paid to private coaching classes (iii) Any sum received under a life insurance
(not having any tie-up with Bharat Ltd.) policy taken on 01.05.2016, if the
was reimbursed by Bharat Ltd. Amount of premium payable for any of the years
fees was ₹ 25,000.. exceeds 10% of the actual capital sum
(iii) Ms. Sapna (daughter of Mr. Jagat) studies assured.
in DPS Public School (owned and (iv) Any payment from National Pension
maintained by Bharat Ltd.). Tuition fees System Trust to an employee on account
paid for Ms. Sapna was ₹ 750 per month of closure of his NPS account.
by Mr. Jagat. Cost of education in similar (a) (i), (ii), (iii), (iv)
institution is ₹ 5,250 per month (b) (i) & (ii)
Compute the amount which is (c) (i), (ii) & (iv)
chargeable to tax under the head (d) (ii) & (iv) (MTP Oct 2019)
“Salaries” in hands of Mr. Jagat for A.Y. Q5) Anirudh stays in New Delhi. His basic salary is
2023-24. ₹ 10,000 p.m., D.A. (60% of which forms part of
(a) ₹ 25,000 pay) is ₹ 6,000 p.m., HRA is ₹ 5,000 p.m. and he
(b) ₹ 37,500 is entitled to a commission of 1% on the
(c) ₹ 66,500 turnover achieved by him. Anirudh pays a rent
(d) ₹ 79,000 of ₹ 5,500 p.m. The turnover achieved by him

 
22 Prepared by : CA Sanchit Grover
  Multiple Choice Questions
  for CA Inter Taxation For May 2023 Exams

during the current year is ₹ 12 lakhs. The (d) ₹ 3,58,800 (MTP March 2022)
amount of HRA exempt under section 10(13A) Q9) Mr. Kashyap received basic salary of ₹ 20,000
is – p.m. from his employer. He also received
(a) ₹ 48,480 children education allowance of ₹ 3,000 for
(b) ₹ 45,600 three children and transport allowance of ₹
(c) ₹ 49,680 1,800 p.m. Assume he is not opting to pay tax
(d) ₹ 46,800 under section 115BAC. The amount of salary
Q6) Mr. Dutta received voluntary retirement chargeable to tax for P.Y. 2022-23 is –
compensation of ₹ 7,00,000 after 30 years 4 (a) ₹ 2,62,600
months of service. He still has 6 years of service (b) ₹ 2,12,600
left. At the time of voluntary retirement, he was (c) ₹ 2,11,600
drawing basic salary ₹ 20,000 p.m.; Dearness (d) ₹ 2,12,200
allowance (which forms part of pay) ₹ 5,000 p.m. Q10) Mr. Karan completed his MBA in April 2022 and
Compute his taxable voluntary retirement joined XYZ Ltd from 01.05.2022. His basic salary
compensation, assuming that he does not claim is ₹ 2,25,000 p.m. He is paid 12% of basic salary
any relief under section 89 – as D.A forming part of retirement benefits. He
(a) ₹ 7,00,000 contributed 11% of his pay and D.A. towards
(b) ₹ 5,00,000 recognized provident fund and the company
(c) ₹ 2,00,000 contributes the same amount. Accumulated
(d) Nil interest on provident fund as on 31.3.2023 is ₹
Q7) Anand is provided with furniture to the value of 49,325. What would be the income chargeable
₹ 70,000 along with house from February, 2022. to tax under the head “Salaries” of Mr. Karan for
The actual hire charges paid by his employer for the A.Y. 2023-24, if he does not opt for section
hire of furniture is ₹ 5,000 p.a. The value of 115BAC?
furniture to be included along with value of (a) ₹ 27,26,442
unfurnished house for A.Y. 2023-24 is- (b) ₹ 27,30,884
(a) ₹ 5,000 (c) ₹ 27,22,000
(b) ₹ 7,000 (d) ₹ 27,71,325
(c) ₹ 10,500 Q11) XYZ Pvt. Ltd. provides a car (below 1.6 ltr cc)
(d) ₹ 14,000 along with a driver to Mr. Sanjay, employee of
Q8) The basic salary of Mr. Raj is ₹ 1,15,000 p.m. He XYZ Pvt. Ltd., partly for official and partly for
is entitled to dearness allowance, which is 30% personal purpose. The expenses incurred by the
of basic salary which forms part of pay for company are: Running and maintenance
retirement benefits. Mr. Raj and his employer, expenses – ₹ 32,000 and driver’s salary – ₹
XYZ Ltd., both contribute 20% of basic salary to 36,000. The taxable value of car facility for A.Y.
the pension scheme referred to in section 2023-24 will be –
80CCD. What is the amount of deduction (a) ₹ 21,600
available to Mr. Raj under section 80CCD for (b) ₹ 10,800
A.Y. 2023-24? (c) ₹ 32,400
(a) ₹ 4,08,800 (d) ₹ 39,600
(b) ₹ 5,05,400 Q12) Mr. Arpit, an employee of MNO Ltd. has
(c) ₹ 3,79,400 contributed ₹ 1,61,280 towards NPS and similar
  
Prepared by : CA Sanchit Grover 23
   CHAPTER 6 : INCOME UNDER THE HEAD SALARY
amount is contributed by his employer. His 23. How much deduction is available under
basic salary is ₹ 80,000 p.m. and dearness Chapter VI-A while computing total income of
allowance is 40% of basic salary which forms Mr. Arpit for the A.Y. 2023-24?
part of retirement benefits. He also paid ₹ (a) ₹ 3,46,280
55,000 towards LIC premium for himself and his (b) ₹ 3,69,400
wife and medical insurance premium of ₹ (c) ₹ 3,19,400
35,000 by crossed cheque for his mother, being (d) ₹ 3,96,280 (RTP Dec 2021)
a senior citizen during the previous year 2022-

Answers to Chapter 6

Ans. 1) Option (c) is correct c) Rent paid (66,000) – 10% of Salary 48,480
(17,520)
Ans. 2) Option (d) is correct
a) Seminar on Perquisite valuation Nil Ans. 6) Option (C) is correct
b) Reimbursement of Tuition fees of 25,000 Amount exempt u/s 10(10C) shall be least of 4:-
Son (Fully taxable) a) Amt. actually received 7,00,000
c) Perquisite Valuation for education 54,000 b) Maximum amt Specified 5,00,000
of daughter** 63,000 c) 3m x (20,000 + 5,000) x 30 years 22,50,000
Less:- Fees recovered (9,000) d) (20,000 + 5000) x 6 x 12 m 18,00,000
Taxable Salary 79,000
Taxable Compensation = 7,00,000 – 5,00,000 = ₹
** Alternative viewpoint is that only the amount in excess of 2,00,000
₹ 1,000 p.m. shall be considered for perquisite valuation.
Valuation in such case shall be (5,250 – 1,000 – 750) x 12 Ans. 7) Option (A) is correct
months = ₹ 42,000 Perquisite value of furniture = Hire charges paid or payable
by the employer = ₹ 5,000
Ans. 3) Option (a) is correct
Gratuity exempt shall be least of 3:- Ans. 8) Option (c) is correct
a) Amt. actually received 1,25,000 Deduction allowed u/s 80CCD(2) in 1,79,400
b) Max amt. of exemption 20,00,000 respect of employer’s contribution [10%
c) 5,200 x 30 x 15/26 90,000 of (13,80,000 + 4,14,000)]
Gratuity exempt (least of 3) 90,000 Deduction u.s 80CCD(1) in respect of 1,50,000
employee’s contribution
Ans. 4) Option (b) is correct (restricted to maximum of ₹ 1,50,000
In respect of point (iii), since the premium exceeds 10% of assuming there is no deduction u/s
sum assured, exemption u/s 10(10D) shall not be 80C & 80CCC)
applicable. Further, in respect of NPS, only 60% of the Balance Deduction in respect of 50,000
amount received is exempt employee’s contribution allowed u/s
80CCD(1B)
Ans. 5) Option (a) is correct Total Deduction u/s 80CCD 3,79,400
HRA exempt shall be lease of 3:-
a) HRA actually received 60,000
b) 50% of (1,20,000 + 43,200 + 12,000) 87,600

 
24 Prepared by : CA Sanchit Grover
  Multiple Choice Questions
  for CA Inter Taxation For May 2023 Exams
Ans. 9) Option (B) is correct Interest taxable = 54,920 x 8.08%
Basic Salary 2,40,000 Gross Salary 27,76,442
Children Education 1,000 Less:- Standard deduction u/s 16 (50,000)
Allowance 3,000 Income u/h Salary 27,26,442
Less:- Exempt 2,000
Transport allowance (1800 x12) 21,600 Ans. 11) Option (C) is correct
Gross Salary 2,62,600 Since the car belongs to employer, expenses are also met
by employer and it is being used partly for official and partly
for personal purpose, value of the perquisite shall be = (₹
Ans. 10) Option (A) is correct 1800 + ₹ 900) x 12 months = ₹ 32,400
Basic Salary (2,25,000 x 11) 24,75,000
Dearness Allowance (12%) 2,97,000 Ans. 12) Option (B) is correct
Employer’s contribution to Provident Nil
Deduction allowed u/s 80C in respect of life 55,000
fund (3,04,920) – since it is less than
insurance premium of self & spouse
11% of Basic Salary & DA, nothing shall
Deduction allowed u/s 80D in respect of 35,000
be taxable. Further, since employer’s
contribution is less than ₹ 7L, nothing medical insurance premium for mother
is taxable u/s 17(2)(vii) (max limit will be ₹ 50,000 since she is
Interest received on RPF in relation to Exempt senior citizen)
employer’s contribution (49,325 x 0.5 / Deduction u/s 80CCD(2) in respect of 1,34,400
3,04,920 = 8.08%). Since it is less than Employer’s contribution to NPS (Max being
9.5% and also Sec 17(2)(via) is not 10% of ₹ 13,44,000)
applicable here, nothing is taxable Deduction u/s 80CCD(1) in respect of own 95,000
Interest received on RPF in relation to 4,442 contribution (Aggregate deduction u/s 80C,
emplyees contribution (Since 80CCC & 80CCD(1) cannot be > ₹ 1.5L)
contribution is more than 2,50,000,
Deduction u/s 80CCD(1B) in respect of own 50,000
interest on ₹ (304920 – 2,50,000) shall
contribution to NPS
become taxable
Total Deduction under Chapter VI-A 3,69,400

  
Prepared by : CA Sanchit Grover 25
   CHAPTER 7 : INCOME UNDER THE HEAD ‘HOUSE PROPERTY’

CHAPTER 7 INCOME UNDER THE HEAD ‘HOUSE PROPERTY’


Q1) In respect of loss from house property, which of (a) ₹ 30,000 each
the following statements are correct (assuming (b) ₹ 2,00,000 each
provisions of Sec 115BAC are not opted)? (c) ₹ 2,50,000 each
(a) While computing income from any house (d) ₹ 5,00,000 each
property, the maximum interest deduction Q4) Vidya received ₹ 90,000 in May, 2022 towards
allowable under section 24 is ₹ 2 lakhs recovery of unrealized rent, which was deducted
(b) Loss from house property relating to a from actual rent during the P.Y. 2020-21 for
particular year can be set-off against determining annual value. Legal expense
income under any other head during that incurred in relation to unrealized rent is ₹
year only to the extent of ₹2 lakhs 20,000. The amount taxable under section 25A
(c) The loss in excess of ₹2 lakh, which is not for A.Y. 2023-24 would be –
set-off during the year, can be carried (a) ₹ 70,000
forward for set-off against any head of (b) ₹ 63,000
income in the succeeding year(s) (c) ₹ 90,000
(d) All of the above (d) ₹ 49,000
Q2) Mr. P has a house property in Delhi whose Q5) Ms. Sheetal and her brother jointly own a
Municipal value is ₹1,00,000 and the Fair bungalow. They had taken a housing loan to
Rental Value is ₹ 1,20,000. The standard rent is purchase the bungalow. The loan is sanctioned
fixed at ₹1,08,000. It was self-occupied by Mr. P in the name of Ms. Sheetal and her brother in
from 01.04.2022 to 31.07.2022. With effect from the year 2015. Interest on housing loan for the
01.08.2022, it was let out at ₹ 10,000 per month. P.Y. 2022-23 amounted to ₹ 4,50,000 which is
Compute the net annual value of the house paid by Ms. Sheetal (₹ 2,25,000) and her brother
property for A.Y. 2023-24 if the municipal taxes (₹ 2,25,000). The bungalow is used by them for
paid by him during the year were ₹ 20,000. their residence. In this case, what will be the
(a) ₹1,00,000 amount of deduction available under section
(b) ₹ 88,000 24(b) to Ms. Sheetal and her brother?
(c) ₹ 60,000 a) ₹ 30,000 each
(d) ₹1,08,000 b) ₹ 2,00,000 each
Q3) Mr. Sujal and his brother jointly own a c) ₹ 2,25,000 each
bungalow. They had taken a housing loan to d) ₹ 4,50,000 each (MTP April 2019)
purchase the bungalow. The loan is sanctioned Q6) Ganesh and Rajesh are co-owners of a self-
in the name of Mr. Sujal and his brother in the occupied property. They own 50% share each.
year 2015. Interest on housing loan for the P.Y. The interest paid by each co-owner during the
2022-23 amounted to ₹ 5,00,000 which is paid previous year 2022-23 on loan (taken for
by Mr. Sujal (₹ 2,50,000) and his brother (₹ acquisition of property during the year 2004) is
2,50,000). The bungalow is used by them for ₹ 2,05,000. The amount of allowable deduction
their residence. In this case, what will be the in respect of each co-owner is –
amount of deduction available under section (a) ₹ 2,05,000
24(b) to Mr. Sujal and his brother? (b) ₹ 1,02,500

 
26 Prepared by : CA Sanchit Grover
  Multiple Choice Questions
  for CA Inter Taxation For May 2023 Exams

(c) ₹ 2,00,000 house for self-occupation. The construction of


(d) ₹ 1,00,000 the house began in June, 2020 and was
Q7) Mr. Raghav has three houses for self- completed on 30-6-2022. He has not repaid any
occupation. What would be the tax treatment for amount of loan so far. The amount of interest
A.Y.2023-24 in respect of income from house deduction u/s 24(b) for A.Y. 2023-24 is –
property? (a) ₹ 1,50,000
(a) One house, at the option of Mr. Raghav, (b) ₹ 1,80,000
would be treated as self-occupied. The (c) ₹ 2,00,000
other two houses would be deemed to be (d) ₹ 2,10,000
let out. Q10) Mr. Ritvik has purchased his first house in
(b) Two houses, at the option of Mr. Raghav, Gwalior for self-occupation on 5.4.2022 for ₹ 45
would be treated as self-occupied. The lakhs (stamp duty value being the same) with
other house would be deemed to be let bank loan sanctioned on 30.3.2022 and
out. disbursed on 3.4.2022. He paid interest of ₹ 3.8
(c) One house, at the option of Assessing lakhs during the P.Y. 2022-23. What is the tax
Officer, would be treated as self-occupied. treatment of interest paid by him?
The other two houses would be deemed (a) Interest of ₹ 2 lakhs allowable u/s 24
to be let out. (b) Interest of ₹ 2 lakhs allowable u/s 24 and
(d) Two houses, at the option of Assessing ₹ 1.8 lakhs allowable u/s 80EEA
Officer, would be treated as self-occupied. (c) Interest of ₹ 2 lakhs allowable u/s 24 and
The other house would be deemed to be ₹ 1.5 lakhs allowable u/s 80EEA
let out. (RTP May 2020) (d) Interest of ₹ 1.5 lakhs allowable u/s 24
Q8) In respect of loan of ₹ 40 lakhs sanctioned by and ₹ 1.5 lakhs allowable u/s 80EEA
SBI in April, 2022 for purchase of residential (RTP May 2020)
house intended for self-occupation, compute Q11) Mr. Ashutosh purchased his first dream home
the interest deduction allowable under the in Bangalore on 16.8.2022. He applied for home
provisions of the Act for A.Y. 2023-24, assuming loan of ₹ 40 lakhs from IDFC bank on 15.7.2022,
that the disbursement was made on 1st June, the same was sanctioned by bank on 20.7.2022.
2022, the rate of interest is 8% p.a. and the loan The stamp duty value of the said house was ₹
sanctioned was 80% of the stamp duty value of 44 lakhs. The interest due on the said home
the property. loan is ₹ 3,75,000 for the financial year 2022-
(a) ₹ 2,00,000 u/s 24 and ₹ 1,20,000 u/s 23. Due to liquidity issues, Mr. Ashutosh could
80EEA only pay ₹ 3,26,000. Compute the total interest
(b) ₹ 1,50,000 u/s 80EEA and ₹ 1,70,000 u/s deduction Mr. Ashutosh can claim for the A.Y.
24 2023-24, assuming Mr. Ashutosh doesn’t opt for
(c) ₹ 2,00,000 u/s 24 and ₹ 50,000 u/s the tax rates under the new scheme.
80EEA (a) ₹ 3,26,000
(d) ₹ 2,00,000 u/s 24 (b) ₹ 2,00,000
Q9) Mr. Vikas took a loan of ₹ 15,00,000 @10% p.a. (c) ₹ 3,75,000
on 1-4-2020 for the construction of residential (d) ₹ 3,50,000 (MTP Oct 2021)

  
Prepared by : CA Sanchit Grover 27
   CHAPTER 7 : INCOME UNDER THE HEAD ‘HOUSE PROPERTY’
Answers to Chapter 7

Ans. 1) Option (b) is correct acquisition of property has been done within 5 years from
The maximum limit for deduction of interest u/s 24(b) is ₹ end of PY in which loan has been taken).
30,000 or ₹ 2,00,000 only in respect of Self-occupied
property. There is no such limit for Let-out property. Ans. 7) Option (b) is correct

Ans. 2) Option (b) is correct Ans. 8) Option (d) is correct


Reasonable Expected Rent 108,000 Total Interest paid during PY 22-23 = ₹ 3,20,000. SDV of HP
Actual rent received/receivable for the 80,000 = ₹ 40L/80% = ₹ 50L. Since HP is self occupied, maximum
period during which it is available for of ₹ 2L deduction shall be allowable u/s 24(b). Further since
let out (10,000 x 8) SDV > 45L, no deduction shall be allowable u/s 80EEA
GAV (being higher of 2) 1,08,000
Less:- Municipal taxes (20,000) Ans. 9) Option (c) is correct
NAV 88,000 Pre construction period = 01-04-2020 to 31-03-2022 =
15,00,000 x 10% x 2 years = ₹ 3,00,000
Ans. 3) Option (b) is correct Deduction u/s 24(b) during PY 22-23 = Interest accrued
In case of co-owners, maximum deduction u/s 24(b) in during PY 22-23 + Pre acquisition period Interest/5 =
respect of Interest shall be 2,00,000/30,000 separately for 1,50,000 + 3,00,000/5 = ₹ 2,10,000
each co-owner. In the given case, it is assumed that However, since the house is used for self occupation,
acquisition of property has been done within 5 years from maximum deduction allowed will be ₹ 2,00,000
end of PY in which loan has been taken & hence ₹ 2,00,000
is the maximum limit Ans. 10) Option (c) is correct
Since loan has been sanctioned between 01-04-2019 to 31-
Ans. 4) Option (b) is correct 03-22, and SDV of HP is not more than 45L, deduction u/s
Amt of unrealized rent is taxable in the year of receipt after 80EEA shall be allowable up to maximum of ₹ 1,50,000.
deducting 30% in respect of Standard deduction . This shall be over & above the deduction of ₹ 2L that is
allowed u/s 24(b).
Ans. 5) Option (b) is correct
Since the HP is self-occupied, maximum deduction u/s 24(b) Ans. 11) Option (d) is correct
shall be ₹ 2,00,000 in case of both co-owners Since conditions of Sec 80EEA are satisfied, ₹ 1,50,000 shall
be eligible for deduction u/s 80EEA. Further, Sec 24 allows
Ans. 6) Option (c) is correct deduction of interest up to maximum of ₹ 2,00,000 on
In case of co-owners, maximum deduction u/s 24(b) in accrual basis. Hence irrespective of amount of interest paid,
respect of Interest shall be 2,00,000/30,000 separately for total deduction allowed shall be ₹ 3,50,000
each co-owner. In the given case, it is assumed that

 
28 Prepared by : CA Sanchit Grover
  Multiple Choice Questions
  for CA Inter Taxation For May 2023 Exams

CHAPTER 8 INCOME UNDER THE HEAD ‘OTHER SOURCES’


Q1) Mr. X, aged 61 years, earned dividend of ₹ P.Y. 2022-23 and taxable, if any, under which
12,00,000 from ABC Ltd. in P.Y. 2022-23. Interest head of income.
on loan taken for the purpose of investment in (a) ₹ 2,50,000 under the head “income from
ABC Ltd., is ₹ 3,00,000. Income included in the other sources”
hands of Mr. X for P.Y. 2022-23 would be – (b) ₹ 4,00,000 under the head “income from
(a) ₹ 12,00,000 other sources”
(b) ₹ 9,60,000 (c) ₹ 1,60,000 under the head “income from
(c) ₹ 9,00,000 other sources”
(d) ₹ 2,00,000 (d) ₹ 1,60,000 under the head “Capital gains”
Q2) Neeraj was working as an accountant with the (MTP Nov 21)
company Ujala Ltd. He died on 30.04.2022 and Q5) A member of parliament is entitled to salary,
on account of his death, his wife Neha started constituency allowance and daily allowance
receiving a pension of ₹10,000 per month w.e.f. when the Parliament is in session. Which of the
01.06.2022. Determine under which head following statements are correct?
of income, the pension received by Neha (a) His entire income is taxable under the
during F.Y. 2022-23 shall be taxable. Also, head "Salaries"
compute the taxable amount in her hands. (b) Only his salary component is taxable
a) Income from other sources: ₹1,00,000 under the head "Salaries". Constituency
b) Income from other sources: ₹85,000 allowance and daily allowance are
c) Income from Salary: ₹1,00,000 exempt.
d) Income from Salary: ₹85,000 (MTP Oct (c) Only his salary component is taxable
2019) under the head "Income from other
Q3) Mr. Harish, aged 40 years, earned income of sources". Constituency allowance and
₹15,000 during P.Y. 2022-23 from post office daily allowance are exempt
savings bank account. What portion of such (d) His salary component and constituency
income would be ultimately included in his total allowance is taxable under the head
income? "Income from other sources". Daily
(a) ₹15,000 allowance is exempt.
(b) ₹5,000 (MTP April 2019)
(c) ₹1,500 Q6) Mrs. Gupta, resident in India, holds many
(d) Nil equity shares of reputed domestic companies.
Q4) Ms. Shalini received interest on enhanced During the previous year 2022-23, total
compensation of ₹ 5,00,000. Out of this interest, dividend earned by her is ₹11,00,000. She is
₹ 1,50,000 relates to the previous year 2020-21, of the belief that dividend income earned by
₹ 1,90,000 relates to previous year 2021-22 and her is tax free. She approaches you to assist
₹ 1,60,000 relates to previous year 2022-23. She her in filing her income tax return. As her tax
paid ₹ 1 lakh to her advocate for his efforts in consultant, will you advise her that any
the matter. What amount would be taxable in dividend income earned by her is tax free?

  
Prepared by : CA Sanchit Grover 29
   CHAPTER 8 : INCOME UNDER THE HEAD ‘OTHER SOURCES’
(a) Yes, as dividend earned by her is fully (d) Correct, as income from subletting of a
exempt from tax u/s 10(34). property is directly attributable to the
(b) No, as any dividend income earned by an property itself and hence, chargeable to
individual is fully chargeable to tax. tax as income from house property.
(c) No, as dividend income earned above Q8) APM Ltd. is a pioneer company in textile
₹10,00,000 is chargeable to tax in her industry. At the end of F.Y. 2022-23, it decided
hands. to distribute deposit certificates (without
(d) Yes, as dividend income above interest) to its shareholders (preference as well
₹10,00,000 is chargeable to tax only in as equity shareholders). Total value of
the hands of the companies and not in accumulated profits of APM Ltd. was ₹ 25 lakhs.
her hands. (MTP March 2019) Mr. A is an equity shareholder of APM Ltd.
Q7) Mr. Agarwal moved to Mumbai. He took a holding 10% of share capital. During F.Y. 2022-
property on rent for his residential purpose. 23, Mr. A received deposit certificates (without
However, the property was not fully occupied by interest) valuing ₹ 5,00,000 from APM Ltd.
him. He let out the property to his friend at ₹ Comment upon taxability of receipt of deposit
15,000 p.m. from 01.04.2022 to 31.03.2023. Mr. certificates in the hands of Mr. A.
Agarwal is of the view that income from (a) Deposit Receipts (without interest) are
subletting of property is taxable as Income from taxable to the extent of ₹2,50,000 under
House Property. As tax advisor of Mr. Agarwal, Income from other sources.
find out whether his view is correct? (b) Deposit Receipts (without interest) are
(a) Correct, as any income from a house fully taxable under Income from other
property is taxable under the head sources.
Income from House Property. (c) Deposit Receipts (without interest) are
(b) Incorrect, as Mr. Agarwal is not the owner exempt since DDT is payable by the
of the property let out by him. The income company.
from subletting shall be taxable under the (d) Deposit Receipts (without interest) are
head Profits and Gains of Business or fully taxable and shall be included in
Profession. Gross total income. But such receipt shall
(c) Incorrect, as Mr. Agarwal is not the owner be allowed as deduction under Chapter-
of the property let out by him. The income VI A.
from subletting shall be taxable under the
head Income from other sources.

Answers to Chapter 8

Ans. 1) Option (b) is correct Ans. 2) Option (b) is correct


As per proviso to Sec 57, deduction in respect of interest Family pension received by family members of deceased
expense cannot be more than 20% of Dividend income employee shall be taxable under the head Other Sources.
received. Hence Net Dividend income taxable would be ₹ Hence, amount taxable in hands of Mr. Neeraj shall be as
12,00,000 – 2,40,000 = ₹ 9,60,000 under:-

 
30 Prepared by : CA Sanchit Grover
  Multiple Choice Questions
  for CA Inter Taxation For May 2023 Exams

Family Pension received 1,00,000 under the head Other Sources. Further, daily allowance &
(10,000 x 10 months) constituency allowance shall be exempt u/s 10(17)
Less:- Deduction u/s 57 (being lower of (15,000)
following 2:-) Ans. 6) Option (b) is correct
rd
a) 1/3 of ₹ 1L = ₹ 33,333 Dividend income is taxable as normal income in hands of
b) ₹ 15,000 shareholder under the head Other Sources. Provisions of
Amt. taxable under the head Other 85,000 Sec 10(34) and Sec 115BBDA have been discontinued
Sources
Ans. 7) Option (c) is correct
Ans. 3) Option (c) is correct As long as recipient of rent is not owner of house property,
In respect of Interest on PO Saving A/c, exemption of ₹ the same cannot be taxed u/h Income from House Property.
3,500 is allowed u/s 10(15) and thereafter deduction u/s Further, since Mr. Agarwal is not in full time business of
80TTA can also be allowed up to maximum of ₹ 10,000. letting out properties, hence income from sub letting shall
be taxable u/h Other Sources.
Ans. 4) Option (a) is correct
Interest on compensation or enhanced compensation is Ans. 8) Option (a) is correct.
taxable on receipt basis. Further, deduction of 50% of As per Sec 2(22)(b) any distribution of bonus deposit
Interest is allowed u/s 57. certificates to shareholders shall be treated as dividend in
their hands to the extent of accumulated profits of the
Ans. 5) Option (c) is correct company. Hence, in this case, ₹ 2,50,000 of deposit
In case of MPs & MLAs since employer-employee certificates would be taxable u/h Other sources in hands of
relationship doesn’t exist, hence their salary is taxable shareholder.

  
Prepared by : CA Sanchit Grover 31
   CHAPTER 9 : INCOME UNDER THE HEAD PGBP

CHAPTER 9 INCOME UNDER THE HEAD PGBP


Q1) An electricity company charging depreciation (a) Nil
on straight line method on each asset (b) Market value of the asset on the date of
separately, sells one of its machinery in April, transfer to business
2022 at ₹ 1,20,000. The WDV of the machinery (c) ₹ 2,75,000 less notional depreciation
at the beginning of the year i.e., on 1st April, under section 32 up to the date of
2022 is ₹ 1,35,000. No new machinery was transfer.
purchased during the year. The shortfall of ₹ (d) Actual cost of the asset i.e., ₹ 2,75,000
15,000 is treated as– Q4) Mr. C aged 35 years is a working partner in M/s
(a) Terminal depreciation BCD, a partnership firm, with equal profit
(b) Short-term capital loss sharing ratio. During the P.Y. 2022-23, the firm
(c) Normal depreciation has paid remuneration to Mr. B, Mr. C and Mr.
(d) Any of the above, at the option of the D, being the working partners of the firm, of ₹
assessee 2,00,000 each. The firm has paid interest on
Q2) Match the following to their respective rate of capital of ₹ 1,20,000 in toto to all the three
depreciation. partners and the same is within the prescribed
L. Pollution control 1. 10% limit of 12%. The firm had a loss of ₹ 1,12,000
equipment after debiting remuneration and interest on
M. Commercial building 2. 40% capital.
N. Oil Wells 3. 100% Note – Remuneration and interest on capital is
4. 15% authorized by the partnership deed
You, being the CA of Mr. C, are in the process of
Select the correct answer from the options
computing his total income. What would be his
given below:
taxable remuneration from the firm?
L. M. N.
(a) ₹ 2,00,000
(a) 2 1 4
(b) ₹ 1,51,600
(b) 4 2 1
(c) ₹ 1,27,600
(c) 2 4 1
(d) ₹ 1,50,000 (MTP April 2021)
(d) 3 1 4
Q5) The W.D.V. of a block (Plant and Machinery, rate
(MTP Oct 2019) of depreciation 15%) as on 1.4.2022 is ₹
Q3) Mr. X acquires an asset in the year 2016-17 for 3,20,000. A second hand machinery costing ₹
the use for scientific research for ₹ 2,75,000. He 50,000 was acquired on 1.9.2022 through
claimed deduction under section 35(1)(iv) in the account payee cheque but put to use on
previous year 2016-17. The asset was brought 1.11.2022. During January 2023, part of this
into use for the business of Mr. X in the P.Y. block was sold for ₹ 2,00,000. The depreciation
2022-23, after the research was completed. The for A.Y. 2023-24 would be –
actual cost of the asset to be included in the (a) ₹ 21,750
block of assets is:- (b) ₹ 25,500
(c) ₹ 21,125
(d) ₹ 12,750

 
32 Prepared by : CA Sanchit Grover
  Multiple Choice Questions
  for CA Inter Taxation For May 2023 Exams

Q6) M/s ABC, an eligible assessee, following Kumar in respect of a particular debtor of cloth
mercantile system of accounting, carrying on business, ₹ 60,000 was allowed by the
eligible business under section 44AD provides Assessing Officer in P.Y. 2021-22. On
the following details: 29.01.2023, Mr. Kumar received a sum of ₹
♦ Total turnover for the financial year 2022- 68,000 from the debtor in full and final
23 is ₹ 130 lakh settlement. How much amount would be
♦ Out of the above: taxable in the hands of Mr. Kumar for A.Y. 2023-
- ₹ 25 lakh received by A/c payee cheque 24?
during the financial year 2022-23; (a) ₹ 28,000
- ₹ 50 lakh received by cash during the (b) ₹ 20,000
financial year 2022-23; (c) ₹ 60,000
- ₹ 25 lakh received by A/c payee bank (d) ₹ 68,000 (MTP March 2022)
draft before the due date of filing of Q9) Mr. Raja, a proprietor, commenced operation of
return; the business of a new three star hotel in
- ₹ 30 lakh not received till due date of Mumbai on 1.7.2022. He had made a total
filing of return. investment of ₹ 7.58 crores till 30.6.2022. Out of
What shall be the amount of deemed profits of total investment of ₹ 7.58 crores, ₹ 1.58 crores
M/s ABC under section 44AD(1) for A.Y. 2023- was used for purchase of land in P.Y.2022-23. ₹
24? 4.70 crores was used for constructing Hotel and
(a) ₹ 10.4 lakh balance of ₹ 1.30 used for purchasing the
(b) ₹ 7.0 lakh furniture in P.Y. 2022-23. He wants to avail the
(c) ₹ 5.5 lakh benefit of deduction under section 35AD as he
(d) ₹ 9.4 lakh satisfied with all the conditions prescribed u/s
Q7) Mr. Ravi incurred loss of ₹ 4 lakh in the P.Y. 35AD. His profit and gains from the business for
2022-23 in retail trade business. Against which P.Y. 2022-23 is ₹ 50 lakhs before claiming
of the following incomes earned during the deduction u/s 35AD. He wants to file his
same year, can he set-off such loss? income-tax return on 12.12.2023. How much
(a) profit of ₹ 1 lakh from wholesale cloth deduction Mr. Raja can claim for A.Y. 2023-24
business and the losses which he can carry forward to
(b) long-term capital gains of ₹ 1.50 lakhs A.Y. 2024-25? (MTP April 2022)
on sale of land (a) He can claim the deduction of ₹ 7.58
(c) Speculative business income of ₹ 40,000 crores from his business income but he
(d) All of the above would not be able to carry forward the
Q8) Mr. Kumar, engaged in wholesale business of business loss of ₹ 7.08 crores
clothes and speculative business, discontinued (b) He can claim the deduction of ₹ 6.00
its operations on 19.10.2022 and 30.09.2022, crores from his business income and can
respectively. The cloth business loss up to carry forward the business loss of ₹ 5.50
19.10.2022 for P.Y. 2022-23 was ₹ 8,000 and (c) He can claim the deduction of ₹ 6.00
speculative business loss up to 30.09.2021 for crores from his business income but
P.Y. 2022-23 was ₹ 40,000. Out of total bad cannot carry forward the business loss of
debts of ₹ 1,00,000 that were claimed by Mr. ₹ 5.50 crores

  
Prepared by : CA Sanchit Grover 33
   CHAPTER 9 : INCOME UNDER THE HEAD PGBP
(d) He can claim the deduction of ₹ 7.58 (c) ₹1,50,000
crores from his business income and can (d) Nil
carry forward the business loss of ₹ 7.08 Q13) An assessee purchases furniture worth ₹ 80,000
crores on 05.09.2022 and makes the payment of ₹
Q10) The benefit of payment of advance tax in one 45,000 by account payee cheque and ₹ 20,000
installment on or before 15th March is available in cash on the same date. The balance of ₹
to assessees computing profits on presumptive 15,000 is paid by the assessee by bearer cheque
basis – on 06.09.2022 when the furniture is delivered in
(a) under section 44AD his office. Compute the amount of actual cost of
(b) under section 44AD and 44ADA furniture to the assessee.
(c) under section 44AD and 44AE (a) ₹ 45,000
(d) under section 44AD, 44ADA and 44AE (b) ₹ 80,000
Q11) M/S Thakural & Sons, paid ₹ 11,00,000 as (c) ₹ 60,000
remuneration to its partner. The same was in (d) ₹ 65,000
accordance with partnership deed. Partners are Q14) X Ltd. credits a sum of ₹ 45,000 as commission
also entitled to interest on capital @ 11% as per to Y Ltd., an Indian company on 25.06.2022
partnership deed. Total interest paid during the without deducting tax at source. Y Ltd. paid its
year is ₹1,30,000. The book profit before interest entire tax liability on its income by way of
on capital and remuneration is ₹ 37,00,000. The advance payment of tax during P.Y. 2022-23
salary allowable as deduction to M/S Thakural and filed its return of income for A.Y. 2023-24
& Sons is: on 15.07.2023. X Ltd. also has a certificate in
(a) ₹ 22,62,000 Form No. 26A from a chartered accountant.
(b) ₹ 11,00,000 Compute the amount of deduction that shall be
(c) ₹ 23,10,000 allowed to X Ltd. in respect of the commission
(d) ₹ 22,32,000 credited to Y Ltd. for PY 22-23 assuming that X
Q12) M/s PQR & Co., a firm carrying on business, Ltd. follows mercantile system of accounting.
furnishes the following particulars for the P.Y. (a) ₹ 45,000
2022-23. (b) Nil
Particulars ₹ (c) ₹ 13,500
Book profits (before setting of 2,70,000 (d) ₹ 31,500
unabsorbed depreciation and Q15) A, a resident individual, is engaged in the
brought forward business loss) business of money lending. For the purpose
Unabsorbed depreciation of P.Y. 1,20,000 of lending money to various persons, A
2016-17 borrows money from other persons. As a part
Brought forward business loss 2,00,000 of his business, A took a loan from B of an
of P.Y. 2021-22 amount of ₹ 10 lacs. B is a non-resident. On
the said loan, A paid an amount of ₹ 1 lac
Compute the amount of remuneration
as interest during the P.Y. 2022-23 to B in
allowable under section 40(b) from the book
India. A did not deduct tax at source while
profit.
crediting/paying the interest amount to B. A is
(a) ₹ 2,43,000
of the view that the amount of ₹1 lac shall be
(b) ₹1,80,000
allowed to him as a deduction under the
 
34 Prepared by : CA Sanchit Grover
  Multiple Choice Questions
  for CA Inter Taxation For May 2023 Exams

Income-tax Act, 1961. Whether A‟s view is (a) ₹1,59,000


correct? (MTP March 2019) (b) ₹75,000
(a) Correct, interest expenses incurred for (c) Nil
business are allowed as deduction u/s (d) ₹84,000
36(1)(iii). Q18) A, a resident individual, is engaged in the
(b) Incorrect, as tax at source has not been business of money lending. For the purpose of
deducted by A on the interest amount, full lending money to various persons, A borrows
amount of interest of ₹ 1 lac shall be money from other persons. As a part of his
disallowed in A.Y. 2023-24. business, A took a loan from B of an amount of
(c) Incorrect, as tax at source has not been ₹10 lacs. B is a non-resident. On the said loan,
deducted by A on the interest amount, A paid an amount of ₹1 lac as interest during
amount of interest of ₹30,000 shall be the P.Y. 2022-23 to B in India. A did not deduct
disallowed in A.Y. 2023-24 tax at source while crediting/paying the interest
(d) Correct, interest expenses incurred for amount to B. A is of the view that the amount
business are allowed as deduction u/s of ₹1 lac shall be allowed to him as a deduction
37(1). under the Income-tax Act, 1961. Whether A‟s
Q16) Sumit acquired a building for ₹15 lakh in June, view is correct?
2020 in addition to the cost of ₹3 lakh in respect (a) Correct, interest expenses incurred for
of the land on which the building is situated. It business are allowed as deduction u/s
was used for personal purposes until he 36(1)(iii).
commenced business in June, 2022 and since (b) Incorrect, as tax at source has not been
then it was used for business purposes (Assume deducted by A on the interest amount, full
building was meant for commercial purpose amount of interest of ₹ 1 lac shall be
and not residential purpose). The amount of disallowed in A.Y. 2023-24.
depreciation eligible in his case for the A.Y. (c) Incorrect, as tax at source has not been
2023-24 would be: deducted by A on the interest amount,
(a) ₹ 1,50,000 amount of interest of ₹ 30,000 shall be
(b) ₹ 1,45,800 disallowed in A.Y. 2023-24.
(c) ₹ 180,000 (d) Correct, interest expenses incurred for
(d) ₹ 1,21,500 business are allowed as deduction u/s
Q17) M/s Unnati Group, a proprietorship firm 37(1).
incurred the following expenditure during the Q19) Mr. C, aged 35 years, is a working partner in M/s
P.Y. 2022-23: BCD, a partnership firm, with equal profit
• Customs duty of FY 21-22: ₹75,000 paid sharing ratio. During the P.Y. 2022-23, the firm
on 15.5.2022 has paid remuneration to Mr. B, Mr. C and Mr.
• Income tax paid for A.Y. 2022-23: ₹84,000 D, being the working partners of the firm, of ₹
paid on 16.6.2022 2,00,000 each. The firm has paid interest on
Calculate the amount of expenditure capital of ₹ 1,20,000 in toto to all the three
allowable to Unnati Group while partners and the same is within the prescribed
computing its business income for A.Y. limit of 12%. The firm had a loss of ₹ 1,12,000
2023-24. after debiting remuneration and interest on
capital.
  
Prepared by : CA Sanchit Grover 35
   CHAPTER 9 : INCOME UNDER THE HEAD PGBP
Note – Remuneration and interest on capital is that shall be allowed to the firm while
authorized by the partnership deed calculating its business income assuming that
You, being the CA of Mr. C, are in the process of the interest is authorized by the partnership
computing his total income. What would be his deed.
taxable remuneration from the firm? (a) ₹ 42,000
(a) ₹ 2,00,000 (b) ₹ 51,000
(b) ₹ 1,51,600 (c) ₹ 52,500
(c) ₹ 1,27,600 (d) ₹ 43,500 (MTP Nov 21)
(d) ₹ 1,50,000 Q22) Mr. Shahid, a wholesale supplier of dyes,
Q20) Which of the following persons are compulsorily provides you with the details of the following
required to get their accounts audited u/s 44AB cash payments he made throughout the year –
of the Income-tax Act, 1961? - 12.06.2022: Loan repayment of ₹ 27,000
(i) An assessee, who has not opted for taken for business purpose from his
presumptive taxation and his turnover friend Kunal. The repayment also
during the P.Y. is ₹ 2 crore. includes interest of ₹5,000.
(ii) A professional whose gross receipts - 19.08.2022: Portable dye machinery
during the previous year amounts to ₹50 purchased for ₹ 15,000. The payment was
lakh, who declares his profits and gains made in cash in three weekly instalments.
from profession u/s 44ADA. - 26.01.2023: Payment of ₹ 10,000 made
(iii) An assessee having turnover of ₹1.5 to electrician due to unforeseen electric
crore, who declares his profits and gains circuit at shop
from business u/s 44AD. - 28.02.2023: Purchases made from
(iv) A lawyer having gross receipts of ₹40 unregistered dealer for ₹ 13,500
lakhs during the P.Y. who claims his What will be disallowance under 40A(3),
profits and gains from the legal if any, if Mr. Shahid opts to declare his
profession to be 40% of the gross income as per the provisions of section
receipts. 44AD?
(v) An individual who opts out of the (a) ₹ 18,500
presumptive taxation scheme u/s 44AD (b) ₹28,500
during the P.Y., however, his total income (c) ₹13,500
for the said year is ₹2,00,000. (d) NIL (MTP Oct 2019)
(a) (i), (iv) Q23) Mr. X, a retailer, acquired furniture on 10th May
(b) (i), (iv), (v) 2022 for ₹ 10,000 in cash and on 15th May
(c) (i), (ii), (iv) 2022, for ₹ 15,000 and ₹ 20,000 by a bearer
(d) (iv), (v) (MTP March 2019) cheque and account payee cheque, respectively.
Q21) K is a working partner in a firm on behalf of his Depreciation allowable for A.Y. 2023-24 would
HUF and the HUF has contributed ₹ 3,00,000 as be –
its capital contribution. Apart from this, K has (a) ₹ 2,000
given a loan of ₹ 50,000 to the firm in his (b) ₹ 3,000
individual capacity. The firm pays interest as per (c) ₹ 3,500
market rate of 15% per annum on capital as (d) ₹ 4,500
well as loan. Compute the amount of interest
 
36 Prepared by : CA Sanchit Grover
  Multiple Choice Questions
  for CA Inter Taxation For May 2023 Exams

Q24) Ms. Rimjhim (aged 32 years), an interior received dividend. What would be her total
decorator, has professional receipts of ₹ income for the A.Y. 2023-24, assuming that she
25,60,000 for the previous year 2022-23. She wishes to make maximum tax savings without
also earned ₹ 1,25,000 as dividend and ₹ getting her books of account audited?
4,65,000 as interest income on fixed deposits. (a) ₹ 18,45,000
She incurred expenses of ₹ 13,00,000 for her (b) ₹ 18,70,000
profession and ₹ 30,000 as interest on loan for (c) ₹ 18,40,000
making investment in shares on which she (d) ₹ 18,25,000 (RTP May 2021)

Answers to Chapter 9

Ans. 1) Option (a) is correct Ans. 5) Option (a) is correct


In case of Power units following SLM method for charging Opening WDV of Block 3,20,000
depreciation, when the asset is sold, if the Sale Price is less Add:- Actual cost of 2 nd
Hand machinery 50,000
than WDV of the asset as on 1st Day of PY, then the st
put to use on 1 Nov 22
difference (₹ 15,000 in this case) shall be allowed as Less:- Sale Proceeds (2,00,000)
Terminal Depreciation u/s 32 Closing WDV 1,70,000
Normal Depreciation 21,750
Ans. 2) Option (a) is correct - 50,000 x 7.5%
Oil wells are covered in definition of Plant & Machinery and - 1,20,000 x 15%
since there is no specific rate of depreciation notified for it, Additional Depreciation (not allowed on Nil
hence 15% rate shall be applicable in this case. nd
2 hand machinery)

Ans. 3) Option (a) is correct Ans. 6) Option (d) is correct


In respect of any capital asset used for scientific purpose, Calculation of Presumptive income u/s 44AD
for which 100% deduction has already been allowed u/s 35,
a) ₹ 25L x 6% 1,50,000
when it is used for any other business purpose, Actual Cost
b) ₹ 50L x 8% 4,00,000
shall be considered as Nil
c) ₹ 25L x 6% 1,50,000
d) ₹ 30L x 8% 2,40,000
Ans. 4) Option (c) is correct
Total Presumptive Income 9,40,000
Book Profits of firm = (112,000) + 6,00,000 = ₹ 4,88,000
Maximum salary allowed to partners u/s 40(b) shall be
Ans. 7) Option (d) is correct
computed as under:-
Loss from Non-speculative business can be set off against
For first ₹ 3L Book Profits 2,70,000
profits of any business, whether speculative or non-
For remaining ₹ 1,88,000 Book 1,12,800
speculative. Further, it can be set off against any head of
Profits @60%
income other than Salary.
Maximum Salary 3,82,800
Hence, Salary taxable in hands of Mr. C (being calculated
Ans. 8) Option (b) is correct
in proportion to Salary allowable to firm) = ₹ 3,28,000/3 = ₹
In the given case, ₹ 28,000 of Bad debts recovered will be
1,27,600
taxable in hands of Mr. Kumar. Loss of ₹ 8,000 related to
Cloth business (non-speculative) can be set off against this
₹ 28,000 income but loss of ₹ 40,000 from speculative
business cannot be set off against income of non-
speculative business
  
Prepared by : CA Sanchit Grover 37
   CHAPTER 9 : INCOME UNDER THE HEAD PGBP
Ans. 9) Option (c) is correct since payment > 10,000 has been made in a single day
Deduction u/s 35AD is allowed in respect of any capital other than through 4 prescribed mediums
expenditure incurred for specified business excluding
expenditure on land, goodwill & financial instruments. Ans. 14) Option (D) is correct
Hence, Mr. Raja can claim deduction of ₹ 6 Crore u/s 35AD Since the amt of commission is more than threshold of ₹
and hence loss from specified business comes out to be ₹ 15,000, hence TDS u/s 194H shall be applicable here.
5.5Crore. However, this loss cannot be carried forward if the Further, since X Ltd. has failed to deduct TDS but Y Ltd has
return is filed after due date of filing return of income filed his ROI on 15-07-2023 (and also paid tax on this
income), X Ltd. shall be deemed to have deducted and
Ans. 10) Option (b) is correct deposited TDS on 15-07-2023. Hence, even in this case,
Benefit of paying advance tax in one installment instead of disallowance being 30% of 45,000 u/s 40(a)(ia) shall be
4 installments is available only in case of Sec 44AD and attracted in this case.
44ADA but not Sec 44AE.
Ans. 15) Option (B) is correct
Ans. 11) Option (b) is correct Since the payment has been made to non-resident without
Book Profits before interest & 37,00,000 deducting TDS, disallowance u/s 40a(i) shall be applicable
remuneration and full 100% of the expenditure shall be disallowed.
Less:- Interest on capital (within the limit (1,30,000)
prescribed u/s 40(b)) Ans. 16) Option (D) is correct
Book Profits for purpose of calculating 35,70,000 Purchase price of Building 15,00,000
maximum salary Less:- Notional depreciation for PY 20- (1,50,000)
Maximum Salary as per 40(b) 21 @10%
a) For first ₹ 3,00,000 of Book Profits 22,32,000 WDV as on 01-04-2021 13,50,000
b) For remaining ₹ 32,70,000 Book Profits Less:- Notional Depreciation for PY 20- (1,35,000)
@ 60% 21 @5%
Salary allowable (Since actual salary paid 11,00,000 WDV as on 01-04-2022 (It will be 12,15,000
is less than maximum permissible limit) considered as Actual Cost, when
introduced in business)
Ans. 12) Option (C) is correct Depreciation for PY 22-23 calculated 1,21,500
Book Profits before interest & 2,70,000 @10%
remuneration
Less:- B/f Unabsorbed Depreciation (1,20,000) Ans. 17) Option (C) is correct
deductible u/s 32 Income tax paid is disallowed u/s 40(a). Further, customs
Less:- B/f Business Loss (not deductible - duty pertaining to FY 21-22 would have been allowed in PY
since it is not covered u/s 30 to 37) 21-22 itself since it has been paid before due date of filing
Book Profits for purpose of calculating 1,50,000 ROI of PY 21-22. Accordingly, no deduction shall be allowed
maximum salary in PY 22-23
Maximum Salary as per 40(b) – higher 1,50,000
of 2 Ans. 18) Option (B) is correct
a) 1,50,000 x 90% In the given case, since the payment has been made to non-
b) ₹ 1,50,000 resident, hence Sec 40a(i) shall be applicable and not
40a(ia). Accordingly, 100% of the expense shall be
Ans. 13) Option (A) is correct disallowed.
As per Sec 43, ₹ 20,000 paid in cash and ₹ 15,000 paid
through bearer cheque shall not form part of Actual Cost

 
38 Prepared by : CA Sanchit Grover
  Multiple Choice Questions
  for CA Inter Taxation For May 2023 Exams
Ans. 19) Option (c) is correct Ans. 22) Option (d ) is correct
Loss after debiting remuneration -1,12,000 In case presumptive income u/s 44AD is calculated, all
Add:- Remuneration paid to partners (₹ 6,00,000 disallowances shall be deemed to have been given effect
2L x 3m) to.
Book Profits 4,88,000
Maximum remuneration allowable to Ans. 23) Option (b) is correct
firm u/s 40(b) 2,70,000 As per Sec 43, ₹ 15,000 shall not form part of Actual cost
a) First ₹ 3,00,000 1,12,800 since payment > 10,000 in a single day has been made
b) Remaining ₹ 1,88,000 @60% other than through 4 prescribed mediums. Hence Actual
3,82,800 Cost = 30,000 and rate of depreciation applicable shall be
Remuneration taxable in hands of each 1,27,600 10% (since asset has been put to use for > 180 days)
partner (3,82,800/3)
Ans. 24) Option (a) is correct
Ans. 20) Option (A) is correct Income from profession computed as per 12,80,000
In case assessee doesn’t opt for presumptive taxation u/s Sec 44ADA (₹ 25,60,000 x 50%). If actual
44AD or 44ADA and his Total income exceeds basic expenditure is deducted, PGBP income
exemption limit, he is required to get books of accounts would be ₹ 12,60,000 but in such case
audited as per Sec 44AB Audit u/s 44AB shall become mandatory
Interest from FD 4,65,000
Ans. 21) Option (d) is correct Dividend Income 1,25,000 1,00,000
Max Interest allowed on capital of Partner as per Sec 40(b) Less:- Interest expense (25,000)
= ₹ 3,00,000 x 12% = ₹, 36,000. (proviso to Sec 57)
However, limits of Sec 40(b) do not apply on Interest on loan Gross Total Income 18,45,000
given by partner. Hence Full 15% on ₹ 50,000 = ₹ 7,500 Less :- Deduction u/s 80TTB (not available Nil
shall be allowed as deduction to firm since Rimjhim is not a senior citizen)
Total Income 18,45,000

  
Prepared by : CA Sanchit Grover 39
   CHAPTER 10 : INCOME UNDER THE HEAD CAPITAL GAINS

CHAPTER 10 INCOME UNDER THE HEAD CAPITAL GAINS


Q1) For an assessee, who is a salaried employee would be the amount of capital gain chargeable
who invests in equity shares, what is the benefit to tax in the hands of Mr. A for the assessment
available in respect of securities transaction tax year 2023-24? Cost inflation index for FY 2021-
paid by him on sale and acquisition of 100 22: 317; 2022-23: 331.
listed shares of X Ltd. which has been held by (a) Short-term capital gain of ₹ 9.75 lakhs
him for 14 months before sale? (b) Short-term capital gain of ₹ 7 lakhs
(a) Rebate under section 88E is allowable in (c) Long-term capital gain of ₹ 2,54,325
respect of securities transaction tax paid (d) Long-term capital gain of ₹ 2,67,531
(b) Securities transaction tax paid is treated Q4) Mr. B acquires 1000 equity shares on
as expenses of transfer and deducted 01.01.2017 at ₹200. The Fair Market Value of
from sale consideration. the said shares on 31.01.2018 is ₹500. Mr. B
(c) Capital gains without deducting STT paid sells the said shares on 30.04.2022 at ₹400.
is taxable at a concessional rate of 10% Calculate the amount of capital gain in the
on such capital gains exceeding ₹ 1 lakh hands of Mr. B, assuming that securities
(d) Capital gains without deducting STT paid transaction tax has been paid by Mr. B on
is taxable at concessional rate of 15% acquisition and transfer of the said equity
Q2) Under section 54EC, capital gains on transfer of shares.
land or building or both are exempted if (a) Nil
invested in the bonds issued by NHAI & RECL or (b) (-) ₹ 1,00,000
other notified bond – (c) ₹ 2,00,000
(a) within a period of 6 months after the date (d) ₹ 3,00,000
of such transfer Q5) Maya had bought 3,000 STT paid listed shares
(b) within a period of 6 months from the end of a company on 25.02.2022 at ₹ 300 per share.
of the relevant previous year The company announces and allots bonus
(c) within a period of 6 months from the end shares in the ratio of 1 share for every 3 held,
of the previous year or the due date for on 01.07.2022. After the allotment of bonus
filing the return of income under section shares, the shares were sold by Maya on
139(1), whichever is earlier 10.03.2023 at ₹400 per share and STT was paid
(d) At any time before the end of the relevant on it. Compute the amount of capital gain/loss
previous year. in her hands for A.Y. 2023-24. CII – F.Y. 2021-
Q3) Mr. A (aged 45 years) sold an agricultural land 22: 317; F.Y. 2022-23: 331
for ₹ 52 lakhs on 04.10.2022 acquired at a cost (a) Long term capital gain of ₹7,00,000.
of ₹ 49.25 lakhs on 13.09.2021 situated at 7 (b) Long term capital gain on sale of original
kms from the jurisdiction of municipality having shares of ₹3,00,000. Short term capital
population of 4,00,000 and also sold another gain on sale of bonus shares of
agricultural land for ₹ 53 lakhs on 12.12.2022 ₹4,00,000.
acquired at a cost of ₹ 46 lakhs on 15.02.2021 (c) Long term capital of ₹6,52,160.
situated at 1.5 kms from the jurisdiction of (d) Long term capital gain on sale of original
municipality having population of 12,000. What shares of ₹2,52,160. Short term capital

 
40 Prepared by : CA Sanchit Grover
  Multiple Choice Questions
  for CA Inter Taxation For May 2023 Exams

gain on sale of bonus shares of company is an Indian company. Hence,


₹4,00,000. no capital gains tax liability shall arise.
Q6) XYZ & Co., a partnership firm, owns a house (c) A transfer between a holding company
property which is utilized by the partners for and 100% subsidiary company is treated
their residence. On 31.10.2019, the firm sells the as a “transfer” as there is no specific
property at a long-term capital gain of exclusion in this regard. Hence, capital
₹3,50,000. Can the firm or partners claim gains tax liability shall arise.
exemption under section 54? (d) Any transfer between a holding company
(a) Yes, the firm can claim exemption u/s 54 and 100% subsidiary company is not
as the firm has earned long term capital treated as a transfer if the transferee
gains from the transfer of a residential company is an Indian company. However,
house. this rule is not applicable if the capital
(b) Yes, the partners can claim exemption u/s asset is transferred as stock-in-trade.
54 as the property was used by them for Hence, capital gains tax liability shall
residential purpose and the said property arise.
has been indirectly transferred by the Q8) Mr. Vaibhav sold his old residential house in
partners only in the capacity of a firm. April, 2021 for ₹28,00,000. Long- term capital
(c) Neither the firm nor the partners can gain arising on transfer of old house amounted
claim deduction u/s 54 as the said to ₹8,40,000. In December, 2021 he purchased
deduction is allowed only in case of a another residential house worth ₹ 5,00,000. The
commercial property. new house was however, sold in April, 2022 for
(d) The firm cannot claim deduction u/s 54 ₹14,00,000 (stamp duty value of the new house
as deduction under the said section is was ₹12,00,000). What will be amount of
allowed only to an individual or HUF. taxable capital gains in the hands of Mr.
Further, the partners cannot claim Vaibhav for the A.Y. 2022-23 and AY 2023-24?
deduction u/s 54 as the transferor in the (a) Long term capital gain of ₹ 3,40,000 in
instant case is the firm. A.Y. 2022-23 and short-term capital gain
Q7) A Ltd. is 100% holding company of B Ltd. A Ltd. of ₹ 14,00,000 in A.Y. 2023-24
transfers a capital asset (acquired in 2002 for ₹ (b) Long term capital gain of ₹ 3,40,000 in
50,000) on 16.06.2022 for ₹3,70,000 to B Ltd. B A.Y. 2022-23 and long term capital gain
Ltd. is an Indian company, while A Ltd. is a of ₹ 5,00,000 and short-term capital gain
foreign company. The capital asset is transferred of ₹ 14,00,000 in A.Y. 2023-24
as stock-in-trade to B Ltd. Determine whether (c) Long term capital gain of ₹ 3,40,000 in
any capital gains shall be chargeable to tax in A.Y. 2022-23 and long term capital gain
the instant case? of ₹ 5,00,000 and short-term capital gain
(a) Any transfer between a holding company of ₹ 9,00,000 in A.Y. 2023-24
and 100% subsidiary company is not (d) Long term capital gain of ₹3,40,000 in
treated as a transfer at all. Hence, no A.Y. 2022-23 and long term capital gain
capital gains tax liability shall arise. of ₹ 5,00,000 and short-term capital gain
(b) Any transfer between a holding company of ₹7,00,000 in A.Y. 2023-24
and 100% subsidiary company is not Q9) Mr. Rana is a resident of India residing in
treated as a transfer if the transferee Meerut. During F.Y. 2010-11 he purchased an
  
Prepared by : CA Sanchit Grover 41
   CHAPTER 10 : INCOME UNDER THE HEAD CAPITAL GAINS
agricultural land situated in Bahadurpur for ₹ date, the fair market value of the said
10 lacs. This land is situated in an area which asset was ₹6,00,000.
has aerial distance of 3 km from the local limits  The stock-in-trade so converted was sold
of Municipality of Bahadurpur. Total population on 15.07.2022 for ₹ 8,50,000.
of this area is 80,000 as per the last preceding Determine the tax implications in the hands of
census. During F.Y. 2022-23, Mr. Rana sold this Tina for A.Y. 2023-24.
land to Mr. Jeet for ₹ 25 lacs on 29.1.2023. Mr. Cost Inflation Index Financial year 2001-02:
Rana invested ₹ 5 lakhs in bonds of NHAI on 100, 2020-21: 301, 2021-22: 317, 2022-23: 331]
31.7.2023. Cost inflation index for F.Y. 2010-11 a) Only business profits of ₹ 2,50,000 shall
and F.Y. 2022-23 is 167 and 331 respectively. be chargeable to tax in the hands of Tina
Compute the amount of capital gain taxable in in A.Y. 2023-24.
the hands of Mr. Rana for A.Y. 2023-24: b) Only long term capital gain of ₹ 5,79,916
(a) ₹ 6,97,605 shall be chargeable to tax in the hands of
(b) ₹ 1,97,695 Tina in A.Y. 2023-24.
(c) ₹ 10,00,000 c) Business profits of ₹ 2,50,000 and long
(d) None of the above term capital gain of ₹ 3,29,916 shall be
Q10) Ms. Jaya acquires 5,000 equity shares on chargeable to tax in the hands of Tina in
01.01.2016 at ₹ 500. The Fair Market Value of A.Y. 2023-24.
the said share on 31.01.2018 is ₹ 250 and on d) Business profits of ₹ 2,50,000 and long
31.03.2018 is ₹ 600. She sells the said shares term capital gain of ₹ 3,43,548 shall be
on 30.04.2022 at ₹ 700. Calculate the amount chargeable to tax in the hands of Tina in
of long term capital gain in the hands of Ms. A.Y. 2023-24.
Jaya assuming that Securities Transaction Tax Q12) Neha sold her residential house for ₹ 85 lakh
has been paid by her on acquisition and on 11.08.2022. Value adopted by the Stamp
transfer of the said equity share. Valuation Authority on the date of registration
CII – F.Y. 2015-16: 254; F.Y. 2018-19: 280, F.Y. of the Conveyance Deed i.e., 17.08.2022 was ₹
2022-23: 331 150 lakh. Neha disputed the valuation done by
(a) ₹10 lakh, out of which ₹9 lakh is the said authority before the Assessing Officer
taxable@10% and filed an application before him to refer her
(b) ₹22.50 lakh, out of which ₹21.5 lakh is case to the Valuation Officer. The Valuation
taxable@10% Officer determined the value of the house on
(c) ₹7.45 lakh, out of which ₹6.45 lakh is date of registration of Conveyance Deed at ₹
taxable @10% 160 lakh. In light of these facts, compute the full
(d) ₹5 lakh, out of which ₹4 lakh is value of consideration to be taken in case of
taxable@10% Neha for the purpose of calculation of capital
Q11) The following information is available with gains in her hands.
respect to Tina: (a) ₹ 85 lakh
 Capital Asset acquired on 01.04.2001 for (b) ₹ 150 lakh
₹85,200 (c) ₹ 160 lakh
 The capital asset was converted into (d) ₹ 89.25 lakh
stock-in-trade on 30.09.2020. On the said Q13) Suman is a Chartered Accountant practicing in
Mumbai since September, 1994. She transfers
 
42 Prepared by : CA Sanchit Grover
  Multiple Choice Questions
  for CA Inter Taxation For May 2023 Exams

her practice to another Chartered Accountant property was purchased by them 25 months
Smita on 19.06.2022 and charges ₹ 14,50,000 back. The indexed cost of acquisition is ₹ 1 crore
towards goodwill. Determine the tax and ₹ 1.75 crore, respectively. Mr. Vishal
implications that may arise in the hands of purchased two residential flats, one in Delhi and
Neha on account of transfer of her practice to one in Agra for ₹ 70 lakhs and ₹ 80 lakhs,
Smita. CII for FY 22-23 is 331:- respectively, in April, 2023. On the same date,
(a) ₹14,50,000 shall be charged to tax as Mr. Guha also purchased two residential flats,
capital gains one in Mumbai and the other in Pune, for ₹ 80
(b) ₹14,50,000 shall be charged to tax as lakhs and ₹ 75 lakhs, respectively. Both of them
income from other sources invested ₹ 30 lakhs in bonds of NHAI in March,
(c) ₹14,50,000 shall be charged to tax as 2023 and ₹ 30 lakhs in bonds of RECL in April,
income from profession 2023.
(d) No tax implications shall arise What is the income taxable under the head
Q14) In P.Y. 2022-2023, Mr. A has transferred the “Capital Gains” for A.Y. 2023-24 in the hands of
following assets: Mr. Vishal and Mr. Guha?
Asset Full Value of Indexe Transfer (a) ₹ 70 lakhs and ₹ 95 lakhs, respectively
Trasfer- Consideratio d Cost Date (b) ₹ 60 lakhs and ₹ 85 lakhs, respectively
red n of (c) Nil and ₹ 95 lakhs, respectively
acquisi (d) Nil and ₹ 20 lakhs, respectively
-tion Q16) Mr. Ram, an Indian resident, purchased a
Residentia 8 crores 6 crores 25.11.202 residential house property at Gwalior on
l House 2 28.05.1999 for ₹ 28.5 lakhs. The fair market
Property value and the stamp duty value of such house
Jewellery 3 Crore 2 crore 05-01-23 property as on 1.4.2001 was ₹ 33.5 lakhs and ₹
Mr. A bought a new residential house property 32.4 lakhs, respectively. On 05.02.2012, Mr. Ram
on 01.04.2021 for ₹ 1 crore and on 28.02.2023 entered into an agreement with Mr. Byomkesh
deposited ₹ 3 crores in a capital gains deposit for sale of such property for ₹ 74 lakhs and
account scheme. On 30.07.2023, Mr. A has received an amount of ₹ 3.9 lakhs as advance.
withdrawn ₹ 3 crores from capital gains deposit However, as Mr. Byomkesh did not pay the
account and acquired a residential house balance amount, Mr. Ram forfeited the advance.
property worth ₹ 2.5 crore. What would be the What would be the indexed cost of acquisition
capital gains in the hands of Mr. A for A.Y. 2023- of Mr. Ram if he sells the property in F.Y. 2022-
24, if the expenses in connection with transfer 23?
of jewellery were ₹ 2,00,000? Cost Inflation Index for F.Y. 2001-02: 100; F.Y.
(a) ₹ 80,50,000 2022-23: 331
(b) ₹ 81,55,705 (a) ₹ 1,10,88,500

(c) ₹ 98,00,000 (b) ₹ 1,07,24,400

(d) ₹ 48,00,000 (c) ₹ 97,97,600


Q15) Mr. Vishal and Mr. Guha sold their residential (d) ₹ 94,33,500
house property in Pune for ₹ 3 crore and ₹ 4 Q17) A building was acquired on 1.4.1995 for ₹
crore, respectively, in January, 2023. The house 20,00,000 and sold for ₹ 80,00,000 on
01.06.2022. The fair market value of the
  
Prepared by : CA Sanchit Grover 43
   CHAPTER 10 : INCOME UNDER THE HEAD CAPITAL GAINS
building on 1.4.2001 was ₹ 25,00,000. Its stamp (a) ₹ 7,18,000
duty value on the same date was ₹ 22,00,000. (b) ₹ 13,80,000
Determine the capital gains on sale of such (c) ₹ 60,00,000
building for the A.Y. 2023-24? CII for F.Y. 2001- (d) (₹ 2,75,000)
02: 100; F.Y. 2022-23: 331

Answers to Chapter 10

Ans. 1) Option (c) is correct 3000 Original 1000 Bonus


In case STT is paid both at the time of acquisition as well as shares Shares
transfer of listed shares and POH > 12m, in such case LTCG POH 25-02-2022 to 01-07-2022 to
u/s 112A shall arise. Further, no deduction is allowed in 10-03-2023 10-03-2023
respect of STT while computing capital gains Type of asset Long term Short term
FVC 12,00,000 4,00,000
Ans. 2) Option (a) is correct COA (without 9,00,000 Nil
6 months period has to be calculated from the date of indexation)
transfer of land & building LTCG u/s 112A 3,00,000 -
STCG u/s 111A - 4,00,000
Ans. 3) Option (B) is correct
First agricultural land is situated beyond 6 km of Ans. 6) Option (d) is correct
municipality (having population of >1L but < 10L) and Sec 54 is allowed only to Individual & HUF assessee in
hence it is rural agricultural land and accordingly not a respect of residential house property transferred by
capital asset. 2nd Agricultural land is situated within 2Km assessee himself.
from municipality having population of 12000 and hence
urban agricultural land. Further, POH is less than 24M and Ans. 7) Option (d) is correct
hence STCG of 7L (53L – 46L) shall arise. Sec 47 excludes the above transactions between holding
and subsidiary only if the following conditions are satisfied:-
Ans. 4) Option (a) is correct a) Transferee company is Indian company
In the given case, Sec 112A is applicable and since the b) Subsidiary is 100% subsidiary company
shares have been purchased before 31-10-18, COA shall be c) The transferee company holds such asset otherwise
determined as per spcl method u/s 55(2)(ac) as under as stock in trade
FMV as on 31-01-2018 (A) 5,00,000
Full Value of Consideration (B) 4,00,000 Ans. 8) Option (a) is correct
Lower of (A) & (B) 4,00,000 LTCG in AY 22-23 before exemption 8,40,000
COA as per normal provisions (C) 2,00,000 Less:- Exemption u/s 54 (5,00,000)
COA as per Sec 55(2)(ac) being higher 4,00,000 Taxable LTCG (after exemption) for AY 3,40,000
of above 2 amounts 2022-23
Full Value of Consideration for new HP 14,00,000
Ans. 5) Option (b) is correct sold in April 2022 (As per Sec 50C, SDV
In the given case, Sec 112A shall be applicable on transfer shall be deemed as FVC)
of 3000 original shares & Sec 111A on transfer of 1000 Less:- Cost of acquisition Nil
bonus shares (although no STT on acquisition of bonus (₹ 5,00,000 – ₹ 5,00,000)
shares but it is CBDT notified transaction). Hence capital Since the new HP has been transferred
gain shall be computed as under:- within 3 years from date of its purchase
STCG 14,00,000

 
44 Prepared by : CA Sanchit Grover
  Multiple Choice Questions
  for CA Inter Taxation For May 2023 Exams
Ans. 9) Option (d) is correct Indexed COA 6 Crore 2 crore
Since the agricultural land is situation beyond 2 km from LTCG before 2 crore 98L
the limits of municipality (having population of > 10,000 but exemption
< 1L), it is rural agricultural land and hence not a capital Less:- Exemption 2 crore 16,44,295
asset u/s 54
LTCG after Nil 81,55,705
Ans. 10) Option (a) is correct exemption
In the given case, Sec 112A is applicable and since the Notes:-
shares have been purchased before 31-10-18, COA shall be 1) HP worth ₹ 2.5Cr. has been purchased before due
determined as per spcl method u/s 55(2)(ac) as under date of return of income for PY 22-23 and hence it
FMV as on 31-01-2018 (A) 12,50,000 will be considered for computing exemption u/s 54
Full Value of Consideration (B) 35,00,000 in respect of Residential HP. HP purchased on 01-
Lower of (A) & (B) 12,50,000 04-2021 shall not be considered since it is not
COA as per normal provisions (C) 25,00,000 purchased within 1 year prior to the date of
COA as per Sec 55(2)(ac) being higher 25,00,000 transfer.
of above 2 amounts 2) Remaining 50L in CG Saving account shall be
considered for calculating exemption u/s 54F in
Ans. 11) Option (d) is correct respect of Jewellery.
In case of conversion of capital asset into stock in trade, Exemption = 98L x 50L/198L = ₹ 16,44,295
transfer takes place on date of conversion but capital gains
are taxable in year when stock is ultimately sold Ans. 15) Option (c) is correct
Full Value of Consideration (FMV as on 6,00,000 Particulars Vishal Guha
30-09-20) Full Value of 3 Crore 4 crore
Less:- Indexed COA (85,200 x 301/100) 2,56,452 Consideration
LTCG taxable in AY 23-24 3,43,548 Less:- Indexed COA 1 Crore 1.75 Crore
PGBP income in AY 23-24 2,50,000 LTCG before 2 Crore 2.25 Crore
(₹ 8,50,000 – ₹ 6,00,000) exemption
Exemption u/s 54 150L 80L
Ans. 12) Option (b) is correct Exemption u/s 50L 50L
Where value determined by Valuation Officer is more than 54EC
SDV, then SDV shall be considered for applying Sec 50C. LTCG after Nil 95L
Further in the given case, SDV (₹ 150L) > 110% of Sale exemption
Consideration (₹ 85L) and hence as per Sec 50C, Full value Notes:-
of consideration shall be ₹ 150L. 1) Since LTCG in case of Mr. Vishal is not more than
2 crore, hence he can consider both new HP
Ans. 13) Option (a) is correct purchased for exemption u/s 54. Accordingly,
In case of Self-generated Goodwill of Profession, COA shall exemption shall be ₹ 70L + 80L = ₹ 150L. Further,
be Nil and entire Sale consideration would become STCG or since total of ₹ 60L has been invested in bonds of
LTCG (as the case may be) NHAI & REC within 6 months, maximum
exemption u/s 54EC shall be ₹ 50L.
Ans.14) Option (b) is correct 2) Since LTCG in case of Mr. Guha is more than ₹ 2
Particulars Residential HP Jewellery Cr., he can consider only 1 HP for exemption u/s
Full Value of 8 Crore 3 crore 54. Since Purchase price of Mumbai HP is more,
Consideration hence exemption u/s 54 shall be ₹ 80L. Further,
Exp on transfer Nil 2L since total of ₹ 60L has been invested in bonds of

  
Prepared by : CA Sanchit Grover 45
   CHAPTER 10 : INCOME UNDER THE HEAD CAPITAL GAINS
NHAI & REC within 6 months, maximum Indexed COA 94,33,500
exemption u/s 54EC shall be ₹ 50L (28,50,000 x 331/100)

Ans. 16) Option (d) is correct


Ans. 17) Option (a) is correct
Actual Purchase Price 28,50,000
Full Value of consideration 80,00,000
FMV as on 1st April 2001 32,40,000
Less:- Indexed Cost of acquisition (22L x 72,82,000
(can’t be more than SDV as on the st
331/100). FMV as on 1 April 2001
same date)
cannot be more than SDV on such date)
Less:- Advance money forfeited on 05- (3,90,000)
LTCG 7,18,000
05-2012, to be deducted from COA

 
46 Prepared by : CA Sanchit Grover
  Multiple Choice Questions
  for CA Inter Taxation For May 2023 Exams

CHAPTER 11 TAXATION OF GIFTS


Q1) Mr. Kashyap has acquired a building from his (d) Mr. Kishore will be liable for tax on cash
friend on 10.10.2022 for ₹ 15,00,000. The stamp gifts only.
duty value of the building on the date of Q4) Ashok took possession of property on 31st
purchase is ₹ 16,20,000. Income chargeable to August 2022 booked by him three years back at
tax in the hands of Mr. Kashyap is:- ₹ 25 lakhs, The Stamp Duty Value (SDV) of the
(a) ₹ 70,000 property as on 31st August 2022 was ₹31 lakh
(b) ₹ 50,000 and on date of booking it was ₹ 29 lakh. He had
(c) Nil paid ₹ 2 lakh by A/c payee cheque as down
(d) ₹ 1, 20,000 payment on date of booking. Which of the
Q2) Mr. Mayank has received a sum of ₹ 75,000 on following will be considered as income, if any,
24.10.2022 from his friend on the occasion of and in which previous year
his marriage anniversary. What would be the (a) ₹4 lakhs in P.Y. 2022-23
taxability of the said sum in the hands of Mr. (b) ₹ 4 lakhs in P.Y. 2019-20
Mayank? (c) ₹ 6 lakhs in P.Y. 2022-23
(a) Entire ₹ 75,000 is chargeable to tax (d) No income shall be taxable, since down
(b) Entire ₹ 75,000 is exempt from tax payment was paid by A/c cheque while
(c) Only ₹ 25,000 is chargeable to tax booking the property
(d) Only 50% i.e., ₹ 37,500 is chargeable to Q5) Mr. X receives the following gifts during the
tax previous year 2022-23:
Q3) Mr. Kishore celebrated his 50th marriage  On 20.09.2022, he gets a gift of ₹7,00,000
anniversary. On this occasion, his wife received from his grandmother.
a diamond necklace worth ₹ 5,00,000 from  On 30.12.2022, he gets by way of gift a
Kishore's brother. Kishore's son gifted him a commercial flat from the elder brother of
luxurious car worth ₹ 15,00,000, His his father-in-law (stamp duty value is ₹
grandchildren gifted them a new furniture set 25,00,000).
worth ₹3,00,000. Also, he received cash gifts  On 20.01.2023, he gets a wrist watch by
from his friends amounting collectively to gift from his friend B (Fair market value:
₹80,000. Which of them the following ₹1,00,000).
statements stand true on taxability.  On 10.02.2023, he gets by way of gift a
(a) Neither Mr. Kishore nor Mrs. Kishore will plot of land in Pune from a partnership
be liable for tax for any gifts since they firm. The partnership firm has only two
have been received on occasion of partners- father of Mr. X and Mrs. X. The
marriage anniversary stamp duty value of the plot of land is
(b) Mr. Kishore & Mrs. Kishore will jointly ₹19,00,000.
share the tax liability on all the gifts Compute the amount chargeable to tax in the
(c) Mrs. Kishore will be liable to pay tax on hands of X under the head “Income from other
diamond set and Mr. Kishore will bear tax sources” for the A.Y. 2023-24
for the cash gifts received a) ₹ 25,00,000
b) ₹ 44,00,000

  
Prepared by : CA Sanchit Grover 47
   CHAPTER 11 : TAXATION OF GIFTS
c) ₹ 45,00,000 The indexed cost of acquisition of shares for Mr.
d) ₹ 52,00,000 (MTP May 2019) Vikas was computed at ₹ 4,25,000. Determine
Q6) Mr. Vikas received a gold ring worth ₹ 60,000 the income chargeable to tax in the hands of
on the occasion of his daughter’s wedding from Mr. Vikas and ABC (P) Ltd. in respect of the
his best friend Mr. Vishnu. Mr. Vishnu also gifted above transaction.
a gold chain to Kavya, daughter of Mr. Vikas, (a) ₹ 90,000 chargeable to tax in the hands
worth ₹ 80,000 on the said occasion. Would of Mr. Vikas as long term capital gains
such gifts be taxable in the hands of Mr. Vikas and nothing is taxable in the hands of
and Ms. Kavya? ABC (P) Ltd
(a) Yes, the gift of gold ring and gold chain is (b) ₹ 75,000 chargeable to tax in the hands
taxable in the hands of Mr. Vikas and Ms. of Mr. Vikas as long-term capital gains
Kavya, respectively and nothing is taxable in the hands of
(b) Such gifts are not taxable in the hands of ABC (P) Ltd.
Mr. Vikas nor in the hands of Ms. Kavya (c) ₹ 90,000 chargeable to tax in the hands
(c) Value of gold ring is taxable in the hands of Mr. Vikas as long-term capital gains
of Mr. Vikas but value of gold chain is not and ₹ 1,65,000 is taxable under the head
taxable in the hands of Ms. Kavya “Income from other sources” in the hands
(d) Value of gold chain is taxable in the of ABC (P) Ltd.
hands of Ms. Kavya but value of gold ring (d) ₹ 75,000 chargeable to tax in the hands
is not taxable in the hands of Mr. Vikas of Mr. Vikas as long-term capital gains
(RTP Dec 2021) and ₹ 1,65,000 is taxable under the head
Q7) Mr. Vikas transferred 600 unlisted shares of XYZ “Income from other sources” in the hands
(P) Ltd. to ABC (P) Ltd. on 15.12.2022 for ₹ of ABC (P) Ltd.
3,50,000 when the market price was ₹ 5,15,000.

Answers to Chapter 11

Ans. 1) Option (c) is correct Ans. 4) Option (a) is correct


Here SDV – Purchase Price is more than ₹ 50,000 but this SDV as on date of booking shall be considered since some
difference is not more than 10% of Purchase Price. Hence, portion of consideration has already been received through
nothing is taxable u/s 56(2)(x) in hands of recipient. A/C Payee Cheque. Hence, ₹ 29L – ₹ 25L = ₹ 4L shall be
taxable u/s 56(2)(x) in the year of purchase (not year of
Ans. 2) Option (c) is correct booking) since this difference is more than 50,000 and also
Here SDV – Purchase Price is more than ₹ 50,000 but this more than 10% of purchase price
difference is not more than 10% of Purchase Price. Hence,
nothing is taxable u/s 56(2)(x) in hands of recipient. Ans. 5) Option (c) is correct
Gift from grandmother is not taxable since it is from relative.
Ans. 3) Option (d) is correct Wrist watch is not specified moveable asset. Elder Brother
Diamond necklace is received by Kishore’s wife from relative of Father-in-law in not covered in definition of relative and
and hence not taxable. Car & Furniture are not specified hence flat worth ₹ 25L received from him would be taxable
movable assets and hence not taxable. Only cash amount u/s 56(2)(x). Plot of land worth ₹ 19L received from firm
received from friends is taxable since its aggregate is more would also be taxable (irrespective of who are the partners
than ₹ 50,000 of such firm)
 
48 Prepared by : CA Sanchit Grover
  Multiple Choice Questions
  for CA Inter Taxation For May 2023 Exams
Ans. 6) Option (c) is correct Ans. 7) Option (c) is correct
Any gifts received by a person on occasion of his/her own In hands of Vikas, Sec 50CA shall be applicable on transfer
marriage shall not be taxable u/s 56(2)(x). However, this of unquoted shares and hence ₹ 5,15,000 shall be
exception is not available to gifts received by family considered as Full value of consideration (Not ₹ 3,50,000).
members of bride or groom. Hence, since gold ring is a Accordingly, LTCG = 5,15,000 – 4,25,000 = ₹, 90,000. Further
specified moveable asset and its FMV is > 50,000, its FMV in hands of ABC (P) Ltd., since shares, being specified
shall be taxable u/s 56(2)(x) in hands of Mr. Vikas moveable property, has been purchased for inadequate
consideration and difference between Purchase price (₹
3,50,000) and FMV of shares (₹ 5,15,000) is more than ₹
50,000, hence entire difference of ₹ 1,65,000 would be
taxable u/s 56(2)(x).

   

  
Prepared by : CA Sanchit Grover 49
   CHAPTER 12 : CLUBBING OF INCOME

CHAPTER 12 CLUBBING OF INCOME


Q1) If the converted property is subsequently ₹ 37,500 was declared on these debentures for
partitioned among the members of the family, P.Y. 2022-23. He transfers interest income to his
the income derived from such converted friend Shyam, without transferring the
property as is received by the spouse of the ownership of these debentures. While filing
transferor will be taxable – return of income for A.Y. 2023-24, Shyam
(a) as the income of the karta of the HUF showed ₹ 37,500 as his income from
(b) as the income of the spouse of the debentures. As tax advisor of Shyam, do you
transferor agree with the tax treatment done by Shyam in
(c) as the income of the HUF. his return of income?
(d) as the income of the transferor-member (a) Yes, since interest income was transferred
Q2) Mr. Vishal started a proprietary business on to Shyam therefore, after transfer it
01.04.2021 with a capital of ₹ 5,00,000. He becomes his income.
incurred a loss of ₹ 1,00,000 during the year (b) No, since Ram has not transferred
2021-22. To overcome the financial position, his debentures to Shyam, interest income on
wife Mrs. Kamini, a Chartered Accountant, gave the debentures is not taxable income of
a gift of ₹ 4,00,000 on 01.04.2022, which was Shyam.
immediately invested in the business by Mr. (c) Yes, if debentures are not transferred,
Vishal. He earned a profit of ₹ 2,00,000 during interest income on debentures can be
the year 2022-23. What is the amount to be declared by anyone, Ram or Shyam, as
clubbed in the hands of Mrs. Kamini for the taxable income depending upon their
Assessment Year 2023-24? discretion.
(a) ₹ 88,888 (d) No, since Shyam should have shown the
(b) ₹ 1,00,000 income as interest income received from
(c) ₹ 2,00,000 Mr. Ram and not as interest income
(d) Nil (MTP Nov 2021) earned on debentures.
Q3) Mr. Aarav gifted a house property valued at ₹ Q5) Mrs. Shivani, wife of Mr. Anurag, is a partner in
50 lakhs to his wife, Geetha, who in turn has a firm. Her capital contribution is ₹ 5 lakhs to
gifted the same to her daughter-in-law Deepa. the firm as on 1.4.2022 which includes ₹ 3.5
The house was let out at ₹ 25,000 per month lakhs contributed out of gift received from
throughout the P.Y. 2022-23. Compute income Anurag. The firm paid interest on capital of ₹
from house property for A.Y. 2023-24. In whose 50,000 and share of profit of ₹ 60,000 during
hands is the income from house property the F.Y.2022-23. The entire interest has been
chargeable to tax? allowed as deduction in the hands of the firm.
(a) ₹ 3,00,000 in the hands of Mr. Aarav Which of the following statements is correct?
(b) ₹ 2,10,000 in the hands of Mr. Aarav (a) Share of profit is exempt but interest on
(c) ₹ 2,10,000 in the hands of Geetha capital is taxable in the hands of Mrs.
(d) ₹ 2,10,000 in the hands of Deepa Shivani.
Q4) Ram owns 500, 15% debentures of Reliance (b) Share of profit is exempt but interest of ₹
Industries Ltd. of ₹ 500 each. Annual interest of 39,286 is includible in the income of Mr.

 
50 Prepared by : CA Sanchit Grover
  Multiple Choice Questions
  for CA Inter Taxation For May 2023 Exams

Anurag and interest of ₹ 10,714 is whose income before this clubbing is


includible in the income of Mrs. Shivani. higher.
(c) Share of profit is exempt but interest of ₹ (d) Income of ₹ 20,13,000 shall be taxable in
35,000 is includible in the income of Mr. the hands of Pihu’s father or mother,
Anurag and interest of ₹ 15,000 is whose income before this clubbing is
includible in the income of Mrs. Shivani. higher.
(d) Share of profit to the extent of ₹ 42,000 Q8) Pankaj gifted an amount of ₹ 3,00,000 to his
and interest on capital to the extent of ₹ wife, Pinky and ₹ 2,00,000 to his daughter, Rinky
35,000 is includible in the hands of Mr. aged 20 years, on 1st April 2019. Both Pinky and
Anurag (MTP May 2020) Rinky invested the amounts on the same date
Q6) Mr. Arvind gifted a house property to his wife, in Government of India 11% Taxable Bonds. The
Ms. Meena and a flat to his daughter-in law, Ms. interest accrues yearly and is reinvested in the
Seetha. Both the properties were let out. Which same bonds. Determine what will be the amount
of the following statements is correct? taxable in hands on Pinky for A.Y. 2023-24?
(a) Income from both properties is to be (a) ₹ 4,473
included in the hands of Mr. Arvind by (b) ₹ 12,132
virtue of section 64. (c) ₹ 33,000
(b) Income from property gifted to wife alone (d) Nil (MTP May 2021)
is to be included in Mr. Arvind’s hands by Q9) Mr. Hari is 65 years old residing in Agra. During
virtue of section 64. F.Y. 2017-18, he purchased a house property in
(c) Mr. Arvind is the deemed owner of house Kamla Nagar for ₹ 25 lacs. This house property
property gifted to Ms. Meena and Ms. was self-occupied by him till F.Y. 2019-20. In F.Y.
Seetha. 2020-21, he shifted to Delhi and the house
(d) Mr. Arvind is the deemed owner of property in Kamla Nagar\ was let out to Mr.
property gifted to Ms. Meena. Income Kishore. His income from house property was ₹
from property gifted to Ms. Seetha would 5 lacs per annum (computed).
be included in his hands by virtue of During F.Y. 2022-23, Mr. Hari earned long-term
section 64 capital gain of ₹ 2.50 lacs, casual income of ₹
Q7) On 20.10.2022, Pihu (minor child) gets a gift of 10 lacs, agricultural income of ₹ 3 lacs and
₹ 20,00,000 from her father’s friend. On the profits from business of ₹ 4 lacs. During the
same day, the amount is deposited as fixed same year, he transferred house property
deposit in Pihu’s bank account. On the said situated in Kamla Nagar to Mrs. Neelam (his
deposit, interest of ₹ 13,000 was earned during son’s wife) without any consideration.
the P.Y. 2022-23. In whose hands the income of Subsequently, income from house property was
Pihu shall be taxable? Also, compute the received by Mrs. Neelam for F.Y. 2022-23.
amount of income that shall be taxable. Compute gross total income of Mr. Hari for A.Y.
(a) Income of ₹ 20,11,500 shall be taxable in 2023-24:
the hands of Pihu’s father. (a) ₹ 16.50 lacs
(b) Income of ₹ 20,13,000 shall be taxable in (b) ₹ 21.50 lacs
the hands of Pihu’s father. (c) ₹ 19.50 lacs
(c) Income of ₹ 20,11,500 shall be taxable in (d) ₹ 24.50 lacs
the hands of Pihu’s father or mother,
  
Prepared by : CA Sanchit Grover 51
   CHAPTER 12 : CLUBBING OF INCOME
Q10) Mr. Raj Makes a gift of ₹ 25,000 to his wife, Mrs. (a) Share of profit is exempt but interest on
Rama, on 27.03.2022. Mrs. Rama, on 2.4.2021, capital is taxable in the hands of Mrs.
invests ₹ 75,000 (25,000 out of gift and 50,000 Bhawna
of her own) in a partnership firm as capital (b) Share of profit is exempt but interest of ₹
which is her total capital contribution in the 80,000 is includible in the income of Mr.
firm. During the year ended 31.03.2023 she Sonu and interest of ₹ 40,000 is
earns an interest of ₹ 12,000 and salary of ₹ includible in the income of Mrs. Bhawna
1,20,000 from the firm. What amount shall form (c) Share of profit is exempt but interest of ₹
part of total income of Mr. Raj for the previous 72,000 is includible in the income of Mr.
year 2021-22? Sonu and interest of ₹ 48,000 is
(a) ₹ 3,000 as interest on capital from firm includible in the income of Mrs. Bhawna
(b) ₹ 3,000 as interest on capital from firm (d) Share of profit to the extent of ₹ 60,000
and ₹ 40,000 as salary from firm and interest on capital to the extent of ₹
(c) ₹ 4,000 as interest on capital from firm 72,000 is includible in the hands of Mr.
and ₹40,000 as salary from firm Sonu. (MTP May 2022)
(d) Nil (MTP May 2022) Q12) Sujata, aged 16 years, received scholarship of
Q11) Mrs. Bhawna, wife of Mr. Sonu, is a partner in a ₹50,000 during the previous year 2022-23.
firm. Her capital contribution of ₹ 10 lakhs to Which of the following statements are true
the firm as on 31.3.2022 included ₹ 6 lakhs regarding taxability of such income:
contributed out of gift received from Sonu. On (a) Such income shall be assessed in hands
1.4.2022, she further invested ₹ 2 lakh out of of Sujata
gift received from Sonu. The firm paid interest (b) Such income to be included with the
on capital of ₹ 1,20,000 and share of profit of ₹ income of parent whose income before
1,00,000 during the F.Y.2022-23. The entire such clubbing is higher
interest has been allowed as deduction in the (c) Such income is completely exempt from
hands of the firm. Which of the following tax
statements is correct? (d) Such income to be clubbed with father's
income

Answers to Chapter 12

Ans.1) Option (d) is correct Amt. of capital as on 01-04-2022 4,00,000


As per Sec 64(2), where any member of HUF transfers his + Amt. invested in business of Mr. Vishal 4,00,000
own property to HUF without consideration to HUF, & out of gift received from wife
thereafter HUF is partitioned then such portion of income Total Capital as on 01-04-2022 8,00,000
from that asset, as is received by the spouse of such Profits earned during PY 22-23 2,00,000
member, shall be clubbed in the hands of member Amt to be clubbed in hands of wife as per 1,00,000
Sec 64 [2L x 4L/8L]
Ans. 2) Option (b) is correct
Amount invested by Mr. Vishal on 01-04- 5,00,000 Ans. 3) Option (b) is correct
2021 Here Aaarav has gifted property to her wife who has further
(-) Loss incurred during FY 21-22 (1,00,000) gifted it to daughter in law. Hence, it would be considered

 
52 Prepared by : CA Sanchit Grover
  Multiple Choice Questions
  for CA Inter Taxation For May 2023 Exams
as indirect transfer by Aarav to daughter in law and hence Interest computed during FY 20-21 on 3,630
rental income received by Deepa would be clubbed in interest accrued during FY 19-20 (11% of
hands of Aarav as per Sec 64. Further, Standard deduction ₹ 33000) – not to be clubbed
of 30% of NAV would be allowed and hence Income u/h Interest computed during FY 21-22 on 4,029
House Property would be ₹ 2,10,000 interest accrued during FY 19-20 & FY 20-
21 (11% of ₹ 33000 + 3,630) – not to be
Ans. 4) Option (b) is correct clubbed
As per Sec 60, where any person transfers only income Interest computed during FY 22-23 on 4,473
derived from any asset to other person, without transferring interest accrued during FY 19-20 & FY 20-
the title of such asset to such other person, then income is 21 & FY 21-22 (11% of ₹ 33000 + 3,630 +
clubbed in the hands of transferor. 4,029) – not to be clubbed

Assuming that Pinky follows accrual system for taxing Other


Ans. 5) Option (c) is correct
sources income, only ₹ 4,473 shall be taxable during PY 22-
Interest on capital to be clubbed in hands of Anurag u/s 64
23
= ₹ 50,000 x 3.5L / 5L = ₹ 35,000. Remaining ₹ 15,000
interest shall not be clubbed and hence would be taxable
Ans. 9) Option (b) is correct
in hands of Shivani only. Further, there shall be no clubbing
Income u/h Capital Gains 2,50,000
in respect of Share of profit received from firm since it is
Agricultural income (exempt) -
exempt u/s 10(2A).
Income u/h PGBP 4,00,000
Ans. 6) Option (d) is correct Casual income 10,00,000
In case of HP transferred otherwise for adequate Income u/h HP received by daughter in 5,00,000
consideration to spouse, Sec 27 provides that transferor law but clubbed in hands of Hari as per
spouse shall be considered as Deemed Owner of the Sec 64
property and hence annual value of HP shall be taxable in GTI 21,50,000
his hands. However, where such HP is transferred otherwise
for adequate consideration to daughter in law, there is no Ans. 10) Option (d) is correct
such provision in Sec 27. Rather in such case, Sec 64 Share of profit received from firm is exempt u/s 10(2A) and
provides for clubbing of rental income received by daughter hence there shall be no clubbing. Further, since the gift
in law in hands of the transferor. received from spouse is not invested on 1st April 2022,
hence there shall no clubbing u/s 64 in respect of Interest
Ans. 7) Option (c) is correct on capital also.
As per Sec 64(1A), income of minor is clubbed in the hands
of that parent who has higher income. Further, exemption Ans. 11) Option (b) is correct
of ₹ 1,500 per child is allowed u/s 10(32) to such parent Share of profit from firm is exempt u/s 10(2A) and hence
while clubbing. there shall be no clubbing. Further Interest on capital to be
clubbed shall be computed as under:-
Ans. 8) Option (a) is correct 1,20,000 x 8L / 12L = ₹ 80,000
There shall be no clubbing of Interest earned by daughter
since she is not minor. However, in respect of interest Ans. 12) Option (c) is correct
earned by Pinky, clubbing u/s 64 shall apply, however there Since the amount of scholarship is fully exempt u/s 10(16),
will be no clubbing in respect of nothing is to be clubbed in hands of parents in this case.
Interest on ₹ 3L bonds to be clubbed in 33,000
hands of Pankaj

  
Prepared by : CA Sanchit Grover 53
   CHAPTER 13 : CARRY FORWARD AND SET OFF OF LOSSES

CHAPTER 13 CARRY FORWARD AND SET OFF OF LOSSES


Q1) Mr. A incurred short-term capital loss of ₹ (c) speculative business income of ₹ 40,000
10,000 on sale of shares through the National (d) All of the above
Stock Exchange. Such loss – Q5) Virat runs a business of manufacturing of shoes
(a) can be set-off only against short-term since the P.Y. 2020-21. During the P.Y. 2020-21
capital gains and P.Y. 2021-22, Virat had incurred business
(b) can be set-off against both short-term losses. For P.Y. 2022-23, he earned business
capital gains and long term capital gain profit (computed) of ₹ 3 lakhs. Considering he
(c) can be set-off against any head of may/may not have sufficient business income
income. to set off his earlier losses, which of the
(d) not allowed to be set-off following order of set off shall be considered:
Q2) According to section 80, no loss which has not (He does not have income from any other
been determined in pursuance of a return filed source)
in accordance with the provisions of section (a) First adjustment for loss of P.Y. 2020-21,
139(3), shall be carried forward. The exceptions then loss for P.Y. 2021-22 and then
to this are – unabsorbed depreciation, if any.
(a) Loss from specified business under (b) First adjustment for loss of P.Y. 2021-22,
section 73A then loss for P.Y. 2020-21 and then
(b) Loss under the head “Capital Gains” and unabsorbed depreciation, if any.
unabsorbed depreciation carried forward (c) First adjustment for unabsorbed
under section 32(2) depreciation, then loss of P.Y. 2021-22
(c) Loss from house property and and then loss for P.Y. 2020-21, if any.
unabsorbed depreciation carried forward (d) First adjustment for unabsorbed
under section 32(2) depreciation, then loss of P.Y. 2020-21
(d) Loss from speculation business under and then loss for P.Y. 2021-22, if any
section 73 Q6) Mr. Ravi incurred loss of ₹ 4 lakh in the P.Y.
Q3) Brought forward loss from house property of ₹ 2022-23 in leather business. Against which of
3,10,000 of A.Y. 2022- 23 is allowed to be set- the following incomes earned during the same
off against income from house property of A.Y. year, can he set-off such loss?
2023-24 of ₹ 5,00,000 to the extent of – (i) Profit of ₹ 1 lakh from apparel business
(a) ₹ 2,00,000 (ii) Long-term capital gains of ₹ 2 lakhs on
(b) ₹ 3,10,000 sale of jewellery
(c) ₹ 2,50,000 (iii) Salary income of ₹ 1 lakh Choose the
(d) ₹ 1,00,000 correct answer:
Q4) Mr. Rohan incurred loss of ₹ 3 lakh in the P.Y. (a) First from (ii) and thereafter from (i); the
2022-23 in retail trade business. Against which remaining loss has to be carried forward.
of the following income during the same year, (b) First from (i) and thereafter from (ii) and
can he set-off such loss? (iii)
(a) profit of ₹ 1 lakh from wholesale cloth (c) First from (i) and thereafter from (iii); the
business remaining loss has to be carried forward
(b) long-term capital gains of ₹ 1.50 lakhs on (d) First from (i) and thereafter from (ii); the
sale of land remaining loss has to be carried forward
 
54 Prepared by : CA Sanchit Grover
  Multiple Choice Questions
  for CA Inter Taxation For May 2023 Exams

Q7) During the A.Y. 2022-23, Mr. A has a loss of ₹ 8 Loss from medical business 1,20,000
lakhs under the head “Income from house Long term capital gains 1,60,000
property” which could not be set off against any Income from other sources 80,000
other head of income as per the provisions of
What shall be the gross total income of Mr.
section 71. The due date for filing return of
Kumar for A.Y. 2023-24?
income u/s 139(1) in case of Mr. A has already
(a) ₹ 4,40,000
expired and Mr. A forgot to file his return of
(b) ₹ 3,20,000
income within the said due date. However, Mr.
(c) ₹ 1,60,000
A filed his belated return of income for A.Y.
(d) ₹ 4,80,000
2022-23. Now, while filing return of income for
Q9) X Ltd. files its return of loss for the A.Y. 2023-24
A.Y. 2023-24, Mr. A wishes to set off the said
on 01.12.2023. The following data is taken from
loss against income from house property for the
return submitted by the company
P.Y. 2022-23. Determine whether Mr. A can
Business Loss for P.Y. 2022-23 1,70,000
claim the said set off.
(before depreciation)
(a) No, Mr. A cannot claim set off of loss of ₹
Depreciation 30,000
8 lakhs during A.Y. 2023-24 as he failed
Short term capital loss 45,000
to file his return of income u/s 139(1) for
Long term capital gain 10,000
A.Y. 2022-23.
Income from other sources 23,000
(b) Yes, Mr. A can claim set off of loss of ₹ 2
lakhs, out of ₹ 8 lakhs, from his income Unabsorbed deprecation 75,000
from house property during A.Y. 2023-24, pertaining to A.Y. 2019-20 and A.Y.
if any, and the balance has to be carried 2020-21 which has been
forward to A.Y. 2024-25. determined in pursuance of return
(c) Yes, Mr. A can claim set off of loss of ₹ 2 filed
lakhs, out of ₹ 8 lakhs, from his income Compute the amount of loss that can be carried
from any head during A.Y. 2023-24 and forward by X Ltd.
the balance can be set off against any (a) ₹1,05,000
other head of income (b) ₹30,000
(d) Yes, Mr. A can claim set off of loss of ₹ 8 (c) ₹2,87,000
lakhs during A.Y. 2023-24 from his (d) Nil (MTP Nov 2019)
income from house property, if any, and Q10) During the A.Y. 2023-24, Mr. Kabir has a loss of
the balance has to be carried forward to ₹ 6 lakhs under the head “Income from house
A.Y. 2024-25. property”, loss of ₹ 5 lakhs from business of
Q8) The details of income/loss of Mr. Kumar for A.Y. profession and income of ₹ 3 lakhs from long
2023-24 are as follows: term capital gains. He filed his return of income
Income from Salary 5,20,000 for the A.Y. 2023-24 on 31.12.2023. Determine
(Computed) the total income of Mr. Kabir for A.Y. 2023-24
Loss from self-occupied house 95,000 and the amount of loss which can be carried
property forward in a manner most beneficial to him?
(a) Total income Nil; loss of ₹ 4,00,000 from
Loss from let-out house 2,25,000
house property and loss of ₹ 4,00,000
property
from business or profession.
Loss from specified business 2,80,000
u/s 35AD

  
Prepared by : CA Sanchit Grover 55
   CHAPTER 13 : CARRY FORWARD AND SET OFF OF LOSSES
(b)
Total income ₹ 1,00,000; loss of ₹ be carried forward for the subsequent
4,00,000 from house property. assessment year?
(c) Total income Nil; No loss is allowed to be (a) Total income ₹ 2,00,000 and loss from
carried forward. house property of ₹ 2,50,000 and
(d) Total income Nil; loss of ₹ 6,00,000 from business loss of ₹ 20,000 to be carried
house property. (MTP May 2021) forward to subsequent assessment year.
Q11) Mr. Arpan (aged 35 years) submits the following (b) Total income ₹ 1,60,000 and loss from
particulars for the purpose of computing his house property of ₹ 2,30,000 to be carried
total income: forward to subsequent assessment year.
Income from salary (computed) 4,00,000 (c) Total income ₹ 1,80,000 and loss from
Loss from let-out house property (-) 2,20,000 house property of ₹ 2,30,000 and
Brought forward loss from let-out (-) 2,30,000 business loss of ₹ 20,000 to be carried
house property for AY 2022-23 forward to subsequent assessment year.
Business Loss (-) 1,00,000 (d) Total income is Nil and loss from house
Bank Interest received (FD) 80,000 property of ₹ 70,000 to be carried forward
Compute the total income of Mr. Arpan for the to subsequent assessment year.
A.Y. 2023-24 and the amount of loss that can (RTP May 2021)

Answers to Chapter 13

Ans. 1) Option (b) is correct for maximum 8 years, loss of PY 20-21 should be given
Any STCL, whether under Sec 111A or not, can be set off priority over loss of PY 21-22
both against STCG and LTCG
Ans. 6) Option (d) is correct
Ans. 2) Option (c) is correct Loss from non-speculative business has to be first set off
In case of loss u/h House property and unabsorbed against income u/h PGBP and thereafter it can be set off
depreciation, loss can be carried forward even if the return against income under any other head, except Salary income.
of income has not be filed before due date
Ans. 7) Option (d) is correct
Ans. 3) Option (b) is correct In case of Loss u/h HP, even if the return is filed after due
There is no limit on amount of B/f HP loss that can be set date, entire loss can be carried forward and be eligible for
off against Income u/h HP of this PY. Hence, the entire ₹ set-off against income u/h HP of subsequent year (Further,
3,10,000 of B/f loss can be set off the limit of ₹ 2L doesn’t apply in case of carry forward, it is
only on Inter-head set-off in the same PY)
Ans. 4) Option (d) is correct
Loss from non-speculative business can be set off against Ans. 8) Option (a) is correct
Profits of speculative business as well as Non-speculative Income from Salary 5,20,000 3,20,000
business. Further, in Inter head set off, such loss can be (Computed)
against any head of income, except Salary. Loss from House property (2,00,000)
head
Ans. 5) Option (a) is correct (remaining ₹ 1,20,000 loss
Since unabsorbed depreciation can be carried forward for has to be carried forward)
unlimited time, it should be given last priority. Further, since Loss from specified Nil Nil
loss from non-speculative business can be carried forward business u/s 35AD (can’t

 
56 Prepared by : CA Sanchit Grover
  Multiple Choice Questions
  for CA Inter Taxation For May 2023 Exams

be set –off against any Since the return has been filed after due date, loss u/h PGBP
other income) cannot be carried forward but there is no restriction on its
Loss from medical business (1,20,000) 40,000 Inter head set off. Further, in respect of loss u/h House
Long term capital gains 1,60,000 property, there shall not be any restriction on Carry forward
Income from other sources 80,000 80,000 or Inter head set off. Hence, it would be prudent to first set
Gross Total Income 4,40,000 off PGBP loss against LTCG and carry forward House
Property loss
Ans. 9) Option (a) is correct
Income under the head Other Sources 23,000 Ans. 11) Option (a) is correct
PGBP Loss before depreciation of PY 22- (23,000) Income from salary (computed) 4,00,000
23 – Remaining loss can’t be carried Loss from let-out house property (-) 2,00,000
forward since belated return (Maximum of ₹ 2L loss can be set-off
LTCG 10,000 from any other head, remaining ₹ 20,000
Less:- STCL set off (Remaining STCL (10,000) to be carried forward)
can’t be c/f since belated return) Brought forward loss from let-out house Nil
Gross Total Income Nil property for AY 2022-23 (B/f HP loss
Total unabsorbed depreciation that can 1,05,000 cannot be set off against any head other
be carried forward (even if belated return than HP)
is filed) Bank Interest received (FD) 80,000
Business Loss (Maximum to the extent (-) 80,000
Ans. 10) Option (d) is correct of ₹ 80,000 can be set off against OS
Income under the head Capital Gain 3,00,000 income, can’t be set off against Salary,
(-) Loss under PGBP head (3,00,000) hence remaining ₹ 20,000 PGBP loss to
GTI₹ Nil be carried forward)
Gross Total Income 2,00,000

   

  
Prepared by : CA Sanchit Grover 57
   CHAPTER 14 : TDS AND TCS

CHAPTER 14 TDS AND TCS


Q1) Mr. X, a resident, is due to receive ₹ 4.50 lakhs credited equally to each of their account. Mr.
on 31.3.2023, towards maturity proceeds of LIC Rajesh approached his tax consultant to seek
policy taken on 1.4.2019, for which the sum clarity in relation to deduction of tax on payment
assured is ₹ 4 lakhs and the annual premium is of the rent. He informed his consultant that he
₹ 1,25,000. Mr. Z, a resident, is due to receive ₹ occupied such flat for his personal
95,000 on 1.10.2022 towards maturity proceeds accommodation and his receipts from his
of LIC policy taken on 1.10.2013 for which the profession during the previous year 2021-22
sum assured is ₹ 90,000 and the annual was ₹ 58 lakhs. As tax consultant, choose the
premium is ₹ 10,000. correct answer
(a) Tax is required to be deducted on income (a) No tax at source is required to be
comprised in maturity proceeds payable deducted since the rental payments are
to Mr. X and Mr. Z towards flat occupied for personal
(b) Tax is required to be deducted on income purpose
comprised in maturity proceeds payable (b) Tax is required to be deducted at source
to Mr. X since the rent payment exceeds ₹
(c) Tax is required to be deducted on income 2,40,000 and Dr. Rajesh is an individual
comprised in maturity proceeds payable having gross receipts from profession
to Mr. Z exceeding ₹ 50 lakh in the preceding
(d) No tax is required to be deducted on financial year.
income comprised in maturity proceeds (c) No tax is required to be deducted at
payable to either Mr. X or Mr. Z source since the rent credited to each co-
Q2) An amount of ₹ 40,000 was paid to Mr. X on owner is less than ₹ 2,40,000
1.7.2022 towards fees for professional services (d) No tax is required to be deducted at
without deduction of tax at source. source since Dr. Rajesh’s gross receipts
Subsequently, another payment of ₹ 50,000 was during the preceding financial year were
due to Mr. X on 28.02.2023, from which tax less than ₹ 1 crore
@10% (amounting to ₹ 9,000) on the entire Q4) Mr. Nihar maintains a savings A/c and a current
amount of ₹ 90,000 was deducted and the net A/c in Mera Bank Ltd. The details of withdrawals
amount was paid on the same day to Mr. X. on various dates during the previous year 2022-
However, this tax of ₹ 9,000 was deposited only 23 are as follows:
on 22.6.2023. The interest chargeable under Date of Cash Saving Current
section 201(1A) would be: withdrawal Account Account
(a) ₹ 320 05-04-2022 15,00,000 -
(b) ₹ 860 10-05-2022 - 22,00,000
(c) ₹ 1,620 25-06-2022 20,00,000 -
(d) ₹ 540 17-07-2022 - 5,00,000
Q3) Mr. Ramesh, Mr. Mahesh and Mr. Suresh, jointly 28-10-2022 35,00,000 -
owned a flat in Mathura, which was let out to 10-11-2022 - 38,00,000
Dr. Rajesh from 01.04.2022. The annual rent 12-12-2022 25,00,000 -
paid by Dr. Rajesh for the flat was ₹ 5,40,000,
 
58 Prepared by : CA Sanchit Grover
  Multiple Choice Questions
  for CA Inter Taxation For May 2023 Exams

Mr. Nihar regularly files his return of income. Is return of income for P.Y. 2019-20, P.Y. 2020-21
Mera Bank Limited required to deduct tax at and P.Y. 2021-22 respectively?
source on the withdrawals made by Mr. Nihar (a) ₹ 1,00,000 and ₹ 2,40,000
during the previous year 2022-23? If yes, what (b) Nil and ₹ 40,000
would the amount of tax deducted at source? (c) ₹ 60,000 and ₹ 1,00,000
(a) TDS of ₹ 3,20,000 is required to be (d) ₹ 50,000 and ₹ 1,20,000
deducted (MTP May 2020)
(b) No, TDS is not required to be deducted as Q7) Mr. Ram acquired a house property at Chennai
the cash withdrawal does not exceed ₹ 1 from Mr. Satyam, a resident, for a consideration
crore neither in saving account nor in of ₹ 85 lakhs, on 23.8.2022 (SDV of Property was
current account 80L on such date). On the same day, Mr. Ram
(c) TDS of ₹ 3,00,000 is required to be made two separate transactions, thereby
deducted. acquiring an urban plot in Gwalior from Mr.
(d) TDS of ₹ 1,20,000 is required to be Vipun, a resident, for a sum of ₹ 50 lakhs (SDV
deducted. (RTP Nov 2020) being 48L) and rural agricultural land from Mr.
Q5) Mr. P is a professional who is responsible for Danish, a resident, for a consideration of ₹ 75
paying a sum of ₹ 2,00,000 as rent for use of lakhs (SDV being 82L). Which of the following
building to Mr. Harshit, a resident, for the month statements are correct assuming that in the
of February, 2023. The gross receipts of Mr. P consideration amounts as aforementioned all
are as under: From 01.04.2021 to 31.03.2022: ₹ the charges incidental to transfer of the
55,00,000 From 01.04.2022 to 28.02.2023: ₹ immovable property are included?
45,00,000 Whether Mr. P is responsible for (a) No tax deduction at source is required in
deducting any tax at source from the rent of ₹ respect of any of the three payments.
2,00,000 payable to Mr. Harshit? (b) TDS@1% is attracted on all the three
(a) Tax at source is required to be deducted payments.
u/s 194-I at the rate of 10% (c) TDS@1% on ₹ 85 lakhs and ₹ 50 lakhs
(b) Tax at source is required to be deducted are attracted. No TDS on payment of ₹ 75
u/s 194-IB at the rate of 5%. lakhs for acquisition of rural agricultural
(c) Tax at source is required to be deducted land.
u/s 194-IB at the rate of 10%. (d) TDS@1% on ₹ 85 lakhs is attracted. No
(d) No tax is required to be deducted at TDS on payments of ₹ 50 lakhs and ₹ 75
source. lakhs (MTP May 2020)
Q6) Mr. A has two bank accounts maintained with Q8) Which of the following details/evidences are
ICICI Bank and HDFC Bank. From 01.04.2022 till required to be furnished by an employee to
31.03.2023, Mr. A withdrew the following his/her employer in respect of deduction of
amounts as cash from both the said accounts; interest under the head "Income from house
HDFC Bank: ₹ 50 Lakh ICICI Bank: ₹ 120 Lakh property", when the employer is estimating the
What shall be the amount of tax to be deducted total income of the employee for the purpose of
at source u/s 194N by HDFC Bank and ICICI tax deduction at source u/s 192?
Bank, respectively, while making payment in (i) Amount of Interest payable or paid
cash to Mr. A assuming Mr. A has filed his (ii) Rate of interest payable or paid
(iii) Name of the lender
  
Prepared by : CA Sanchit Grover 59
   CHAPTER 14 :
(iv) Address of the lender (a) No tax is deductible at source
(v) PAN or Aadhaar number as the case may (b) Tax is deductible@ 1%
be, of the lender (c) Tax is deductible@ 5%
(vi) TAN of the lender (d) Tax is deductible@ 10%
Choose the correct answer: Q13) ABC Ltd. wants 400 customized backpacks
(a) (i), (iii), (v) which will be distributed by the company to its
(b) (i), (iii), (iv), (v) employees on the annual event. ABC Ltd. hires
(c) (ii), (iv), (v), (vi) a local vendor for this task and ABC Ltd. informs
(d) (i), (ii) (RTP May 2021) the local vendor about its specifications for the
Q9) Mr. X paid fees for professional services of ₹ backpacks. The local vendor procures its own
40,000 to Mr. Y, who is engaged only in the raw material and supplies the required
business of operation of call centre, on backpacks to the Company. He charges
15.7.2022. Tax is to be deducted by Mr. X at the ₹1,00,000 for the backpacks from ABC Ltd.
rate of – While making payment to the vendor, at what
(a) 0.75% rate ABC Ltd. is required to deduct tax at source?
(b) 1% (a) 1%
(c) 1.5% (b) 2%
(d) 2% (c) 10%
Q10) A firm pays salary and interest on capital to its (d) No tax is required to be deducted at
resident partners. The salary and interest paid source.
fall within the limits specified in section 40(b). Q14) Mr. Prakash is employed with XYZ Ltd. from
Which of the following statements is true? 05.11.2018. He resigned on 31.03.2023 and
(a) Tax has to be deducted u/s 192 on salary wants to withdraw the accumulated balance of
and u/s 194A on interest employer’s contribution in his EPF Account i.e.,
(b) Tax has to be deducted u/s 192 on salary ₹ 55,000. The tax deducted on such withdrawal
but no tax needs to be deducted on would be –
interest (a) ₹ 500 u/s 192
(c) No tax has to be deducted on salary but (b) ₹ 5,500 u/s 192
tax has to be deducted u/s 194A on (c) ₹ 4,125 u/s 192A
interest (d) ₹ 5,500 u/s 192A (MTP Nov 2021)
(d) No tax has to be deducted at source on Q15) Dr. Sargun maintained two bank A/cs, one
either salary or interest current A/c with Canara Bank for her profession
Q11) Mr. X, a resident Indian, wins ₹ 10,000 in a and a saving bank A/c with State Bank of India.
lottery. Which of the statement is true? The following are the details of her withdrawals
(a) Tax is deductible u/s 194B@30% from these A/cs during the previous year 2022-
(b) Tax is deductible u/s 194B@ 30.9% 23:
(c) No tax is deductible at source Date of Canara SBI
(d) None of the above Withdrawal Bank
Q12) ₹ 2 lakh is paid to Mr. Vallish, a resident 25-04-2022 25,00,000 -
individual on 15.3.2023 by the State of Haryana 27-04-2022 - 15,50,000
on compulsory acquisition of his urban 31-08-2022 29,00,000 -
agricultural land.
 
60 Prepared by : CA Sanchit Grover
  Multiple Choice Questions
  for CA Inter Taxation For May 2023 Exams

01-09-2022 14,20,000 - (c) Such dividend is taxable in the hands of


05-09-2022 - 14,00,000 Mr. T. However, Indian company is not
07-10-2022 18,21,000 - required to deduct tax at source since it
11-12-2022 26,23,000 - does not exceed ₹ 5,000.
12-02-2023 7,56,000 - (d) Such dividend is exempt in the hands of
25-03-2023 - 16,13,000 Mr. T. Hence, Indian company is not
required to deduct tax at source.
She furnished her return of income for the A.Y.
(RTP Dec 2021)
2022-23 and A.Y. 2021-22 on or before the time
Q17) TPR & Co., a partnership firm selling its product
limit prescribed u/s 139(1). However, for the A.Y.
X through the digital facility provided by MKY
2020-21 and A.Y. 2019-20, she has furnished
Limited (an E-commerce operator). MKY Limited
her return of income belatedly. Is any tax
has credited in its books of account, the account
deductible at source u/s 194N on the
of TPR & Co. on 31st January, 2023 by sum of ₹
withdrawals made by Dr. Sargun from Canara
4,80,000 for the sale of product X made during
Bank and SBI Bank? If yes, at what rate and
the month of January 2023. Out of ₹ 4,80,000, it
what amount?
made payment for ₹ 4,00,300 on 3rd February,
(a) TDS is deductible at source on ₹
2023. Further, Mr. Pawan, who purchased the
1,00,20,000 @ 5% by Canara Bank and
product X through the facility provided by MKY
tax is deductible at source @2% on ₹
Limited, has made the payment of sum of ₹
25,63,000 by SBI.
40,000 directly to TPR & Co. on 15th January,
(b) TDS is deductible at source on ₹
2023. Which statement is correct regarding
20,20,000 @ 5% by Canara Bank and no
requirement of deduction of tax at source by
tax is deductible by SBI.
MKY Limited?
(c) TDS is deductible at source on ₹
(a) No tax is required to be deducted at
20,20,000 @ 2% by Canara Bank and no
source.
tax is deductible by SBI.
(b) MKY Limited is required to deduct tax at
(d) TDS is deductible at source on ₹
source ₹ 4,800 under section 194C.
1,00,20,000 @ 5% by Canara Bank and
(c) MKY Limited is required to deduct tax at
tax is deductible at source @5% on ₹
source ₹ 3,900 under section 194-O.
25,63,000 by SBI. (RTP May 2021)
(d) MKY Limited is required to deduct tax at
Q16) Mr. T, an Indian Citizen and resident of India,
source ₹ 5,200 under section 194-O.`
earned dividend income of ₹ 4,500 from an
(MTP May 2021)
Indian company, which was declared on
Q18) Mr. Kumar made the following cash
1.10.2022 and paid in cash to Mr. T. What are
withdrawals during the P.Y.2022-23 -
the tax implications with respect to the dividend
Date Amount From
in the hands of Mr. T and Indian Company?
01-06-2022 ₹ 70 Lakhs Canara Bank
(a) Such dividend is taxable in the hands of
01-07-2022 ₹ 45 Lakhs HDFC
Mr. T and Indian company is required to
01-08-2022 ₹ 50 Lakhs Canara Bank
deduct tax at source @7.5%.
01-09-2022 ₹ 15 Lakhs HDFC
(b) Such dividend is taxable in the hands of
Mr. T and Indian company is required to
deduct tax at source @10%.
  
Prepared by : CA Sanchit Grover 61
   CHAPTER 14 :
01-10-2022 ₹ 60 Lakhs Repco Bank dealer firm, for his daughter’s hearing and ₹
(Cooperative 21,00,000 from same Vijay Associates for
Bank brokerage service provided in relation to
01-11-2022 ₹ 10 Lakhs SBI purchase of one property. Both bills were raised
01-12-2022 ₹ 10 Lakhs Repco Bank on 21-12-2022 but payment were made in
02-01-2023 ₹ 15 Lakhs HDFC instalments.
10-01-2023 ₹ 15 Lakhs HDFC 1st Instalment of ₹ 5,00,000 as advance was
20-01-2023 ₹ 20 Lakhs Repco Bank payment on 15-11-2022, 2nd Instalment of ₹
01-02-2023 ₹ 15 Lakhs Repco Bank 45,00,000 on 25-03-2023 and balance amount
10-03-2023 ₹ 75 Lakhs SBI ₹ 13,00,000 on 11-05-2023.
20-02-2023 ₹ 15 Lakhs HDFC Determine the TDS liability for Mr. Ajay Sahu, if
01-03-2023 ₹ 15 Lakhs SBI any, for A.Y. 2023-24?
(a) ₹ 2,50,000
Which of the above banks are required to (b) ₹ 3,15,000
deduct tax at source on cash withdrawals (c) ₹ 65,000
made by Mr. Kumar in the P.Y.2021-22 if he (d) Nil
regularly files his return of income? Q20) While deciding liability of an individual to
(a) Canara Bank & HDFC deduct tax on payment of fees for professional
(b) HDFC, SBI & Repco services, which of the following is immaterial –
(c) HDFC, Repco & Canara Bank (a) Amount paid to professional
(d) HDFC & Repco (MTP May 2022) (b) Turnover of financial year immediately
Q19) Mr. Ajay Sahu, proprietor of M/s Blue Bird preceding financial year in which
Enterprises having turnover of ₹ 65 lakhs and payment made
not subject to tax audit under the Income-tax (c) Turnover of financial year in which
Act, 1961 during P.Y. 2021-22, has received two payment is made
bills for payment. The first bill is for ₹ 42,00,000 (d) Amount of fees for professional services.
from Vijay Associates, an advocate and property (MTP Nov 2021)

Answers to Chapter 14

Ans. 1) Option (b) is correct Interest for late deposit of TDS (₹ 9,000 x 540
In case of Mr. X, since the annual premium exceeds 10% of th
1.5% x Period from 28 Feb 23 to 22 June nd

sum assured, exemption u/s 10(10D) shall not be available. 23)


Further, maturity amount is also more than ₹ 1L and hence Total Interest u/s 201(1A) 860
TDS u/s 194DA shall be applicable. However, in case of Mr.
Z since the maturity amount is less than ₹ 1L, no TDS u/s Ans. 3) Option (c) is correct
194DA is applicable. In the given case, Deductor (i.e. Mr. Rajesh) has receipts
from profession > 50L , hence we need to evaluate Sec 194-
Ans. 2) Option (b) is correct I & not 194-IB. Sec 194-I covers rental payment, whether
Interest for late deduction of TDS (₹ 4,000 x 320 made for personal purpose or business purpose. However,
st th
1% x Period from 1 July 22 to 28 Feb 23) threshold for TDS u/s 194-I is ₹ 2,40,000 (Further CBDT has
clarified that in case HP is owned by co-owners, limit of ₹

 
62 Prepared by : CA Sanchit Grover
  Multiple Choice Questions
  for CA Inter Taxation For May 2023 Exams
2,40,000 has to be considered for each owner separately). Since salary paid by firm to partners is not taxable u/h
Since Rent payable to each owner is RS. 1,80,000 in this Salary in hands of partner, hence TDS u/s 192 shall not be
case, hence no TDS u/s 194I is applicable in the given case. deductible. Further, as per Sec 194A, no TDS is to be
deducted by firm on interest paid to partners.
Ans. 4) Option (d) is correct
Since Mr. Nihar is regularly filing his return of income, Ans. 11) Option (c) is correct
threshold limit shall be ₹ 1 Crore. Since Total withdrawals Since the amount of winning doesn’t exceed ₹ 10,000, no
from both the accounts is ₹ 1.6 Crore, TDS u/s 194N shall TDS shall be applicable.
be 2% of ₹ 60L = ₹ 1,20,000
Ans. 12) Option (a) is correct
Ans. 5) Option (d) is correct As per Sec 194LA, TDS is to be deducted only in case of
Since Payer (i.e. Mr. P) has professional receipts > 50L in PY Compulsory acquisition of any immovable property (not
21-22, he shall be covered by Sec 194I and not 194IB. being agricultural land, whether such land is in urban or
Further, since the total rent paid during PY 22-23 < ₹ rural area). Hence, in the given case, no TDS shall be
2,40,000 no TDS u/s 194I shall be applicable in this case. applicable irrespective of the amount of compensation

Ans. 6) Option (b) is correct Ans. 13) Option (d) is correct


Since Mr. A has filed his return of income for PY 2019-20 Under Sec 194C, the definition of ‘work’ includes sale of
as well as PY 2020-21, applicable threshold for Sec 194N goods manufactured by seller according to specifications of
shall be ₹ 1 Crore. Further, this threshold of ₹ 1 Cr. shall be buyer only if material is procured from the buyer. In the
applicable separately from bank accounts maintained with given case, since local vendor procures his material himself
different Banks. Accordingly, HDFC Bank is not required to to manufacture & sell backpacks, this sale of goods would
deduct TDS u/s 194N since withdrawals < 1Cr. during PY not qualify as ‘work’ u/s 194C and hence no TDS will be
22-23. Further, TDS to be deducted by ICICI Bank = 2% of ₹ applicable. Further, in this case, conditions of Sec 194Q and
20L = ₹ 40,000 194O are also not getting satisfied.

Ans. 7) Option (c) is correct Ans. 14) Option (d) is correct


Sec 194-IA provides for deduction of TDS @1% where sale Since EPF balance has been withdrawn before expiry of 5
consideration or SDV for Land & Building (other than rural years, hence it shall be taxable. Accordingly, TDS u/s 192A
agricultural land) is ₹ 50L or more (Even when exactly ₹ 50L shall be deductible @10% of ₹ 55,000 since the amount is
TDS is applicable). Further TDS is to be applied on the more than threshold limit of ₹ 50,000.
amount of Sale consideration or SDV, whichever is higher. Note:- It has assumed that employment has not been
Thus, in the given case Mr. Ram is required to deduct TDS terminated due to reasons beyond the control of the
@1% while paying ₹ . 85L to Mr. Satyam and also deduct employee
TDS @1% while paying ₹ 50L to Vipun (in both these cases,
Sale consideration is more than SDV). Ans. 15) Option (c) is correct
In this given case, due date of filing return u/s 139(1) has
Ans. 8) Option (b) is correct expired for AY 2021-22, AY 2020-21 and AY 2019-20 as on
Only the amount of interest for which deduction u/s 24(b) 1st April 2022. While return for AY 20-21 and AY 19-20 were
is to be claimed has to be furnished by employee, not the filed after due date, return for AY 2021-22 was filed before
rate of interest. Further, only PAN (or aadhar) of lender is to the due date prescribed u/s 139(1). Accordingly, proviso to
be given, there is no need to furnish TAN of lender. Sec 194N shall not be applicable here, the threshold limit
for checking TDS applicability shall be ₹ 1 Crore. Total
Ans. 9) Option (d) is correct withdrawals in cash from Canara Bank = ₹ 1,20,20,000
Since withdrawals exceed ₹ 1 Crore, TDS @ 2% shall be
Ans. 10) Option (d) is correct applicable on ₹ 20,20,000

  
Prepared by : CA Sanchit Grover 63
   CHAPTER 14 :
Ans. 16) Option (b) is correct Total Cash
Dividend is taxable in the hands of shareholder u/h Other withdrawals
sources as a normal income. Further, Sec 194 provides that Canara Bank ₹ 1,20,00,000
no TDS is required to be deducted by resident company HDFC ₹ 1,05,00,000
where amt of dividend is not more than ₹ 5,000 and such Repco Bank ₹ 1,05,00,000
amount has been paid to an individual in any mode other SBI ₹ 1,00,00,000
than cash. In the given case, since dividend of ₹ 4,500 has Hence, SBI is not required to deduct TDS in above case since
been paid in cash hence the above exception shall not total withdrawals do not exceed ₹ 1 Crore
apply and TDS @10% shall be applicable u/s 194.
Ans. 19) Option (b) is correct
Ans. 17) Option (d) is correct Since Mr. Ajay doesn’t have turnover exceeding ₹ 1 Crore in
Under Sec 194-O, TDS is to be deducted by ECO @1% at the preceding FY, he is not liable to deduct TDS u/s 194J.
time of credit or payment whichever is earlier. Further, Sec However, Sec 194M shall become applicable here since
194-O clarifies that where payment is made directly by Fees for professional services paid exceeds RS. 50L. Hence
customer to Electronic commerce participant, it would also Amount to be deducted shall be 5% of (42L + 21L) = ₹
deemed to have been made through ECO and hence ECO 3,15,000
is required to deduct TDS on that as well. Hence TDS u/s
194-O would be 1% of ₹ (4,80,000 + 40,000) = ₹ 5,200. Ans. 20) Option (c) is correct
Further, Sec 194C shall have no applicability in this case In case Payer is individual or HUF, he is required to deduct
since goods have not been manufactured by seller using TDS u/s 194J on payment of professional services fees if his
material purchased from buyer. turnover/receipts from business or profession in
immediately preceding FY exceeds ₹ 1Cr or 50L. In case it
Ans. 18) Option (c) is correct is not, he may be required to deduct TDS u/s 194M. But
Since he regularly files return, proviso to Sec 194N shall not under both these sections, turnover of the payer in relevant
be applicable and hence threshold limit shall be ₹ 1 Crore PY is not relevant.
for each bank

   

 
64 Prepared by : CA Sanchit Grover
  Multiple Choice Questions
  for CA Inter Taxation For May 2023 Exams

CHAPTER 15 ADVANCE TAX


Q1) Mr. A, whose total sales is ₹ 201 lakhs, declares lotteries. What shall be his advance tax liability
profit of ₹ 10 lakhs for the F.Y. 2022-23. He is for A.Y. 2023-24 if all tax deductible at source
liable to pay advance tax – has been duly deducted and remitted to the
(a) in one instalment credit of Central Government on time? Assume
(b) in two instalments he does not opt to pay tax under section
(c) in three instalments 115BAC.
(d) in four instalments (a) ₹ 2,20,000 + Cess ₹ 8,800 = ₹ 2,28,800,
Q2) Mr. Raj (a non-resident and aged 65 years) is a being the tax payable on total income of
retired person, earning rental income of ₹ ₹ 12 lakhs
40,000 per month from a property located in (b) ₹ 2,10,000 + Cess ₹ 8,400 = ₹ 2,18,400,
Delhi. He is residing in Canada. Apart from being the tax payable on lottery income
rental income, he does not have any other of ₹ 7 lakhs
source of income. Is he liable to pay advance (c) ₹ 10,000 + Cess ₹ 8,800 = ₹ 18,800, being
tax in India? the net tax payable on salary income,
(a) Yes, he is liable to pay advance tax in since tax would have been deducted at
India as he is a non-resident and his tax source from lottery income.
liability in India exceeds ₹ 10,000. (d) Nil
(b) No, he is not liable to pay advance tax in Q5) An interior decorator has opted for presumptive
India as his tax liability in India is less taxation scheme under section 44ADA for A.Y.
than ₹ 10,000. 2023-24. –
(c) No, he is not liable to pay advance tax in (a) He is liable to pay advance tax on or
India as he has no income chargeable before 15.3.2023
under the head “Profits and gains of (b) He is not liable to advance tax
business or profession” and he is of the (c) He is liable to pay advance tax in three
age of 65 years. instalments i.e., on or before 15.9.2022,
(d) Both (b) and (c) 15.12.2022 and 15.3.2023
Q3) The benefit of payment of advance tax in one (d) He is liable to pay advance tax in four
installment on or before 15th March is available instalments i.e., on or before 15.6.2022,
to assessees computing profits on presumptive 15.9.2022, 15.12.2022 and 15.3.2023
basis Q6) Which of the following persons is/are liable to
(a) only under section 44AD pay advance tax as per the provisions of
(b) under section 44AD and 44ADA Income-tax Act, 1961?
(c) under section 44AD and 44AE I. A resident individual aged 62 years
(d) under section 44AD, 44ADA and 44AE having only income from capital gains of
Q4) Mr. Jha, an employee of FX Ltd, attained 60 ₹ 20,00,000 (estimated) during P.Y. 2022-
years of age on 15.05.2022. He is resident in 23.
India during F.Y. 2022-23 and earned salary II. A resident individual aged 58 years
income of ₹ 5 lakhs (computed). During the having only income from other sources of
year, he earned ₹ 7 lakhs from winning of

  
Prepared by : CA Sanchit Grover 65
   CHAPTER 15 : ADVANCE TAX
₹ 2,00,000 (estimated) during P.Y. 2022- (c) Mr. Jai is liable to pay advance tax but Mr.
23 Veeru is not liable to pay advance tax
III. A private company having estimated total (d) Mr. Veeru is liable to pay advance tax but
income of ₹ 1,00,000 during P.Y. 2022-23. Mr. Jai is not liable to pay advance tax
IV. A partnership firm which has estimated (MTP May 2022)
its total income to be Nil for P.Y. 2022-23. Q8) Mr. Kabir (a non-resident and aged 70 years) is
V. A HUF having estimated total income of a retired person, earning rental income of
₹ 6,00,000 during P.Y. 2022-23 ₹45,000 per month from a property located in
(a) I, III, V Mumbai. He is residing in Canada. Apart from
(b) I, II, III, IV, V rental income, he does not have any other
(c) III, V source of income. Is he liable to pay advance
(d) III, IV, V tax in India?
Q7) Mr. Jai, a resident Indian aged 60 years, won ₹ a) Yes, he is liable to pay advance tax in
9 lakhs and Mr. Veeru, resident Indian aged 55 India as he is a non-resident and his tax
years, won ₹ 8 lakhs from lotteries. Tax liability in India exceeds ₹10,000.
deductible at source under section 194B was b) No, he is not liable to pay advance tax in
duly deducted. Assuming that this is the only India as his tax liability in India is less
source of income of Mr. Jai and Mr. Veeru for than ₹10,000.
A.Y. 2023 -24, are Mr. Jai and Mr. Veeru liable to c) No, he is not liable to pay advance tax in
pay advance tax for that year? India as he is a senior citizen and has no
(a) No, Mr. Jai and Mr. Veeru are not liable to income chargeable under the head
pay advance tax “Profits and gains of business or
(b) Yes, Mr. Jai and Mr. Veeru are liable to pay profession”.
advance tax d) Both (b) and (c) (MTP May 2019)

Answers to Chapter 15

Ans. 1) Option (d) is correct Ans. 3) Option (b) is correct


Since Turnover of Mr. A is more than 2 Cr., he shall not be The benefit of paying advance tax in 1 installment is
eligible for presumptive scheme u/s 44AD. Accordingly, the available only to persons who opt for presumptive scheme
benefit of paying advance tax in 1 installment shall not be u/s 44AD & 44ADA. It is not available for assessees opting
available to him. Sec 44AE

Ans. 2) Option (b) is correct Ans. 4) Option (d) is correct


NAV of House property (₹ 40,000 x 12m) – 4,80,000 Under Sec 192, Employer is required to compute Total tax
Since the property is located in India, liability in hands of employee and then deduct TDS from
rental income shall be deemed to accrue salary income. Accordingly, if entire TDS has been duly
in India and hence taxable in hands of NR deducted and deposited by employer, then Advance tax
Less:- Standard Deduction @30% (1,44,000) liability of Mr. Jha would be Nil
Income u/h House Property 3,36,000
Tax computed on above income 4,470 Ans. 5) Option (a) is correct

 
66 Prepared by : CA Sanchit Grover
  Multiple Choice Questions
  for CA Inter Taxation For May 2023 Exams
Any professional who opts to pay tax u/s 44ADA shall be - In case of Mr. Jai, amount of tax after TDS credit is
liable to pay advance tax only in 1 installment i.e. on or more than ₹ 10,000. However, since he is a resident
before 15th March of PY senior citizen having no income under PGBP head,
he shall not be liable to pay any advance tax
Ans. 6) Option (c) is correct - In case of Mr. Veeru, since the amount payable
Advance tax is to be paid only when Advance tax liability after considering TDS credit is less than ₹ 10,000,
(i.e. after considering TDS & TCS credit) is ₹ 10,000 or more. no advance tax is payable
Further, a resident senior citizen is not required to pay any
advance tax if he doesn’t have any income u/h PGBP. (even Ans. 8) Option (b) is correct
if his tax liability is more than ₹ 10,000) Advance tax is to be paid only when Advance tax liability
(i.e. after considering TDS & TCS credit) is ₹ 10,000 or more.
Ans. 7) Option (a) is correct Further, a resident senior citizen is not required to pay any
Particulars Jai Veeru advance tax if he doesn’t have any income u/h PGBP. (even
Winnings from lotteries 9,00,000 8,00,000 if his tax liability is more than ₹ 10,000)
Tax computed @30% u/s 2,70,000 2,40,000 In the given case, since assessee is not resident, the benefit
115BB of being a senior citizen wont apply.
Add:- HEC 10,800 9,600 NAV of House property (₹ 45,000 x 12m) – 5,40,000
Total Tax after HEC 2,80,800 2,49,600 Since the property is located in India,
Less:- TDS deducted u/s (2,70,000) (2,40,000) rental income shall be deemed to accrue
194B @30% in India and hence taxable in hands of NR
Amount payable after TDS 10,800 9,600 Less:- Standard Deduction @30% (1,62,000)
credit Income u/h House Property 3,78,000
Tax computed on above income 6,660

   

  
Prepared by : CA Sanchit Grover 67
   CHAPTER 16 : FILING OF RETURN OF INCOME

CHAPTER 16 FILING OF RETURN OF INCOME


Q1) In which of the following transactions, quoting (c) ₹ 10,000 under section 234F
of PAN is mandatory by the person entering into (d) Not liable to pay any fee
the said transaction? Q4) Arun’s gross total income of P.Y. 2022-23 is ₹
I Opening a Basic savings bank deposit 2,45,000. He deposits ₹ 45,000 in PPF. He pays
account with a bank electricity bills aggregating to ₹ 1.20 lakhs in
II Applying to a bank for issue of a credit the P.Y.2022-23. Which of the statements is
card. correct?
III Payment of ₹ 40,000 to mutual fund for (a) Arun is not required to file his return of
purchase of its units income u/s 139(1) for P.Y. 2022-23, since
IV Cash deposit with a post office of ₹ his total income before giving effect to
1,00,000 during a day. deduction under section 80C does not
V A fixed deposit of ₹ 30,000 with a NBFC exceed the basic exemption limit.
registered with RBI aggregating the total (b) Arun is not required to file his return of
deposits to ₹ 3,50,000 for the F.Y up to to income u/s 139(1) for P.Y. 2022-23, since
the date of this deposit made. his electricity bills do not exceed ₹
VI Sale of shares of an unlisted company for 2,00,000 for the P.Y.2022-223
an amount of ₹ 60,000 (c) Arun is not required to file his return of
Choose the correct answer: income u/s 139(1) for P.Y. 2022-23, since
(a) II, IV neither his total income before giving
(b) II, III, IV effect to deduction under section 80C
(c) I, II, III, V, VI exceeds the basic exemption limit nor his
(d) II, IV, VI electricity bills exceed ₹ 2 lakh for the P.Y.
Q2) An individual client has consulted you on the 2022-23.
matter of PAN. He is carrying on the business of (d) Arun is required to file his return of
sale & purchase of electronic appliances. His income u/s 139(1) for P.Y. 2022-23, since
turnover is ₹ 3,00,000 and the profit is ₹ 75,000 his electricity bills exceed ₹ 1 lakh for the
for the P.Y. 2022-23. He has asked you to P.Y. 2022-23. (RTP May 2020)
provide him threshold of turnover, if any, Q5) Which of the following returns can be revised
exceeding which he has to apply for PAN. under section 139(5)?
(a) More than ₹ 2,00,000 (i) A return of income filed u/s 139(1)
(b) More than ₹ 2,50,000 (ii) A belated return of income filed u/s
(c) More than ₹ 3,00,000 139(4)
(d) More than ₹ 5,00,000 (iii) A return of loss filed u/s 139(3)
Q3) Mr. Z, a salaried individual, has a total income Choose the correct answer:
of ₹ 8 lakhs for A.Y. 2023-24. He furnishes his (a) Only (i)
return of income for A.Y. 2023-24 on 28th (b) Only (i) and (ii)
August, 2023. He is liable to pay fee of – (c) Only (i) and (iii)
(a) up to ₹ 1,000 under section 234F (d) (i), (ii) and (iii)
(b) ₹ 5,000 under section 234F

 
68 Prepared by : CA Sanchit Grover
  Multiple Choice Questions
  for CA Inter Taxation For May 2023 Exams

Q6) Iskon Inc., a foreign company and non-resident saving A/c and rental income during the
in India for A.Y. 2023-24, engaged in the previous year 2022-23. He incurred expenditure
business of trading of tube-lights outside India. of ₹ 2,00,000 for his son for a study tour to
The principal officer of the company has Europe. Whether he is required to file return of
approached you to enlighten him regarding the income for the assessment year 2023-24? If yes,
provisions of the Income-tax Act, 1961 what is the due date?
pertaining to the person who is required to (a) Yes, 31st July of A.Y
verify the return of income in case of Iskon Inc. (b) Yes, 30th September of A.Y
Advise him as to which of the following (c) Yes, 31st October of A.Y
statements are correct, assuming that the (d) No, he is not required to file return of
company has a managing director income (RTP Nov 2020)
I The return of income in case of Iskon Inc. Q9) Mr. X is a working partner and Mr. Y is a non-
can be verified by the managing director. working partner of XYZ partnership firm. XYZ
II The return of income in case of Iskon Inc. Partnership firm subjected to tax audit under
can be verified by any director, section 44AB for the P.Y. 2022-23. What is the
irrespective of the availability or otherwise due date for filing return of income for Mr. X
of the managing director. and Mr. Y for the A.Y. 2023-24?
III The return of income in case of Iskon Inc. (a) 31st July, 2023 for both Mr. X and Mr. Y
may be verified by a person who holds a (b) 31st October, 2023 for both Mr. X and Mr.
valid power of attorney from such Y
company to do so, irrespective of the (c) 31st July, 2023 for Mr. X and 31st October,
availability or otherwise of the managing 2023 for Mr. Y
director. (d) 31st July, 2023 for Mr. Y and 31st October,
Choose the correct answer: 2023 for Mr. X (RTP Dec 2021)
(a) I or II or III Q10) Mr. Kapoor is a partner in Sai baba Enterprises.
(b) Only I The turnover of the firm for the financial year
(c) I or III 2022-23 amounted to ₹1,96,00,000. The firm
(d) Only III has declared income @8% on presumptive
Q7) Mr. Pawan is engaged in the business of basis under section 44AD of the Act. Apart from
roasting and grinding coffee beans. During F.Y. remuneration, interest and share of profit from
2022-23, his total income is ₹ 4.5 lakhs. Mr. the firm of ₹ 30 lakh, Mr. Kapoor is not having
Pawan filed his return of income for A.Y. 2023- any other source of income. What will be the
24 on 3rd December, 2023. What shall be the due date of filing of return of income by the
fee payable for default in furnishing in return of partnership firm and by Mr. Kapoor for the P.Y.
income for A.Y. 2023-24? 2022-23?
(a) ₹ 5,000 (a) Due date of filing return of income by the
(b) Not exceeding ₹ 1,000 partnership firm shall be 31.10.2023 and
(c) ₹ 10,000 due date of filing return of income by Mr.
(d) No fees payable as total income is below Kapoor shall be 31.10.2023.
₹ 5,00,000 (b) Due date of filing return of income by the
Q8) Mr. Dinesh, a resident in India, has gross total partnership firm shall be 31.12.2023 and
income of ₹ 2,30,000 comprising of interest on
  
Prepared by : CA Sanchit Grover 69
   CHAPTER 16 : FILING OF RETURN OF INCOME
due date of filing return of income by Mr. (c) No deduction under Chapter VI-A under
Kapoor shall be 31.07.2023. the heading Part:B – Deduction in respect
(c) Due date of filing return of income by the of certain payments
partnership firm shall be 31.07.2023 and (d) All of the above (MTP March 2019)
due date of filing return of income by Mr. Q13) Ms. Dilar who is not required to furnish return
Kapoor shall be 31.07.2023. u/s 139(1) as his gross total income itself is less
(d) Due date of filing return of income by the than basic exemption limit, has incurred
partnership firm shall be 31.07.2023 and expenditure of ₹ 2,00,000 for her daughter for
due date of filing return of income by Mr. travel to U.S.A. during P.Y. 2022-23. Is she
Kapoor shall be 31.10.2023. required to file return for A.Y. 2023-24? If yes,
Q11) Mr. Kumar, aged 62 years resident and what is the due date?
ordinarily resident, is a retired employee with a (a) Yes; 31st July, 2023
monthly pension of ₹ 22,000. He has no other (b) Yes; 30th September, 2023
source of income. He has a house property in (c) Yes; 31st August, 2023
Bhatinda and his only son is living in London (d) No, she is not required to file return of
and has a house over there. He met with an income for A.Y. 2023-24
accident and died and thereby leaving the (MTP Oct 2020)
house at London in the name of his father, Mr. Q14) Mr. Alex is a resident but not ordinarily resident
Kumar. Mr. Kumar seeks your advice, as to in India for P.Y. 2022-23. He is doing job in M/s
whether he is required to file his income-tax Kothari Chemicals as Accountant & earns ₹
return u/s 139? 25,000 per month. He had no other income in
(a) Yes, he is mandatorily required to file his India but having a vacant land in Canada which
income-tax return as he is a resident and he had got from his father after his demise. He
ordinarily resident in India and has asset had no income from Canada also. Mr. Alex
located outside India come to you for consulting whether he is
(b) No, he is not required to file return of required to file his return of income for A.Y.
income as his income is below basic 2023-24?
exemption limit (a) Alex is not required to file his return of
(c) Yes, he is required to file his return of income as his total income does not
income as his income exceeds the basic exceed the basic exemption limit
exemption limit (b) Alex is required to file his return of
(d) No, he is not required to file his return of income as he is beneficiary of the assets
income as he is a senior citizen and located outside India
retired employee (MTP Nov 2021) (c) Alex is not required to file his return of
Q12) Which of the following is not a consequence of income as his total income does not
late filing of return? exceed the basic exemption limit and he
(a) Levy of interest under section 234A is resident but not ordinarily resident
(b) Loss (other than loss under the head during the P.Y. 2022-23
“Income from house property”) cannot be (d) Alex is required to file his return of
carried forward income as his total income exceeds the
basic exemption limit

 
70 Prepared by : CA Sanchit Grover
  Multiple Choice Questions
  for CA Inter Taxation For May 2023 Exams

Answers to Chapter 16

Ans. 1) Option (a) is correct Ans. 9) Option (b) is correct


Where firm is subjected to audit u/s 44AB, due date of
Ans. 2) Option (d) is correct filing return of all its partners (whether working or
non-working) shall be 31st October.
Ans. 3) Option (b) is correct
Since Mr. Z has only salary income, his due date to file Ans. 10) Option (c) is correct
ROI would be 31st July. Since return has been filed after Since Firm has opted to pay tax on presumptive
due date but within time permitted u/ 139(4), late fees income computed in accordance with Sec 44AD(1),
of ₹ 5,000 shall be applicable. accordingly no audit requirement and hence due date
shall be 31-07-2023. Further, since the partner also
Ans. 4) Option (d) is correct has no other income except for amounts received from
In the given case, Total income of Mr. Arun before firm, his due date shall also be 31-07-2023
considering Chapter VI-A deductions and exemptions
u/s 54 etc. is not more than ₹ 2,50,000. However, since Ans. 11) Option (a) is correct
electricity bill paid during the year is more than ₹ 1L, In this case, although Gross Total Income of Mr. Kumar
he shall have to mandatorily file his ROI for AY 23-24 is less than exemption limit but he is the beneficial
owner of any asset located outside India. Since he is
Ans. 5) Option (d) is correct ROR, he shall be mandatorily required to file Return of
Return of loss filed u/s 139(3) is also considered to be income u/s 139.
return filed u/s 139(1) and hence it can also be revised
as per Sec 139(5). Ans. 12) Option (c) is correct
Deductions under Part : B of Chapter VI-A are allowed
Ans. 6) Option (c) is correct even if return is filed after due date. However,
As per Sec 140, MD is required to sign on ROI, but Deductions under Part C of Chapter VI-A is allowed
where Company has no MD or MD is not able to sign only when return is filed on or before due date of
due to any reason, in such case any other director of return of income
the Company or any person duly authorized can sign
the return. In case of non- resident Companies, any Ans. 13) Option (d) is correct
person holding valid power of attorney can sign Since GTI is less than exemption limit and further
irrespective of whether MD is available or not. expense on foreign travel is also not more than ₹ 2L,
hence there shall be no requirement of filing return of
Ans. 7) Option (b) is correct income
Since Total income of Mr. Pawan is less than ₹ 5L, late
fees payable u/s 234F shall not exceed ₹ 1,000. Ans. 14) Option (c) is correct
A person who is the beneficiary or beneficial owner of
Ans. 8) Option (d) is correct any asset located outside India is mandatorily required
In the given case, Gross Total Income of Mr. Dinesh is to file return only when such person is Resident &
not more than ₹ 2,50,000 and also the amount of Ordinarily resident. In the given case, since Mr. Alex is
expenditure incurred for travel abroad is not more RNOR and his Total income (before Chapter VI-A
than ₹ 2L, hence there is no mandatory requirement deductions) is less than exemption limit, he will not be
to file ROI u/s 139 covered under requirement to file return of income u/d
139

  
Prepared by : CA Sanchit Grover 71
   CHAPTER 17 : MISCELLANEOUS TOPICS

CHAPTER 17 MISCELLANEOUS TOPICS


Q1) Which of the following statements is not true (d) ₹ 7 lakhs
with respect to the A.Y. 2023-24: Q4) Mr. Bandu, aged 37 years, provides the
(a) No exemption under section 80TTA would following details for P.Y. 2022-23 as follows:
be available to resident senior citizens Particulars ₹ in lakh
(b) Share of profit will not be exempt in the Textile business income 22
hands of partner, if firm claims exemption Speculative business loss (4)
of income under section 10AA Textile business loss b/f from PY (5)
(c) Exemption will be available in respect of 2020-21
STT paid long term capital gains of ₹ Business income of the spouse 2
90,000 on listed equity shares clubbed in hands of Mr. Bandu as
(d) Exemption under section 10(32) on per Sec 64(1)(iv)
income of minor child is allowed for more Deductions available under Chapter 3
than two children also VI-A
Q2) Which of the following benefits are not TDS 1
allowable to Ms. Sakshi, a non-resident, while TCS 0.5
computing her total income and tax liability for Advance Tax paid 1.3
A.Y. 2023-24 under the Income-tax Act, 1961?
(a) Deduction of 30% of gross annual value What shall be the net tax payable/(refundable)
while computing her income from house as per regular provisions of the Income-tax Act,
property in Bangalore 1961 for A.Y. 2023-24 for Mr. Bandu?
(b) Tax rebate of ₹ 4,500 from tax payable on (a) ₹ 24,200
her total income of ₹3,40,000 (b) (₹ 1,00,600)
(c) Deduction for donation made by her to (c) ₹ 2,11,400
Prime Minister’s National Relief Fund (d) ₹ 12,500
(d) Deduction for interest earned by her on Q5) Mr. Raj, aged 32 years, presents you the
NRO savings account. following data for A.Y. 2023-24:
Q3) Mr. Uttam presents you the following data Particulars ₹ in lakh
related to his tax liability for A.Y. 2023-24:- Gross Receipts from business 70
Particulars ₹ in conducted entirely through banking
lakhs channels (opted for section 44AD)
Tax liability as per regular provisions of 15 Capital Gains u/s 112A 5
Income Tax Act 1961 Capital Gains u/s 111A 3
Tax liability as per Sec 115JC 12 Winnings from horse races 1
AMT credit brought forward from AY 5 What would be the tax liability as per the
2022-23 regular provisions of the Income-tax Act, 1961
What shall be the tax liability of Mr. Uttam for of Mr. Raj for the A.Y.2022-23?
A.Y. 2023-24? (a) ₹ 1,28,440
(a) ₹ 12 lakhs (b) ₹ 1,05,560
(b) ₹ 15 lakhs (c) ₹ 1,38,840
(c) ₹ 10 lakhs (d) ₹ 1,45,080

 
72 Prepared by : CA Sanchit Grover
  Multiple Choice Questions
  for CA Inter Taxation For May 2023 Exams

Ans. to Chapter 17

Ans. 1) Option (b) is correct Gross Total Income 19


CBDT circular has clarified that even if firm is not liable to Deductions available under Chapter VI- (3)
pay tax on its income on account of any exemption or A
deduction, still share of profit received by partners from such Total Income 16
firm will be exempt u/s 10(2A). Total tax liability 3,04,200
- TDS credit (1,00,000)
Ans. 2) Option (b) is correct - TCS credit (50,000)
Rebate u/s 87A is allowed only to resident individual. - Advance Tax paid (1,30,000)
However, deduction u/s 24(b), 80G and exemption u/s 10(6)
Net Tax liability 24,200
are available to non-resident as well.

Ans. 5) Option (a) is correct


Ans. 3) Option (a) is correct
Particulars ₹
In the given case, since regular tax is more than amount of
Income u/h PGBP (calculated on 4,20,000
AMT, regular tax shall be payable but after adjusted of B/f
presumptive basis) – 6% of 70L
AMT credit. Amount of AMT credit that can be adjusted shall
Capital Gain u/s 112A 5,00,000
be lower of following:-
Capital Gain u/s 111A 3,00,000
a) Total AMT credit B/f = ₹ 5L
Casual Income 1,00,000
b) Regular Tax – AMT = ₹ 3L
Gross Total Income 13,20,000
c) Hence Tax payable = Regular tax – AMT credit adjusted
Tax on normal income of ₹ 4,20,000 (Nil 8,500
= ₹ 15L – ₹ 3L = ₹ 12L
+ 8,500)

Ans. 4) Option (a) is correct Tax on LTCG u/s 112A (₹ 4L x 10%) 40,000
Particulars ₹ in lakh Tax on LTCG u/s 112A (₹ 3L x 15%) 45,000

Textile business income 22 Tax on casual income (₹ 1L x 30%) 30,000

Textile business loss b/f from PY 2020- (5) Total tax before HEC 1,23,500
21 HEC @4% 4,940
Business income of the spouse clubbed 2 Total Tax after HEC 1,28,440
in hands of Mr. Bandu as per Sec
64(1)(iv)

   

  
Prepared by : CA Sanchit Grover 73
   CHAPTER 18 : CASE SCENARIO BASED MCQS

CHAPTER 18 CASE SCENARIO BASED MCQS


Case Study No. 1 :- Residential Status + Clubbing + Sec 115BAC
Mr. Shashikant, aged 45 years, is an Indian citizen and a member of the crew of a Singapore bound Indian ship
engaged in carriage of passengers in international traffic departing from Chennai port on 29th May, 2022
Particulars Date
Date entered into the Continuous Discharge Certificate in respect of joining the ship 29 May 2022
th

by Mr. Shashikant
Date entered into the Continuous Discharge Certificate in respect of signing off the 19th December 2022
ship by Mr. Shashikant

He stayed in India in the last 4 previous years preceding the P.Y. 2022-23 for 400 days and for a period of 750 days
in the last 7 previous years preceding to P.Y. 2022-23. He received salary of ₹ 7,20,000 in his NRE account maintained
with State Bank of India, Chennai Branch. He also furnished details of other income earned by him during the
previous year 2022-23:
S No. Particulars Amount
1. Dividend declared in the month of April, 2022 by X limited, a Singapore company. 1,00,000
The same was received by him in Singapore
2. Agriculture income from land in Pakistan received in India 2,50,000
3. Rent received from house property in Chennai 3,60,000

Based on the facts of the case scenario given above, choose the most appropriate answer to the following
questions:

Q1) What is Mr. Shashikant’s residential status for (d) ₹ 6,02,000


the P.Y 2022-23? Solution) Option (c) is correct.
(a) Resident and ordinarily resident Salary received in NRE account Nil
(b) Resident but not ordinarily resident (Income by way of salary received by
(c) Non-resident non-resident seafarers, for services
(d) Deemed resident outside India on a foreign bound ship
Solution) Option (c) is correct. and received into NRE bank account
No. of Days stay in India = 365 days – (29th May to 19th maintained with Indian bank shall not
December) = 160 days taxable as per CBDT circular)
Since Shashikant left India as a crew member of Dividend from foreign company Nil
Indian ship, 2nd basic condition shall not be applicable. (accrued as well as received outside
Since 1st Basic condition is not getting satisfied, he India and hence not taxable in hands
shall be a non-resident of non-resident)
Q2) What would be the total income of Mr. Agricultural income from a land 2,50,000
Shashikant for A.Y. 2023-24 assume that he located outside India but received in
does not opt to pay tax under section 115BAC? India (Exemption u/s 10(1) shall not
(a) ₹ 7,10,000 apply)
(b) ₹ 11,72,000 Rent received from house property in 3,60,000
(c) ₹ 5,02,000 Chennai (deemed to accrue in India)
 
74 Prepared by : CA Sanchit Grover
  Multiple Choice Questions
  for CA Inter Taxation For May 2023 Exams

Less:- Standard deduction u/s 24(a) (1,08,000) (c) ₹ 23,610


Total Income 5,02,000 (d) ₹ 34,840
Solution) Option (b) is correct.
Q3) Assume for the purpose of answering this
Tax liability if Sec 115BAC not opted on Total
question that Mr. Shashikant has transferred his
Income of ₹ 5,02,000
house property in Chennai to his minor married
Tax on first ₹ 2,50,000 Nil
daughter on 1st April, 2022 and his wife is a
Tax on next ₹ 2,50,000 @5% 12,500
housewife and does not have any income. The
minor married daughter receives the rent from Tax on remaining ₹ 2,000 @20% 400
house property. In such case, his total income Tax before HEC 12,900
would be– HEC @4% 516
(a) ₹ 5,00,500 Tax after HEC 13,416
(b) ₹ 6,00,500 Rounded off 13,420
(c) ₹ 5,02,000 Tax liability if Sec 115BAC opted on Total Income of
(d) ₹ 6,02,000 ₹ 5,02,000 (No change in income since none of the
Solution) Option (a) is correct. restricted deductions has been allowed in the
Since the property has been transferred to minor present case)
married daughter, Sec 27 shall not be applicable. Tax on first ₹ 2,50,000 Nil
However, clubbing provisions u/s 64(1A) shall be Tax on next ₹ 2,50,000 @5% 12,500
applicable and Income u/s HP of ₹ 2,52,000 shall be Tax on remaining ₹ 2,000 @10% 200
clubbed in hands of Mr. Shashikant but exemption of
Tax before HEC 12,700
₹ 1,500 u/s 10(32) shall be available.
HEC @4% 508
Q4) Mr. Shashikant would like to minimize his tax
Tax after HEC 13,208
liability and consulted you to compute the
Rounded off 13,210
amount of same for the P.Y. 2022-23.
Accordingly, his tax liability (rounded off) would Hence in order to minimize tax liability, he should opt
be – for Sec 115BAC and in such Case tax liability would
(a) ₹ 13,420 be ₹ . 13,210
(b) ₹ 13,210

Case Study No. 2 :- Exemptions + Income u/h House Property + Deductions under Chapter VI-
A + Income u/h Other Sources
Mr. Suraj (aged 48 years) furnishes the following particulars for the previous year 2022-23 in respect of an industrial
undertaking established in "Special Economic Zone" in March 2015. It began manufacturing in April 2016
Particulars ₹
Total Sales 85,00,000
Export Sales (proceeds received in India) 45,00,000
Domestic Sales 40,00,000
Profit from the above undertaking 20,00,000

Export Sales of F.Y. of 2022-23 include freight and insurance of ₹ 5 lacs for delivery of goods outside India. He
received rent of ₹ 25,000 per month for a commercial property let out to Mr. Sudhir, a salaried individual. He earned
interest on savings bank A/c of ₹ 12,500 and interest on Post Office savings A/c of ₹ 5,500 during the P.Y. 2022-23.
  
Prepared by : CA Sanchit Grover 75
   CHAPTER 18 : CASE SCENARIO BASED MCQS
Based on the facts of the case scenario given above, choose the most appropriate answer to the following
questions:

Q1) Compute the amount of export turnover and (a) ₹ 10,00,000


total turnover for purpose of computing (b) ₹ 9,41,154
deduction under section 10AA for A.Y. 2023-24. (c) ₹ 11,25,000
(a) ₹ 45,00,000 and ₹ 85,00,000, respectively (d) ₹ 5,00,000
(b) ₹ 40,00,000 and ₹ 80,00,000, respectively Solution) Option (a) is correct.
(c) ₹ 45,00,000 and ₹ 80,00,000, respectively In case SEZ unit operations were started in PY 2019-
(d) ₹ 40,00,000 and ₹ 85,00,000, respectively 20, PY 22-23 would be 4th year of operation and hence
Solution) Option (b) is correct. 100% of export profits would be eligible for Sec 10AA
Sec 10AA clarifies that export turnover for the purpose deduction.
of calculating Sec 10AA deduction shall not include Q4) Compute the total income of Mr. Suraj for the
any freight & insurance for delivery of goods outside previous year 2022-23, assuming that he does
India. Hence Export Turnover = ₹ 45L – ₹ 5L = ₹, 40L not opt to pay tax under section 115BAC.
Total Turnover + ₹ 40L + ₹ 40L = ₹ 80L (a) ₹ 12,14,500
Q2) Compute the amount of deduction available (b) ₹ 17,18,000
under section 10AA to Mr. Suraj under section (c) ₹ 17,14,500
10AA for A.Y. 2023-24. (d) ₹ 17,28,000
(a) ₹ 10,00,000 Solution) Option (c) is correct
(b) ₹ 4,70,577
Income u/h House property 2,10,000
(c) ₹ 5,62,500
(₹ 3,00,000 – 30%)
(d) ₹ 5,00,000
Income u/h PGBP (₹ 20,00,000 – ₹ 15,00,000
Solution) Option (d) is correct.
5,00,000)
Export profits of SEZ unit = ₹ 20L x 40L/80L = ₹
Interest on saving Bank A/c 12,500
10,00,000. Since PY 22-23 is 8th year of operation of
Interest on PO saving A/c (₹ 5,500 – ₹ 2,000
SEZ unit, deduction u/s 10AA shall be restricted to
3,500)
50% of export profits.
Gross Total Income 17,24,500
Q3) Assume for the purpose of this question only
Less:- Deduction u/s 80TTA (10,000)
that Mr. Suraj established SEZ Unit and began
Total Income 17,14,500
manufacturing in April, 2019. Compute the
amount of deduction available under section
10AA for A.Y. 2023-24.

Case Study No. 3 :- Agricultural Income + Income under the head ‘House Property’
Mr. Krishna is engaged in the following activities on agricultural land situated in India, total area of land is 5 acres.
Activity A: He grows saplings or seedlings in a nursery spreading over on one acre land, the sale proceeds of which
is ₹ 5,00,000. Cost of plantation is ₹ 1,40,000. Basic operations are not performed for growing saplings or seedlings
Activity B: He grows cotton on 3 acres land. 40% of cotton produce is sold for ₹ 4,00,000, the cost of cultivation of
which is ₹ 2,25,000. The cost of cultivation of balance 60% cotton is ₹ 3,37,500 and the market value of the same is

 
76 Prepared by : CA Sanchit Grover
  Multiple Choice Questions
  for CA Inter Taxation For May 2023 Exams

₹ 6,00,000, which is used for the purpose of manufacturing yarn. After incurring manufacturing expenses of ₹
1,00,000, yarn is sold for ₹ 8,50,000
Activity C: Land measuring 1 acres is let out to Mr. Ramesh on monthly rental of ₹ 15,000 which is used by Mr.
Ramesh as follows:
- 50% of land is used for agricultural purpose
- 50% of land is used for non-agricultural purpose.
Based on the facts of the case scenario given above, choose the most appropriate answer to the following
questions:

Q1) What amount of income arising from activity A (b) ₹ 2,62,500 as agricultural income and ₹
would constitute agricultural income in the 1,50,000 as business income
hands of Mr. Krishna? (c) ₹ 3,37,500 as agricultural income and ₹
(a) ₹ 5,00,000 1,50,000 as business income
(b) Nil (d) ₹ 4,12,500 as business income
(c) ₹ 3,60,000 Solution) Option (b) is correct
(d) ₹ 1,40,000 In respect of 60% cotton that has been used for
Solution) Option (c) is correct manufacturing yarn (that is ultimately sold), it
As per Sec 2(1A) income from growing seedlings & constitutes composite business and hence agricultural
saplings in a nursery shall be treated as agricultural income would have to be computed as under:-
income even if basic operations are not performed. In Market Value of 60% Cotton used for 6,00,000
the given case, Agricultural income = ₹ 5,00,000 – ₹ manufacturing Yarn
1,40,000 = ₹ 3,60,000 Less:- Cost of cultivation of this 60% (3,37,500)
Q2) What amount of income from activity B with Cotton
respect to sale of cotton would constitute Amt. to be treated as Agricultural 2,62,500
agricultural income or/and business income in income
the hands of Mr. Krishna?
Q4) What amount of income arising from activity C
(a) ₹ 1,75,000 as agricultural income
constitute agricultural income or otherwise in
(b) ₹ 1,75,000 as business income
the hands of Mr. Krishna?
(c) ₹ 1,75,000 as agricultural income and ₹
(a) Whole amount of ₹ 1,80,000 would be
2,62,500 as business income
agricultural income
(d) ₹ 4,00,000 as agricultural income
(b) Whole amount of ₹ 1,80,000 would be
Solution) Option (a) is correct
business income.
In respect of 40% cotton that is sold, entire profit
(c) ₹ 90,000 would be agricultural income
would constitute agricultural income since no
and ₹ 63,000 is chargeable to tax as
subsequent operations have been performed. Hence
income from house property
Agricultural income = ₹ 4,00,000 – ₹ 2,25,000 = ₹
(d) ₹ 90,000 would be agricultural income
1,75,000
and ₹ 90,000 is chargeable to tax under
Q3) What amount of the income from activity B with
the head “Income from Other Sources”
respect to sale of yarn constitute agricultural
Solution) Option (d) is correct
income or/and business income in the hands of
Since 50% of Land is used for agricultural operations,
Mr. Krishna?
Hence Rent of ₹ 7,500 x 12 = ₹ 90,000 would be
(a) ₹ 1,50,000 as agricultural income
  
Prepared by : CA Sanchit Grover 77
   CHAPTER 18 : CASE SCENARIO BASED MCQS
treated as Agricultural income u/s 2(1A). Further, rent (b) ₹ 3,30,000
related to remaining 50% of land used for non- (c) ₹ 5,02,500
agricultural operations would be taxable under the (d) ₹ 2,13,000
head Other Sources (Since vacant land doesn’t Solution) Option (a) is correct
constitute House property, this rental income cant be Non Agricultural income shall be computed as under:-
taxable under Income from HP head. Further standard Business Income from sale of Yarn 1,50,000
deduction of 30% is not available under Other Sources (8,50,000 – 1,00,000 – 6,00,000)
head) Income u/h Other Sources 90,000
Q5) Compute the gross total income of Mr. Krishna (Rent from 50% of land being used
for the P.Y. 2022-23, assuming he has no other for non-agricultural operations)
source of income. Gross Total Income 2,40,000
(a) ₹ 2,40,000

Case Study No. 4 :- Residential Status + Income under the head Salary + Computation of Tax
Liability
Mr. Rajesh Sharma, aged 54 years, an Indian citizen, is working as Assistant Manager in ABC India Ltd. He is getting
basic salary of ₹ 58,000 per month. He used to travel frequently out of India for his office work. He left India from
Delhi Airport on 5th October, 2022 and returned to India on 2nd April, 2023.
For previous year 2022-23, following information are relevant;
(a) Dearness Allowance - 10% of Basic Pay (considered for retirement purposes)
(b) Bonus - ₹ 98,000
(c) Medical allowance paid during P.Y. 2022-23 amounting to ₹ 60,000
(d) He was also reimbursed medical bill of his mother amounting to ₹ 15,000.
(e) He was also transferred a laptop by company for ₹ 15,000 on 31st December, 2022. The laptop was acquired
by company on 1st October, 2019 for ₹ 1,00,000. Company was charging depreciation at 31.666% assuming
useful life of laptop as 3 years.
(f) He was also reimbursed salary of house servant of ₹ 4,000 per month.
(g) Professional Tax paid by employer amounting to ₹ 2,400.
(h) 400 equity shares allotted by ABC India Ltd. at the rate of ₹ 250 per share against fair market value of share
of ₹ 350 on the date of exercise of option.
(i) Short-term capital gain on sale of shares of listed company on which STT is paid amounting to ₹ 94,000.
(j) Mr. Rajesh does not opt for the provisions of section 115BAC. (MTP Oct 2020)
Based on the facts of the case scenario given above, choose the most appropriate answer to the following
questions:

Q1) What is Mr. Rajesh Sharma’s residential status (b) Non-Resident


for the A.Y. 2023-24? (c) Resident but not ordinarily resident
(a) Resident but can’t determine resident and (d) Resident and ordinarily resident
ordinarily resident or resident but not Solution) Option (a) is correct
ordinarily resident from the given No of Days Stay in India during PY 22-23 = 188 days.
information Since 1st Basic condition is being satisfied here, he
 
78 Prepared by : CA Sanchit Grover
  Multiple Choice Questions
  for CA Inter Taxation For May 2023 Exams

shall be Resident. However, no information has been Taxable Value of Perquisites 1,05,400
provided regarding preceding 7 PYs or 10 PYs and Gross Salary 10,29,000
hence in absence of such information, we cannot Less:- Standard Deduction (50,000)
comment whether he is ROR or RNOR. Less:- Professional tax (2,400)
Q2) What are his taxable perquisites for A.Y. 2023- Income under the head Salary 9,76,600
24?
Q4) The total tax liability of Mr. Rajesh Sharma for
(a) ₹ 55,000
A.Y. 2023-24 is:
(b) ₹ 90,400
(a) ₹ 1,26,800
(c) ₹ 1,05,400
(b) ₹ 1,40,710
(d) ₹ 1,03,000
(c) ₹ 1,12,130
Solution) Option (C) is correct
(d) ₹ 1,39,960
Calculation of taxable Perquisites
Solution) Option (a) is correct
Reimbursement of medical bill of 15,000
Income under the head Salary 9,76,600
ailing mother (Fully taxable)
STCG u/s 111A 94,000
Perquisite value of laptop transferred Nil
Gross Total Income 10,70,600
(1,00,000 – 87,500 – 15,000)
Calculation of Tax Liability
(For the purpose of calculating
Tax computed on Normal Income 1,07,820
perquisite value, 50% depreciation on
(Nil + 12,500 + 95,320)
WDV basis shall be computed for 3
Tax computed on Special Income 14,100
completed years of use by the
(94,000 x 15%)
employer)
Tax before HEC 1,21,920
Reimbursement of Salary of house 48,000
servant (₹ 4,000 x 12m) Add:- HEC @4% 4,876.80

Professional tax of employee paid by 2,400 Total tax liability 1,26,796.80


employer Tax liability (after round-off) 1,26,800
Perquisite Value of ESOPs 40,000 Q5) Assume for the purpose of this question only,
[400 shares x (₹ 350 – ₹ 250) that Mr. Rajesh was found owner of ₹ 5 lakh
Taxable Value of Perquisites 1,05,400 worth jewellery acquired in F.Y. 2022-23, of
Q3) What is the income chargeable under the head which he could not provide any satisfactory
“Salaries” in the hands of Mr. Rajesh Sharma for explanation about source of income. What
A.Y. 2023-24? would be the tax liability (without considering
(a) ₹ 9,76,600 surcharge and Health and education cess, if
(b) ₹ 9,86,600 any) of Mr. Rajesh Sharma towards such

(c) ₹ 9,71,600 unexplained expenditure:

(d) ₹ 9,61,600 (a) ₹ 1,00,000


Solution) Option (a) is correct (b) ₹ 1,50,000
Calculation of taxable Perquisites (c) ₹ 3,00,000
Basic Salary (₹ 58,000 x 12m) 6,96,000 (d) ₹ 3,90,000
Solution) Option (c) is correct
Dearness Allowance 69,600
In respect of undisclosed & unexplained assets,
Bonus 98,000
income is subjected to tax u/s 115BBE in which tax
Medical Allowance 60,000
  
Prepared by : CA Sanchit Grover 79
   CHAPTER 18 : CASE SCENARIO BASED MCQS
rate is 60%. Hence, Tax liability before surcharge = ₹
5,00,000 x 60% = ₹ 3,00,000

Case Study No. 5 :- Income under the head Salary + Deductions under Chapter VI-A + Sec
11BAC
Mr. Hardik (age 45 years) is appointed as senior executive officer in Sky India Limited, Mumbai on 01.02.2022 in the
scale of ₹ 35,000-3500-65,000. He is paid dearness allowance @ 40% of salary forming part of retirement benefits.
He is given rent free unfurnished accommodation on 01.5.2022 which he occupied only from 01.10.2022. The
company pays lease rent of ₹ 5,000 p.m.
He has been provided a car of above 1.6 liters capacity which is used by him for private purposes only. The actual
cost of the car is ₹ 8,00,000. The monthly expenditure of car is ₹ 5,000, which is fully met by the employer. Car is
owned by his employer.
He pays lumpsum premium of ₹ 1,20,000 towards health insurance for self and his wife (age 43 years) for 48 months
on 01.10.2022 by account payee cheque. He also contributes ₹ 1,50,000 towards PPF.
Mr. Hardik is interested to opt for concessional tax regime available under section 115BAC. (RTP May 2020)
Based on the facts of the case scenario given above, choose the most appropriate answer to the following
questions:

Q1) What would be the value of rent-free (a) ₹ 30,000


accommodation chargeable to tax in the hands (b) ₹ 15,000
of Mr. Hardik? (c) ₹ 24,000
(a) ₹ 44,835 (d) ₹ 25,000
(b) ₹ 44,100 Solution) Option (c) is correct
(c) ₹ 45,570 In case of medical insurance premium that is paid for
(d) ₹ 30,000 more than 1 year, amount eligible for deduction u/s
Solution) Option (d) is correct 80D shall be Premium divided by no. of years starting
Value of Rent Free accommodation shall be lower of with PY in which premium is paid and ending with PY
the following two:- up to which policy is effective.
Lease charges paid or payable 30,000 Here, Policy starts in PY 22-23 and ends in PY 26-27.
(₹ 5,000 x 6 m) Hence Eligible amount u/s 80D = ₹ 1,20,000/5years =
15% of RFA Salary 45,570 ₹ 24,000 (within the max. limit of ₹, 25,000)
15% of [(35,000 x 4 + 38,500 x 2) + Q3) What would be perquisite value of car
40% ] chargeable to tax in the hands of Mr. Hardik?
Value of RFA 30,000 (a) ₹ 28,800
(b) ₹ 21,600
Q2) What amount of health insurance premium
(c) ₹ 60,000
paid during the previous year 2022-23 by Mr.
(d) ₹ 1,40,000
Hardik can be claimed as deduction while
Solution) Option (d) is correct
computing total income, if he does not opt to
Since car is being used only for private purposes,
pay tax under section 115BAC?
perquisite value shall be computed as under:-

 
80 Prepared by : CA Sanchit Grover
  Multiple Choice Questions
  for CA Inter Taxation For May 2023 Exams

Expenditure incurred by employer 1,40,000 Calculation of Total income as per Normal


(₹ 60,000 + 10% of ₹ 8,00,000) provisions
Less:- Amt. recovered from employee Nil Basic Salary [35,000 x 10 + 38,500 x 2) 4,27,000
Taxable value of perquisite 1,40,000 Dearness Allowance (40% of 4,27,000) 1,70,800
Value of unfurnished RFA 30,000
Q4) Would you advise to Mr. Hardik to opt to pay tax
Taxable Value of Motor car 1,40,000
under section 115BAC?
Gross Salary 7,67,800
(a) Yes, Mr. Hardik can opt for section
Less:- Standard Deduction (50,000)
115BAC, since in such case his tax
liability would be ₹ 22,760, being lower Gross Total Income 7,17,800
than the tax liability under normal Less:- Deduction u/s 80C (1,50,000)
provisions of the Act Less:- Deduction u/s 80D (24,000)
(b) Yes, Mr. Hardik can opt for concessional Total Income 5,43,800
tax regime, since in such case his tax Tax liability computed as per normal 21,260
liability would be ₹ 17,560 being lower provisions
than the tax liability under normal (Nil + 12,500 + 8,760)
provisions of the Act. Add:- HEC @4% 850.40
(c) No, Mr. Hardik should not opt, since as Total Tax liability 22,110.40
per normal provisions of the Act, his tax Final Tax liability (rounded off) 22,110
liability would be ₹ 32,510, being lower
than the tax liability under section Calculation of Total income as per Sec 115BAC
115BAC Total Income as per normal provisions 5,43,800
(d) No, Mr. Hardik should not opt, since as Add:- Deduction u/s 16 not allowed 50,000
per normal provisions of the Act, his tax
Add:- Deduction u/s 80C not allowed 1,50,000
liability would be ₹ 22,110, being lower
Add:- Deduction u/s 80D not allowed 24,000
than the tax liability under section
Total Income as per Sec 115BAC 7,67,800
115BAC
Tax liability computed as per rates 40,170
Solution) Option (d) is correct
given u/s 115BAC
(Nil + 12,500 + 25,000 + 2,670)
Add:- HEC @4% 1,606.80
Tax liability u/s 115BAC 41,776.80
Final Tax liability after rounding off 41,780

Case Study No. 6 :- Residential Status + Income under the head House Property
Ananya Gupta, a citizen of India, lives with her family in New York since the year 2000. She visited India from 21st
March, 2022 to 28th September, 2022 to take care of her ailing mother. In the last four years, she has been visiting
India for 100 days every year to be with her mother. She owns an apartment at New York, which is used as her
residence. The expected rent of the house is $ 32,000 p.a. The value of one USD ($) may be taken as ₹ 75. Municipal
taxes paid in New York in January, 2023 are $ 2,000.

  
Prepared by : CA Sanchit Grover 81
   CHAPTER 18 : CASE SCENARIO BASED MCQS
She took ownership and possession of her house in New Delhi on 25th March, 2022, for self-occupation, while she
is in India. The municipal valuation is ₹ 4,20,000 p.a. and the fair rent is ₹ 4,50,000 p.a. She paid property tax of ₹
22,000 to Delhi Municipal Corporation on 21st March, 2023. She had taken a loan of ₹ 16 lakhs @ 10% p.a. from
IDBI Bank on 1st April, 2018 for constructing this house and the construction got completed on 20th March, 2022.
No amount has been paid towards principal repayment so far. The house is vacant for the rest of the year i.e., from
October 2022 to March 2023.
She had a house property in Mumbai, which was sold on 28th March, 2022. In respect of this house, she received
arrears of rent of ₹ 3,00,000 on 4th February, 2023. This amount has not been charged to tax earlier. She does not
have any income under any other source in India during previous year in 2022-23.
Ananya Gupta does not want to opt for the new tax regime under section 115BAC for A.Y. 2023-24.
Based on the facts of the case scenario given above, choose the most appropriate answer to the following
questions :

Q1) What would be the residential status of Ananya u/s 23(2) is available in respect of two
Gupta for A.Y. 2023-24? house properties.
(a) Resident and ordinarily resident Solution) Option (a) is correct
(b) Resident but not ordinarily resident Since she is a non-resident, annual value of apartment
(c) Deemed resident but not ordinarily in New York is not chargeable to tax in her hands.
resident in India Hence, she would be claiming benefit of Nil NAV only
(d) Non-resident in respect of New Delhi house which is self-occupied
Solution) Option (d) is correct by her throughout the PY (even during Oct 2022 to
No. of days stay during PY 22-23 = 1 April 2022 to
st
March 23, house was in her constructive possession
28 Sep 2022 = 181 days
th
since it was not being used for any other purposes)
Aggregate stay during 4 preceding PYs = 400 Q3) What is the income chargeable under the head
In the given question, it is clear that Total Income “Income from house property” of Ananya Gupta
(excluding Income from foreign sources) < 15L, hence for A.Y. 2023-24?
2 nd
Basic condition is not applicable here. Further, (a) ₹ 15,65,000
since 1 Basic condition is not getting satisfied, we can
st
(b) ₹ 3,09,600
conclude that she is Non-resident (c) ₹ 1,00,000
Q2) Ms. Ananya Gupta can claim benefit of “Nil” (d) ₹ 10,000
Annual Value under section 23(2) in respect of Solution) Option (d) is correct
- NAV of Self Occupied property in Delhi Nil
(a) Her Delhi house Less:- Deduction in respect of Interest (2,00,000)
(b) Her New York house, since it is more on Borrowed capital
beneficial; her Delhi house will be (₹ 1,60,000 + 3,20,000 /5) subject to
deemed to be let out and expected rent maximum of ₹ 2,00,000
would be the annual value. Loss from Delhi HP (2,00,000)
(c) Her Delhi house alone; her New York Arrears of Rent taxable u/s 25A (after 2,10,000
house will be deemed to be let out and considering deduction of 30%)
expected rent would be the annual value. Income u/h House Property 10,000
(d) Both her Delhi house and New York
house, since benefit of Nil Annual value
 
82 Prepared by : CA Sanchit Grover
  Multiple Choice Questions
  for CA Inter Taxation For May 2023 Exams

Q4) Assuming that, for the purpose of this question (c) ₹ 17,95,000
alone, Ananya Gupta has let out her flat in New (d) ₹ 18,85,000
York during the six months (April to September) Solution) Option (a) is correct
when she is in India, for a sum of $ 6,000 p.m. Since rental income is with respect of property in New
Such rent was received in a bank account in York, it will be considered to have been accrued
New York and then remitted to India through outside India. Further, since it is also received outside
approved banking channels. What would be the India (i.e. first occasion of receipt), it becomes a foreign
income from house property chargeable to tax income which is not taxable in hands of non-resident.
in her hands in India for A.Y. 2023-24? Hence, even in this case, there shall be no change in
(a) ₹ 10,000 the Total Income of Ms. Ananya
(b) ₹ 17,85,000

Case Study No. 7 :- Income under House Property + Income under PGBP + Taxation of Gifts
Ram Builders & Developers is the sole-proprietorship concern of Mr. Ram. The main business of the concern is
construction, development and sale of residential and commercial units. Ram Builders & Developers developed a
project named Luxuria Heaven, which has both residential and commercial units with its own funds. It obtained
certificate of completion for the said project with effect from 31/03/2022. Ram sold majority of its residential units
and commercial units in the F.Y. 2022-23. However, around 30 residential units and 15 commercial units were held
by him as stock in trade as on 31.3.2023. During this period, there was a slump in the real estate sector. In order to
earn some income from these units, Ram incidentally let out some of the units held as stock-in-trade. The details of
units constructed, sold and held as stock-in-trade are given hereunder:

Particulars Total Units Units sold Units held Units let out Units vacant Actual rent per
constructed as Stock in during PY during the unit per month [in
Trade as on 2022-23 out whole of PY respect of let out
31-03-2023 of (4) 2022-23 units mentioned
[(2) – (3)] [(4) – (5)] in (5)]
(1) (2) (3) (4) (5) (6) (7)
Residential 100 70 30 10 20 10,000 p.m.
Units
Commercial 40 25 15 5 10 18,000 p.m.
Units
140 95 45 15 30

Out of the residential units sold, 5 residential units were sold to his friend, Mr. Gaurav, who is also a real estate
developer, on 15.2.2023, for ₹ 20 lakhs each. The stamp duty value on the date of sale was ₹ 23 lakhs each. However,
the agreement of sale was entered into on 1.11.2022, on which the date the stamp duty value was ₹ 22 lakhs. Mr.
Ram received ₹ 1 lakh by way of account payee bank draft on 1.11.2022 from Mr. Gaurav.
Based on the facts of the case scenario given above, choose the most appropriate answer to the following
questions:

  
Prepared by : CA Sanchit Grover 83
   CHAPTER 18 : CASE SCENARIO BASED MCQS
Q1) While computing the total income of Mr. Ram, Solution) Option (C) is correct
the income from residential and commercial In case of House Property held as stock in trade
units let out during the P.Y. 2022-23 will be (whether residential or commercial), no annual value
taxed under head: shall be chargeable to tax within a period of 2 years
(a) Income from house property from end of FY in which completion certificate is
(b) Profits and gains of business or obtained, provided such house property is lying vacant
profession and not been actually let out.
(c) Income from let out residential units will Q3) What would be the full value of consideration
be taxed under the head “Income from in respect of sale of units to Mr. Gaurav for the
house property” and income from let out purpose of computing profits and gains from
commercial units will be taxed under the transfer of units?
head “Profits and gains of business or (a) ₹ 1,00,00,000
profession” (b) ₹ 1,15,00,000
(d) Income from other source (c) ₹ 1,10,00,000
Solution) Option (a) is correct (d) ₹ 99,00,000
In case House Property is held as Stock in trade, Solution) Option (A) is correct
annual value of such house property is chargeable to In the given case, SDV to be considered for the
tax under the head ‘Income from House Property’ purposes of applying Sec 43CA shall be ₹ 22L i.e. SDV
Q2) What would be the tax treatment of vacant as on date of agreement. Since SDV is not more than
residential and commercial units held as stock 110% of Sale consideration, Sec 43CA shall not be
in trade as on 31.3.2023? applicable and accordingly Full Value of consideration
(a) The vacant residential units would be for purposes of computing PGBP income shall be ₹
deemed to be let out and expected rent 20L x 5 units = ₹ 1,00,00,000
would be deemed as the annual value Q4) Assume that ₹ 1 lakh was paid in cash by Mr.
chargeable to tax under the head “Income Gaurav to Mr. Ram on 1.11.2022 instead of by
from house property” for A.Y. 2023-24. way of account payee bank draft, what would
(b) The vacant units, both residential and be the income chargeable under section
commercial, would be deemed to be let 56(2)(x) in the hands of Mr. Gaurav?
out and expected rent would be deemed (a) ₹ 15 lakh
as the annual value chargeable to tax (b) ₹ 10 lakh
under the head “Income from house (c) Nil, since the stamp duty value is within
property” for A.Y. 2023-24. the permissible deviation limit
(c) The annual value of both vacant (d) Nil, since section 56(2)(x) is not
residential and commercial units would applicable in this case
be Nil for A.Y. 2023-24. Hence, no income Solution) Option (D) is correct
is chargeable for such units under the In the given case, property is a stock in trade in hands
head “Income from house property” for of Mr. Gaurav and hence provisions of Sec 56(2)(x)
A.Y. 2022-23. shall not be applicable in his case (This is irrespective
(d) Vacant units held as stock-in-trade can of the fact that difference between SDV of 23L and
never be deemed as let out at any point Actual purchase price of 20L is more than 10% of
of time Purchase price)

 
84 Prepared by : CA Sanchit Grover
  Multiple Choice Questions
  for CA Inter Taxation For May 2023 Exams

Case Study No. 8 :- Income under House Property + Other Sources


For the assessment year 2023-24, Mr. Sonu submits the following information
Particulars Building at Chennai Building at Kochi
Municipal Valuation 35,000 80,000
Standard Rent 36,000 70,000
Fair Rent 31,000 82,000
Rent received 38,000 68,000
Municipal taxes paid by tenant Mr. Ramu for building at 3,000 4,000
Chennai and paid by Mr. Sonu for Building at Kochi
Repairs paid by tenant Mr. Ramu for Chennai building and 500 18,000
Mr. Sonu paid for Kochi building
Land revenue paid 2,000 16,000
Insurance premium paid 500 2,000
Interest on loan borrowed for payment of municipal tax of 200 400
house property
Nature of occupation Let out for residence Let out for business
Date of completion of construction 01-04-1996 01-07-2008

Mr. Sonu is constructing one more building in Mumbai during the previous year 2022-23. Mr. Raju, a film director,
took on rent the building under construction in Mumbai at ₹ 5,000 per month for his film shooting. The construction
of the said building would be completed by April 2023. Mr. Sonu is a real estate developer and letting out properties
is not the business of Mr. Sonu.
Based on the facts of the case scenario given above, choose the most appropriate answer to the following
questions:

Q1) Which of the building’s income is chargeable to Q2) Which of the following payments/expenditure is
tax under the head “Income from house allowable as deduction while computing
property” in the hands of Mr. Sonu? income under the head “Income from house
(a) Building at Chennai only property” incurred in respect of the building at
(b) Building at Kochi only Chennai and Kochi?
(c) Both buildings at Chennai and Kochi (a) Municipal taxes paid by Mr. Sonu and Mr.
(d) All the three buildings at Chennai, Kochi Ramu
and Mumbai (b) Municipal tax, land revenue, insurance
Solution) Option (C) is correct premium, interest on loan borrowed for
In the given case, property is a stock in trade in hands payment of Municipal tax paid by Mr.
of Mr. Sonu (since he is a real estate developer) and Sonu
hence income from both buildings in Chennai & Kochi (c) Only municipal tax paid by Mr. Sonu
shall be chargeable to tax in his hands. Further, since (d) Both Municipal tax and repairs paid by
Property in Mumbai is still under construction and not Mr. Sonu
yet complete, it would not be chargeable to tax in PY Solution) Option (C) is correct
22-23 Deduction for Municipal taxes is allowed from Gross
annual value only when such taxes have been paid by
  
Prepared by : CA Sanchit Grover 85
   CHAPTER 18 : CASE SCENARIO BASED MCQS
the owner of the property (& not tenant). Further, apart Q4) What is the amount chargeable to tax under the
from municipal taxes and Interest on borrowed capital, head “Income from house property” in the
no separate deduction is allowed in respect of any hands of Mr. Sonu for the P.Y. 2022-23?
other expense (since already standard deduction u/s (a) ₹ 72,800
24(a) is allowed on deemed basis being equal to 30% (b) ₹ 81,200
of NAV) (c) ₹ 1,14,800
Q3) Under which head of income, the amount (d) ₹ 70,700
received from Mr. Raju would be chargeable to Solution) Option (A) is correct
tax? Particulars Chennai Kochi
(a) Income from house property Reasonable Expected Rent 35,000 70,000
(b) Profits and gains from business or Rent received or receivable 38,000 68,000
profession GAV (being higher of 2) 38,000 70,000
(c) Income from other sources Less:- Municipal taxes paid Nil (4,000)
(d) Income from house property or Income by owner
from other sources, at the option of Mr. NAV 38,000 66,000
Sonu Less:- Standard Deduction (11,400) (19,800)
Solution) Option (C) is correct being 30% of NAV allowed
Since rent is being received for an under-construction u/s 24(a)
property, it cannot be made taxable under House Less:- Interest on borrowed Nil Nil
property head. Accordingly, it would subjected to tax capital (not allowed since
under the head ‘Other Sources’ it is not for specified
purposes)
Annual Value of Property 26,600 46,200

Case Study No. 9 :- Income under the head House Property + Capital Gains + Set-off & Carry
Forward of Losses
Mr. Ganesha (a salaried person) has three houses. One in Thane (Maharashtra), second in Jaipur (Rajasthan) and
third in Ratlam (Madhya Pradesh). Details of the flats/houses are as follows:
- Thane flat: 3 BHK flat purchased in April, 2004 for ₹ 90 lakhs. Afterwards, interior work done in 2007 of ₹ 15
lakhs. Mr. Ganesha took loan of ₹ 65 lakhs for purchase of this flat in 2002 and settled full loan in 2020.
- Jaipur house: Purchased in July, 2020 of ₹ 62 lakhs and interior work done in September, 2021 of ₹ 15 lakhs.
Loan taken for purchase of this house of ₹ 15 lakhs in June, 2020. As per interest certificate, he paid ₹
12,00,500 and ₹ 43,500 towards principal and interest, respectively, during the P.Y. 2022-23.
- Ratlam House: Purchased in December 2021 for ₹ 70 lakhs (stamp duty value of ₹ 65 lakhs). For acquiring
this house, he took loan of ₹ 40 lakhs from Canara Bank. Loan was sanctioned on 1.8.2021. He pays EMI of
₹ 38,100 per month. As per interest certificate, for the previous year 2022-23, he paid ₹ 60,900 and ₹ 3,96,300
towards principal and interest, respectively.
Particulars Thane House Jaipur House (April Ratlam House
22 to Dec 22)

 
86 Prepared by : CA Sanchit Grover
  Multiple Choice Questions
  for CA Inter Taxation For May 2023 Exams

Municipal Taxes paid 18,574 8,090 6,909


Municipal value (per month) 30,500 6,800 7,200
Fair Rent (per month) 33,000 7,000 7,500
Standard Rent (per month) 32,000 8,000 7,300

Other details are as follows:


- He has sold Jaipur house on 1st January, 2023 for ₹ 90 lakhs and invested ₹ 15 lakh in RECL bonds issued
by the Central Government on 10th August 2023.
- Mr. Ganesha is working in WinDoor Exports Pvt Ltd, Mumbai and self-occupied Thane flat. He earned salary
of ₹ 22,50,350 for the previous year 2022-23.
- He has no other income from any source for the P.Y. 2022-23.
- He has given Ratlam house on rent for F.Y. 2022-23 to Mr. Pratap on a monthly rent of ₹ 8,500.
- He has given Jaipur house on rent for the period of April, 2022 to June, 2022 to Mrs. Madhura Mahto on
monthly rent of ₹ 7,100 and vacant for remaining period from July, 2022 to December, 2022.
Mr. Ganesha would not like to opt concessional tax rates available under section 115BAC.
Cost inflation index (CII) for the Financial Year (F.Y.) 2020-21 is 301; 2021-22: 317; F.Y. 2022-23: 331.
Based on the facts of the case scenario given above, choose the most appropriate answer to the following
questions:

Q1) What would be Net Annual Value of each house Q2) What would be income/loss under the head
for the previous year 2022-23? “Income from house property” in the hands of
(a) Thane – Nil; Jaipur – ₹ 13,210; Ratlam – Mr. Ganesha?
₹ 95,091 (a) Loss of ₹ 1,67,689
(b) Thane – Nil; Jaipur – ₹ 54,910; Ratlam – (b) Loss of ₹ 2,86,236
₹ 95,091 (c) Loss of ₹ 3,20,489
(c) Thane – Nil; Jaipur – ₹ 21,300; Ratlam – (d) Loss of ₹ 3,63,989
₹ 1,02,000 Solution) Option (D) is correct
(d) Thane – Nil; Jaipur – ₹ 13,210; Ratlam – Thane Jaipur Ratlam
₹ 80,691 (Self (Let Out) (Let Out)
Solution) Option (A) is correct Occupied)
Thane Jaipur Ratlam NAV - 13,210 95,091
(Self (Let Out) (Let Less:- Standard - (3,963) (28,527)
Occupied) Out) Deduction u/s
RER - 84,000 87,600 24(a)
Rent received - 85,200 1,02,000 Less:- Interest Nil (43,500) (3,96,300)
Higher of 2 - 85,200 1,02,000 on borrowed (Full loan
(-) Loss due to - (63,900) Nil capital u/s repaid)
vacancy 24(b)
GAV - 21,300 1,02,000 Loss from Nil (34,253) (3,29,736)
Less:- - (8,090) (6,909) House Property
Municipal
Taxes
NAV - 13,210 95,091
  
Prepared by : CA Sanchit Grover 87
   CHAPTER 18 : CASE SCENARIO BASED MCQS
Q3) How much amount will be carried forward as Period of Holding of Building July 2019 to 1st Jan
loss from house property for the subsequent 2022
assessment year 2024-25? Type of Capital Asset Long term
(a) ₹ 3,63,989 Full Value of Consideration 90,00,000
(b) ₹ 1,63,989 Less:- Indexed Cost of (68,17,940)
(c) ₹ 2,00,000 Acquisition (62L x 331/301)
(d) ₹ 1,50,000 Less:- Indexed Cost of (15,66,246)
Solution) Option (B) is correct Improvement (15L x
Total Loss u/h HP during PY 22-23 is ₹ 3,63,989. Since 331/317)
there is sufficient amount of Salary income, HP loss to LTCG before exemption 6,15,814
the extent of ₹ 2,00,000 shall be set-off against Salary Less:- Exemption u/s 54EC Not allowed since
income. Remaining Loss of ₹ 1,63,989 shall be carried investment made
forward after 6M
Q4) What would the amount of capital gains LTCG 6,15,814
chargeable to tax in the hands of Mr. Ganesha Q5) What would be the gross total income of Mr.
during the previous year 2022-23? Ganesha for the A.Y. 2023-24?
(a) Short-term capital gains of ₹ 13,00,000 (a) ₹ 26,66,164
(b) Long-term capital gains of ₹ 21,82,060 (b) ₹ 24,66,164
(c) Long-term capital gain of ₹ 6,15,814 (c) ₹ 26,16,164
(d) Long-term capital gain of Nil, since he is (d) ₹ 24,55,931
eligible for deduction u/s 54EC in respect Solution) Option (C) is correct
of amount invested in RECL bonds issued Income u/h Salary (22,50,350 – 22,00,350
by Central Government 50,000)
Solution) Option (C) is correct Loss u/h HP (maximum set-off (2,00,000)
allowed of ₹ 2L)
LTCG 6,15,814
GTI 26,16,164

Case Study No. 10 :- Income under the head PGBP


“LUX Enterprise” a proprietorship firm of Mr. Lucifer Mornigstar, a resident individual, in Maharashtra engaged in
business of printing and publishing. The following details pertain to the assets of the business:
Particulars Date of Purchase Date of Put to Amount
use
Office building superstructure constructed on leased 30.09.2022 30.12.2023 1,85,00,000
land
BMW M4 convertible car 23.08.2020 25.08.2021 94,80,000
Machineries used in printing and publishing process 25.09.2022 15.10.2022 9,12,500
Notes:
(1) Car is also used for personal purposes; disallowance for personal use may be taken at 20%.
(2) Written down value of Plant & Machinery (Depreciable @15%) as on 1.4.2021 is ₹ 1,45,00,000.
Based on the facts of the case scenario given above, choose the most appropriate answer to the following
questions, assuming all the aforementioned assets are purchased through account payee cheque:
 
88 Prepared by : CA Sanchit Grover
  Multiple Choice Questions
  for CA Inter Taxation For May 2023 Exams

Q1) What would be the amount of depreciation (d) ₹ 1,57,25,000


allowable on plant and machinery (@15%) for Solution) Option (C) is correct
the previous year 2022-23? Opening WDV of Building 10% Block Nil
(a) ₹ 24,25,938 as on 01-04-2022
(b) ₹ 23,34,688 Add:- Actual Cost of Building (Put ot 1,85,00,000
(c) ₹ 24,94,375 use for less than 180 days)
(d) ₹ 24,03,125 Closing WDV 1,85,00,000
Solution) Option (B) is correct Normal Depreciation 9,25,000
Opening WDV of P&M 15% Block as 1,45,00,000 (185L x 5%)
on 01-04-2022 Additional Depreciation Not
Add:- Actual Cost of P&M used in 9,12,500 allowed
Printing Press (Put ot use for less than Opening WDV as on 01-04-2023 1,75,75,000
180 days) Q4) What would be the amount of depreciation
Closing WDV 1,54,12,500 allowable on BMW M4 convertible car for the
Normal Depreciation 22,43,438 previous year 2022-23?
(145L x 15% + 9,12,500 x 7.5%) (a) ₹ 22,75,200
Additional Depreciation 91,250 (b) ₹ 11,37,600
(9,12,500 x 10%) (c) ₹ 8,80,957
Total Depreciation allowable during 23,34,888 (d) ₹ 13,14,156
PY 22-23 Solution) Option (C) is correct
Normal depreciation allowable on the car will be 15%.
Q2) What would be the WDV of plant and machinery
Further, no additional depreciation is allowed on car.
(Depreciable@15%) as on 1.4.2023?
Hence, Total depreciation allowable on car during PY
(a) ₹ 1,29,86,562
22-23 will be calculated as under:-
(b) ₹ 1,29,18,125
Purchase of Car in PY 20-21 (put to 94,80,000
(c) ₹ 1,30,77,812
use for more than 180 days)
(d) ₹ 1,30,09,375
Solution) Option (C) is correct Less:- Depreciation @30% for PY 20- (11,37,600)
21, allowed only to the extent of 80%
Closing WDV of 15% P&M Block as 1,54,12,500
being used for business
on 31-03-2022
Opening WDV of Car as on 1st April 83,42,400
Less:- Normal + Additional (23,34,888)
2021,
depreciation allowed u/s 32
Less:- Depreciation @30% for PY 21- (10,01,088)
Opening WDV of Block as on 01-04- 1,30,77,813
22 allowed only to the extent of 80%
2023
being used for business
Q3) What would the WDV of Office building Opening WDV of Car as on 1st April 73,41,312
superstructure constructed on leased land as on 2022
1.4.2023? Less:- Depreciation @30% for PY 22- 8,80,957
(a) ₹ 1,85,00,000 23 allowed only to the extent of 80%
(b) ₹ 1,66,50,000 being used for business
(c) ₹ 1,75,75,000

  
Prepared by : CA Sanchit Grover 89
   CHAPTER 18 : CASE SCENARIO BASED MCQS
Case Study No. 11 :- Income under the head PGBP + TDS
Mr. X has set up a manufacturing unit in Chittor, Andhra Pradesh on 1st April 2021.
During the previous year 2021-22 and 2022-23, Mr. X has purchased following assets:
Date of Put to Use Asset Amount in ₹
7 June 2021 Plant & Machinery X 14,75,340
25 July 2021 Office Furniture 7,65,400
14 Jan 2022 Plant & Machinery Y 5,00,000
15 May 2022 Plant & Machinery Z 8,00,000

He has paid professional fees of ₹ 35,000 each to Mr. A, Mr. B and Mr. C, respectively on 10th September 2021
credited in the books on the same day, to discuss some legal matter related to business.
The net profit computed in accordance with “Chapter IV-D - Computation of business income” of the Income-tax Act,
1961 for the previous year 2021-22 is ₹ 1.2 crore.
Based on the facts of the case scenario given above, choose the most appropriate answer to the following
questions:

Q1) What would be the amount of depreciation in deduction while computing profits and gains
respect of Plant & Machinery “Y” allowable as from business or profession?
deduction while computing income under the (a) ₹ 3,77,481
head “Profit & Gains from business or (b) ₹ 3,71,856
profession” for the previous year 2021-22? (c) ₹ 5,54,607
(a) ₹ 61,875 (d) ₹ 6,04,607
(b) ₹ 1,11,875 Solution) Option (D) is correct
(c) ₹ 69,375 Particulars P&M Furniture
(d) ₹ 63,750 Purchase of assets during 19,75,340 7,65,400
Solution) Option (B) is correct PY 21-22
Purchase of P&M Y on 14 Jan 2022
th
5,00,000 Less:- Normal Depreciation (2,58,801) (76,540)
(Put to use for less than 180 days during during PY 21-22
PY 21-22) Less:- Additional (3,45,068) Nil
Less:- Normal Depreciation @7.5% (37,500) Depreciation during PY 21-
during PY 21-22 22
Less:- Additional Depreciation @10% (50,000) Opening WDV as on 1st 13,71,471 6,88,860
during PY 21-22 April 2022
Opening WDV as on 1 April 2022
st
4,12,500 Add:- Purchased Plant Z 8,00,000 Nil
Normal Depreciation @15% for PY 22- 61,875 Closing WDV as on 31-03- 21,71,471 6,88,860
23 2023
Additional Depreciation @10% allowed 50,000 Normal Depreciation 3,25,721 68,886
during PY 22-23 @15% for PY 22-23
Total Depreciation during PY 22-23 1,11,875 Additional Depreciation on 50,000 -
Plant Y @10% allowed
Q2) What shall be the total amount of depreciation
during PY 23-2
for the previous year 2022-23 allowable as

 
90 Prepared by : CA Sanchit Grover
  Multiple Choice Questions
  for CA Inter Taxation For May 2023 Exams

Additional Depreciation on 1,60,000 - (c) No, tax is to be deducted, since amount


Plant Z @20% does not exceed the threshold limit
Total Depreciation during 5,35,721 68,886 (d) Yes, TDS amounting to ₹ 10,500 @10% on
PY 22-23 ₹ 1,05,000 is to be deducted
Solution) Option (D) is correct
Q3) Mr. X wanted to know from you, whether tax is
Under Section 194J, TDS is required to be deducted by
required to be deducted on professional fees
any payer (other than an individual or HUF having
paid to Mr. A, Mr. B and Mr. C respectively. If tax
total turnover not exceeding ₹ 1 Crore in immediately
has to be deducted, then what would be the rate
preceding FY) on payment of Fees for Professional
and amount of tax to be deducted at source?
services if the amount paid or credited during PY
(a) Yes, TDS amounting to ₹ 7,875 @7.5% on
exceeds ₹ 30,000. Accordingly, in the given case, Mr.
₹ 1,05,000 is to be deducted
X is required deduct TDS @10% on amount of ₹ 35000
(b) Yes, TDS amounting to ₹ 1,575 @1.5% on
each paid to Mr. A, Mr. B and Mr. C
₹ 1,05,000 is to be deducted

Case Study No. 12 :- Income under the head PGBP + Filing of Returns under Income Tax
ABC & Co. is a partnership firm engaged in the business of sale of footwear. The partnership firm consist of three
partners – A, B & C. A & B are working partners and C is a sleeping partner. The firm is liable to tax audit under
section 44AB of the Act. It has a book profit of ₹ 11,50,000.
Following payments were made to partners as authorised by the partnership deed:
• Remuneration to A & B - ₹ 32,000 p. m. to each partner
• Remuneration to C - ₹ 10,000 p. m.
• Interest on capital @ 19.5% to A & B - ₹ 18,500 p. a. to each partner
• Interest on capital @ 17% to C - ₹ 10,540 p. a.
The firm has following brought forward losses of past years
A.Y. Business Loss Unabsorbed depreciation Long-term Capital Loss
2020-21 26,000 17,600 5,300
2021-22 78,000 29,860 -
2022-23 1,05,670 54,180 13,470

Based on the facts of the case scenario given above, choose the most appropriate answer to the following
questions:

Q1) What amount of interest is allowable as As per Sec 40(b) r/w Sec 184, Interest on capital to
deduction in the hands of firm while computing partners allowable to a firm shall be lower of the
profits and gains from business or profession? following 2:-
(a) ₹ 29,040  Rate specified in Deed
(b) ₹ 22,770  12%
(c) ₹ 47,540 Max. Interest allowable on Capital of 11,385
(d) ₹ 30,210 Partner A [18,500/ 19.5% x 12%]
Solution) Option (D) is correct Max. Interest allowable on Capital of 11,385
Partner B [18,500/ 19.5% x 12%]
  
Prepared by : CA Sanchit Grover 91
   CHAPTER 18 : CASE SCENARIO BASED MCQS
Max. Interest allowable on Capital of 7,440 (c) 31st October 2023 for both Mr. A and Mr.
Partner C [10,540/ 17% x 12%] C
Maximum permissible Interest 30,210 (d) 31st October 2023 for Mr. C and 31st July
2023 for Mr. A
Q2) What amount of remuneration not allowable as
Solution) Option (C) is correct
deduction in the hands of firm while computing
In case of firm that is subjected to tax audit, Due date
profits and gains from business or profession?
of partners of the firm (whether working partner or
(a) ₹ 1,20,000
non-working partner) shall be 31st October of A.Y.
(b) Nil
Q4) What would be the income under the head
(c) ₹ 1,08,000
“Profits and gains from business or profession”
(d) ₹ 78,000
in the hands of ABC & Co. for the A.Y. 2023-24?
Solution) Option (A) is correct
(a) ₹ 70,690
Maximum Permissible salary to partners allowed as
(b) ₹ 1,72,330
per Sec 40(b) shall be computed as under:-
(c) ₹ 51,920
For first ₹ 3,00,000 Book Profits ₹ 2,70,000
(d) ₹ 1,53,560
(Calculated @90% of Book profits or
Solution) Option (B) is correct
₹ 1,50,000 w.e.v is higher)
Book Profits of the firm 11,50,000
For remaining ₹ 8,50,000 Book ₹ 5,10,000
(after considering all allowances &
Profits (Calculated @60% of Book
disallowances except Salary to
profits)
partners)
Maximum permissible salary to ₹ 7,80,000
Less:- Salary permissible as per Sec (7,68,000)
working partners
40(b)
Salary already paid to working ₹ 7,68,000
Income under the head PGBP 3,82,000
partners (32,000 x 12 x 2)
(Before Set off)
Thus, Salary paid to working partners shall be fully
Less:- Set-off of B/F business loss 2,09,670
allowed. However Salary of ₹ 1,20,000 paid ot non-
Income u/h PGBP 1,72,330
working partner C shall be disallowed.
Note:- Unabsorbed depreciation has not been
Q3) What is the due date of filing of return of separately deducted since it must have already
income for Mr. A and Mr. C for the A.Y. 2023- been deducted while computing Book Profits of ₹
24? 11,50,000 (While computing Book Profits all
(a) 31st July 2023 for Mr. C and 30th deductions u/s 30 to 37 are considered, accordingly
September 2023 for Mr. A since unabsorbed depreciation is deductible u/s 32,
(b) 31st July 2023 for Mr. C and 31st October it must have been considered)
2023 for Mr. A

Case Study No. 13 :- Income under the head Capital Gains + Taxation of Gifts + TDS
Mr. Sarthak (aged 37 years) a share broker, sold a building to his friend Anay, who is a dealer in automobile spare
parts, for ₹ 120 lakh on 10.11.2022, when the stamp duty value was ₹ 150 lakh. The agreement was, however,
entered into on 1.9.2022 when the stamp duty value was ₹ 140 lakh. Mr. Sarthak had received a down payment of
₹ 15 lakh by a crossed cheque from Anay on the date of agreement. Mr. Sarthak purchased the building for ₹ 95
lakh on 10.5.2018. Further, Mr. Sarthak also sold an agricultural land (situated in a village which has a population
 
92 Prepared by : CA Sanchit Grover
  Multiple Choice Questions
  for CA Inter Taxation For May 2023 Exams

of 5,800) for ₹ 60 lakhs to Mr. Vivek on 01.03.2023, which he acquired on 15.06.2015 for ₹ 45 lakhs. Stamp duty
value of agricultural land as on 1.3.2023 is ₹ 65 lakhs.
CII for F.Y. 2015-16; 254; F.Y. 2018-19: 280; F.Y. 2022-23: 331 (RTP May 2020)
Based on the facts of the case scenario given above, choose the most appropriate answer to the following
questions: :

Q1) Is there any requirement to deduct tax at source considered for Sec 50C shall be ₹
on consideration paid or payable for transfer of 150L which is more than 110% of
building and agricultural land? Sale consideration)
(a) Yes; Mr. Anay and Mr. Vivek both required Less:- Indexed Cost of Acquisition (1,12,30,357)
to deduct tax at source under section [95L x 331/280]
194-IA LTCG on Building 37,69,643
(b) Yes; Mr. Anay is required to deduct tax at
Q3) Assuming that Mr. Sarthak has other income
source under section 194-IA.
exceeding basic exemption limit, the tax
(c) Yes; Mr. Vivek is required to deduct tax at
payable (excluding surcharge and health and
source under section 194-IA.
education cess) on transfer of building and
(d) Yes; Mr. Sarthak is required to deduct tax
agricultural land, would be -
at source under section 194-IA.
(a) ₹ 5,53,930
Solution) Option (B) is correct
(b) ₹ 1,53,930
As per Sec 194IA, TDS is required to be deducted only
(c) ₹ 7,53,930
when Sale consideration or SDV of immovable property
(d) ₹ 16,50,000
(other than rural agricultural land) exceeds ₹ 50L. In
Solution) Option (C) is correct
the given case, since agricultural land is situated
In case of transfer of rural agricultural land, no capital
within local limits of municipality having population of
gain shall arise. Further In respect of LTCG on Building,
up to 10,000 it will be treated as rural agricultural land
Tax amount shall be 20% of 42,4 = ₹ 8,48,930/-
and hence no TDS is applicable in its case
Q4) What amount of income is chargeable to tax in
Q2) What amount of capital gains are chargeable to
the hands of Mr. Anay in respect of transfer of
tax in the hands of Mr. Sarthak in respect of
building?
transfer of building?
(a) ₹ 20 lakh
(a) Long-term capital gains of ₹ 37,69,643
(b) ₹ 30 lakhs
(b) Long-term capital gains of ₹ 27,69,643
(c) ₹ 15 lakhs
(c) Long-term capital gains of ₹ 7,69,643
(d) Nil
(d) Short-term capital gains of ₹ 55,00,000
Solution) Option (B) is correct
Solution) Option (A) is correct
In the hands of buyer, since Difference between SDV
Full Value of Consideration 1,50,00,000
of ₹ 150L and purchase price of ₹ 120L is more than
(Since downpayment is received by
₹ 50,000 as well as 10% of Purchase, price, this
crossed cheque, SDV to be
difference of ₹ 30L shall be taxable u/s 56(2)(x)

  
Prepared by : CA Sanchit Grover 93
   CHAPTER 18 : CASE SCENARIO BASED MCQS
Case Study No. 14 :- Residential Status + TDS + Income under the head Salary + Income
under the head Capital Gains
Mr. Narendra Sharma, aged 54 years, an Indian citizen, carrying on retail business in Dubai. He frequently visits India
for business purpose. Details of his visits in India are as follows:
1) Came to India on 03.12.2017 and left India on 26.04.2018
2) Again came to India on 09.09.2020 and left India on 10.01.2021
3) Again came to India on 27.12.2021 and left India on 20.02.2022
Afterwards he decided to shift permanently in India and closed his business in Dubai. So, he came to India on
27.11.2022 and joined Indian Company “Cosmos Heritage India Limited” at registered office in Mumbai from
01.12.2022. From December 2022, he has taken a flat on rent for ₹ 60,000 per month from Mr. Sarthak, an Indian
resident, and Mr. Sarthak has provided his PAN No. to Mr. Narendra Sharma.
Following details of his salary income earned in India:
- Basic Salary – ₹ 2,75,675 per month
- COLA (Cost of Living Allowance) (forms part of retirement benefits) – ₹ 1,20,200 per month
- HRA – ₹ 1,37,838 per month
- Other Allowances – ₹ 1,56,000 per month
For the period from April 2022 to November 2022, his business income arising in Dubai is ₹ 26,00,000 and his
turnover for the P.Y. 2021-22 is ₹ 95,00,000. He is not liable to pay any tax in Dubai. Such business is controlled from
Dubai.
He is active in equity share trading after coming to India. Following are the details of his portfolio
No Sale/ Purchase Company Date of Purchase/ Qty Price per Brokerage
Sale Share (₹)
1. Purchase First Smile Ltd 10.12.2022 250 203 1.5%
2 Purchase Rainbow Ltd 10.12.2022 50 503 1.5%
3. Purchase Mega Service Ltd 12.12.2022 150 82 1.5%
4. Sale First Smile Ltd 18.12.2022 100 325 1.8%
5. Purchase Mega Service Ltd 15.12.2022 110 110 1.5%
6. Sale Mega Service Ltd 26.12.2022 150 100 1.8%
7. Purchase Rainbow Ltd 28.12.2022 200 385 1.5%
8. Purchase Rainbow Ltd 03.01.2023 100 465 1.5%
9. Sale First Smile Ltd 23.03.2023 200 150 1.8%
10. Sale Mega Service Ltd 26.03.2023 110 110 1.8%
Following additional details is also given by Mr. Narendra:
- First Smile Limited issued bonus shares 1:1 on 01.02.2023 and credited the shares in his account on
10.02.2023.
- Rainbow Limited declared an interim dividend of 200% on 28.02.2023 (face value of each share is ₹ 10). The
record date was 31.1.2023.
He does not opt to pay tax as per section 115BAC.
Based on the facts of the case scenario given above, choose the most appropriate answer to the following
questions:

 
94 Prepared by : CA Sanchit Grover
  Multiple Choice Questions
  for CA Inter Taxation For May 2023 Exams

Q1) What is the residential status of Mr. Narendra As on record date (31-01-2023), Mr. Narendra is
for the previous year 2022-23? holding 350 shares of Rainbow Ltd. havig face value
(a) Resident of ₹ 3,500. Hence, Dividend received by him shall be
(b) Resident and ordinary resident 200% of ₹ 3,500 = ₹ 7,000. This shall be taxable as
(c) Non-resident normal income under the head Other Sources.
(d) Deemed resident Further, as per Sec 194, TDS @10% is deductible on
Solution) Option (D) is correct dividend paid if the amount exceeds ₹ 5,000.
In the given case, Mr. Narendra Sharma is an Indian Q3) What shall be the TDS liability of Mr. Narendra
citizen who usually resides outside India and comes for rent paid to Mr. Sarthak?
on a visit to India. Further, from the figures of Salary (a) There is no TDS liability of Mr. Narendra,
in India, it is clear that his Total Income (excluding since he is a salaried individual.
Income from foreign sources is more than ₹ 15L). (b) Mr. Narendra is liable to deduct TDS u/s
Hence, 2 modified Basic condition is applicable.
nd
194-I of ₹ 6,000 for each month.
Number of Days Stay during PY 22-23 = 121 days (c) Mr. Narendra is liable to deduct TDS u/s
Number of Days Stay during 4 preceding PYs = 245 194-IB of ₹ 3,000 for each month
days. (d) Mr. Narendra is liable to deduct TDS u/s
Since neither 1 st
Basic Condition nor 2 nd
Basic 194-IB of ₹ 12,000 in the month of March
condition is getting satisfied in this case, he shall be 2023.
non-resident as per Sec 6(1). However, conditions of Solution) Option (D) is correct
Sec 6(1A) are getting satisfied in this case and hence In the given case, since turnover of Mr. Narendra in
he shall be deemed RNOR as per Sec 6(1A). immediately preceding FY is less than ₹ 1 Crore, he is
Q2) Which of the following statements is correct, in not required to deduct TDS u/s 194I. Further, Since the
respect of dividend paid by Rainbow Ltd. to Mr. rent paid is more than ₹ 50,000, TDS @5% of ₹
Naredra? 2,40,000 is deductible u/s 194-IB. This TDS of ₹ 12,000
(a) Dividend received from Rainbow Ltd is shall be deductible in the month of March
exempt in the hands of Mr. Narendra. Q4) What would be income chargeable to tax under
Hence, no tax is required to be deducted the head “Income from Salaries” in the hands of
at source. Mr. Narendra for the A.Y. 2023-24:
(b) Dividend received from Rainbow Ltd is (a) ₹ 26,27,202
taxable in the hands of Mr. Narendra but, (b) ₹ 26,77,202
since the dividend is less than ₹ 10,000, (c) ₹ 27,08,852
no tax is required to be deducted at (d) ₹ 26,58,852
source. Solution) Option (A) is correct
(c) Dividend received from Rainbow Ltd is Basic Salary (2,75,675 x 4m) 11,02,700
taxable in the hands of Mr. Narendra. Tax Cost of Living Allowance 4,80,800
of ₹ 525 is required to be deducted at (1,20,200 x 4m)
source. HRA 5,51,352 4,69,702
(d) Dividend received from Rainbow Ltd is Less:- Exempt u/s 10(13A) (81,650)
taxable in the hands of Mr. Narendra. Tax Other Allowances 6,24,000
of ₹ 700 is required to be deducted at Gross Salary 26,77,202
source. Less:- Standard Deduction (50,000)
Solution) Option (D) is correct
  
Prepared by : CA Sanchit Grover 95
   CHAPTER 18 : CASE SCENARIO BASED MCQS
Income under the head Salary 26,27,202 Particulars First Smile Ltd Mega Service
Working Notes Ltd.
HRA exempt shall be least of 3:- Sale 62,500 27,100
a) Actually received = ₹ 5,51,352 consideration (100 shares x (150 shares x
b) 50% of (11,02,700 + 4,80,800) = ₹ 7,91,950 325 + 150 100 + 110
c) Rent Paid – 10% of Salary shares x 150 + shars x 110)
= 2,40,000 – 1,58,350 = ₹ 81,650 50 shares x
150)
Q5) What is the amount of short-term capital gain
Expenses on 1,125 487.80
chargeable to tax in the hands of Mr. Narendra
transfer (32,500 x 1.8% (15,000 x 1.8%
on sale of shares for the P.Y. 2022-23:
+ 30,000 x + 12,100 x
(a) ₹ 21,860
1.8%) 1.8%)
(b) ₹ 13,556
Cost of 51,511.25 24,766
(c) ₹ 8,018
acquisition (250 shares x (150 shares x
(d) ₹ 11,710
206.045 + Nil) 83.23 + 110
Solution) Option (D) is correct
shars x 111.65)
STCG 9,863.75 1846.20

Case Study No. 15 :- Taxation of Gift + Income under the head Capital Gains + Clubbing of
Income
Mr. Akshaya Biyani celebrated his 26th birthday on 15th May 2022 and arranged a grand party at Radisson Blu
hotel. On this occasion, he invited his friends, blood relatives and distant relatives to attend the party. The ceremony
was very grand, the feast was also very spectacular. All the arrangements and decorations were absolutely wonderful.
At the end of party, Mr. Akshaya was awarded by gifts and flower’s bouquet as infra:
Gifts received from Type of Gift Remarks
Mother One 22K Gold Chain She purchased on the same day for ₹ 37,822
Father One 22K Gold Bracelet He purchased on the same day for ₹ 56,075
Wife 4 Gold Rings She purchased these rings on 15.5.2021 for ₹ 35,500
each. Fair market value on 15th May 2022 is ₹ 37,429
each.
Sister Painting This painting is made by her. Fair market value is ₹
45,000.
Cousin brother (Father’s One Gold chain He purchased it on the same day for ₹ 18,200.
brother’s son)
Closest cousins (mother’s I-20 Car Value of ₹ 4,10,000
sister’s sons/daughters)
Friends and other distant Cash ₹ 1,51,000
relatives

Mr. Akshaya desires to set up a new manufacturing unit with his friend in partnership on 1.12.2022. For making
investment in the firm, he sold following jewellery which he has received on his 26th birthday celebration as gifts:

 
96 Prepared by : CA Sanchit Grover
  Multiple Choice Questions
  for CA Inter Taxation For May 2023 Exams

- Mother’s gifted Gold Chain for ₹ 42,150


- Father’s gifted Gold Bracelet for ₹ 60,180
- Cousin brother’s gifted Gold Chain for ₹ 20,600
His wife gave him ₹ 1 lakh as a gift so that he could invest sufficient money in the unit.
On 1st December 2022, he invested ₹ 6,00,000 (including the amount received on sale of above gifts and amount
received from his wife) and his friend invested ₹ 4,00,000 in the firm.
On 1st February 2023, his wife again gave him ₹ 1 lakh as a gift to invest such money in the firm and apart from
that he invested ₹ 50,000 more from his individual savings. On this day, his friend also invested ₹ 1,00,000 in the
firm.
Since the firm is a manufacturing unit and at initial stage, the firm requires sufficient fund so Mr. Akshaya sold his
wife’s gifted Gold Rings for ₹ 40,250 each as on 31st March 2023 and he deployed the funds as partner’s capital in
the firm on 01st April, 2023.
Based on the facts of the case scenario given above, choose the most appropriate answer to the following
questions:

Q1) What is the amount of capital gain taxable in STCG on sale of Gold Chain received 2,400
the hand of Mr. Akshaya for P.Y. 2022-23? from Cousin Brother ((₹ 20,600 – ₹
(a) Short term capital gains ₹ 10,833 18,200 )
(b) Short term capital gains ₹ 29,833 STCG taxable during PY 22-23 10,833
(c) Short term capital gains ₹ 22,117 Note:- In all the cases above, since nothing has
(d) No, capital gains is taxable in his hands, been taxable u/s 56(2)(x) in hands of Mr. Akshaya,
since he received the capital assets as hence concept of cost to previous owner shall be
gift. applicable here
Solution) Option (A) is correct
Q2) What is the gift amount not considered as
In the given case, capital assets sold by Mr. Akshaya
income under section 56(2)(x) for P.Y. 2022-23?
during PY 22-23 are as under:-
(a) ₹ 8,98,613
1. Gold chain received from Mother
(b) ₹ 3,06,813
2. Gold Bracelet received from Father
(c) ₹ 9,16,813
3. Gold Chain received from Cousin Brother
(d) ₹ 7,16,813
4. 4 Gold rings received from Wife
Solution) Option (C) is correct
Out of above 4, capital gain arising on transfer of first
Gift of Gold Chain from mother (not 37,822
3 capital assets would be taxable in hands of Mr.
taxable since received from relative)
Akshaya but capital gains on transfer of 4th Capital
Gift from Father (not taxable since 56,075
asset (i.e. Gold rings) would be clubbed in hands of
received from relative)
Wife u/s 64. Hence Capital gains for Mr. Akshaya shall
Gift of 4 Gold rings from Wife (not 1,49,716
be computed as under:-
taxable since received from relative)
STCG on sale of Gold chain from Mother 4,328 Gift of Painitng from Sister (not taxable 45,000
((₹ 42,150 – ₹ 37,822) since received from relative)
STCG on sale of Gold Bracelet from 4,105 Gift of Gold Chain from Cousin brother 18,200
Father ((₹ 60,180 – ₹ 56,075) (although received from non-relative, it

  
Prepared by : CA Sanchit Grover 97
   CHAPTER 18 : CASE SCENARIO BASED MCQS
is not taxable since aggregate value of (a) No
gift is less than 50,000) (b) Yes; ₹ 15,284
Gift of I-20 Car (although received from 4,10,000 (c) Yes; ₹ 19,000
non-relative, it is not taxable since car (d) Yes; ₹ 11,284
is not specified moveable asset ) Solution) Option (C) is correct
Cash gift from wife for investment in 2,00,000 In the given case, gifts received from wife were
business (not taxable since received invested in the business during PY 22-23 and not on
from relative) 1st day of PY (i.e. 01-04-2022). Hence profits from
Gifts not taxable 9,16,813 business shall not be clubbed in hands of wife.
However, capital gains computed in sale of 4 Gold
Q3) What is the gift amount taxable in the hands of
rings gifted by wife shall be clubbed in hands of wife
Mr. Akshaya for P.Y. 2022-23?
in accordance with Sec 64
(a) ₹ 1,51,000
Period of Holding of rings 15-05-2021 to
(b) ₹ 1,69,200
31st March
(c) ₹ 5,79,200
2023
(d) ₹ 5,61,000
Type of Capital Asset Short term
Solution) Option (A) is correct
capital asset
The only gift taxable in hands of Mr. Akshaya shall be
Full value of consideration 1,61,000
₹ 1,51,000 received in cash from friends since it is not
(₹ 40,250 x 4)
received from relatives and the aggregate amount of
Less:- Cost of Acquisition being 1,42,000
money received without consideration exceeds ₹
equal to purchase price in hands
50,000
of previous owner
Q4) Is any amount taxable in the hands of
(₹ 35,500 x 4 rings)
Akshaya’s wife in respect of sale of jewellery by
STCG to be clubbed in hands of 19,000
Mr. Akshaya, if yes, what shall be the taxable
wife as per Sec 64
amount in her hands for P.Y. 2022-23?

Case Study No. 16 :- Clubbing of Income + Section 115BAC


Mr. Rajesh gifted ₹ 15 lakhs to his wife, Raavi, on her birthday on 23rd February, 2022. Raavi lent ₹ 6,00,000 out of
the gifted amount to Karuna on 1st April, 2022 for six months on which she received interest of ₹ 30,000. The said
sum of ₹ 30,000 was invested in shares of a listed company on 18th October, 2022, which were sold for ₹ 66,000 on
25th March, 2023. Securities transactions tax was paid on purchase and sale of such shares. The balance amount
of gift was invested on 1st April 2022, as capital by Raavi in her new business. She suffered loss of ₹ 22,000 in the
business in Financial Year 2022-23. Raavi is working with a Private company as sales executive at a salary of ₹
62,000 p.m. She paid ₹ 3,500 p.m towards tuition fees for her daughter Riya studying in St. Thomas School, Mumbai.
Rajesh is working with an MNC on a monthly salary of ₹ 64,000. He has gifted ₹ 1,25,000 to Riya on her 13th
Birthday. This amount is deposited as 2 years term deposits with SBI bank in her name. On which interest of ₹ 11,500
is earned during the previous year 2022-23. Both Mr. Rajesh and Mrs. Raavi opt to pay tax under section 115BAC.
Based on the facts of the case scenario given above, choose the most appropriate answer to the following questions:

 
98 Prepared by : CA Sanchit Grover
  Multiple Choice Questions
  for CA Inter Taxation For May 2023 Exams

Q1) In whose hands, the interest income received Since capital in the business of Mrs. Raavi is entirely
from Karuna and interest on fixed deposits in out of the gift received from husband (& it has been
the name of Riya would be included? invested on the first day of PY 22-23 itself), accordingly
(a) both interest income to be included in the entire loss of ₹ 22,000 from such business shall be
hands of Mr. Rajesh clubbed in hands of Mr. Rajesh in accordance with
(b) both interest income to be included in the Sec 64.
hands of Mrs. Raavi However, shares have been purchased by Mrs. Raavi
(c) interest income from Karuna to be not out of gift received from husband but out of
included in the hands of Mrs. Raavi and Interest income of ₹ 30,000 earned. Accordingly,
interest on two years term deposits to be Capital Gain on sale of such shares shall not be
included in the hands of Mr. Rajesh. clubbed (Income generated from accumulated income
(d) interest income from Karuna to be is not clubbed)
included in the hands of Mr. Rajesh and Q3) What would be the total income of Mrs. Raavi
interest on two years term deposits to be for the previous year 2022-23?
included in the hands of Mrs. Raavi. (a) ₹ 6,88,000
Solution) Option (D) is correct (b) ₹ 7,80,000
Interest income earned by Mrs. Raavi shall be clubbed (c) ₹ 7,91,500
in the hands of her husband Mr. Rajesh in accordance (d) ₹ 7,90,000
with Sec 64. However, Interest earned by minor Solution) Option (C) is correct
daughter Riya shall be clubbed in hands of that parent Calculation of Total Income of Mrs. Raavi
whose income is otherwise higher (in the given Interest earned on Loan given to Karuna Nil
question, Total income of Mrs. Raavi is more than Total (Clubbed in hands of husband)
Income of Mr. Rajesh) Salary Income (62,000 x 12 ) 7,44,000
Q2) In whose hands loss from business and capital Business Loss (clubbed in hands of Nil
gains would be included in Assessment Year husband)
2023-24? Assume that capital invested in the Interest earned by minor daughter 11,500
business was entirely out of the funds gifted by [clubbed in accordance with Sec 64(1A)]
her husband. STCG on sale of shares (66,000 – 36,000
(a) Both loss from business and capital gains 30,000)
would be included in the hands of Mr. GTI 7,91,500
Rajesh Less:- Chapter VI-A deductions Nil
(b) Both loss from business and capital gains Total Income 7,91,500
would be included in the hands of Mrs. Note:-
Raavi Since Mrs. Raavi has opted for Sec 115BAC, she
(c) Loss from business included in the hands wont be entitled to claim standard deduction u/s 16
of Mr. Rajesh and capital gains included or Exemption u/s 10(32) or Deduction under
in the hands of Mrs. Raavi Chapter VI-A
(d) Loss from business included in the hands
Q4) What would be total income of Mr. Rajesh for
of Mrs. Raavi and capital gains included
the previous year 2022-23?
in the hands of Mr. Rajesh
Solution) Option (C) is correct (a) ₹ 7,76,000
(b) ₹ 8,09,500
  
Prepared by : CA Sanchit Grover 99
   CHAPTER 18 : CASE SCENARIO BASED MCQS
(c) ₹ 8,08,000 PGBP Loss of Mrs Raavi to be clubbed (22,000)
(d) ₹ 7,98,000 (eligible for set off against Interest
Solution) Option (A) is correct income)
Calculation of Total Income of Mrs. Raavi GTI 7,76,000
Income under the head Salary 7,68,000 Less:- Deduction under Chapter VI-A Nil
(₹ 64,000 x 12) Total Income 7,76,000
Interest income of Mrs. Raavi to be 30,000 Note:-
clubbed Since Mr. Rajesh has opted for Sec 115BAC, he
wont be entitled to claim standard deduction u/s 16
Deduction under Chapter VI-A

Case Study No. 17 :- Income under the head House Property + Sec 115BAC + Return of
Income + Advance Tax
Miss Hetal transferred to his husband, Mr. Hemant, a residential property worth ₹ 45 lakhs located in Nagpur without
any consideration. The expected rent of such property is ₹ 5 lakhs. Municipal tax of ₹ 5,000 paid by Miss Hetal for
this property during the previous year 2022-23. Miss Hetal has three residential properties in Mumbai. The expected
rent from the 3 properties situated in Mumbai is ₹ 10 lakhs, ₹ 11 lakhs and ₹ 12 lakhs respectively. She purchased
the properties out of her own funds. Municipal taxes due are ₹ 15,000, ₹ 20,000 and ₹ 25,000. The same have,
however, not been paid this year in respect of the three properties. The expected rent is lesser than the standard rent
in case of all the aforementioned properties. Miss Hetal does not have any income from any other source
Miss Hetal’s father, aged 58 years had capital gains of ₹ 5 crores from sale of house property. He reinvested the
proceeds from sale in another residential house of ₹ 4.98 crores and the remaining sale proceeds were deposited in
his savings bank account. He has paid ₹ 1,50,000 towards LIC premium. He has no other source of income.
Miss Hetal’s grandfather is aged 81 years and has interest income on fixed deposits of ₹ 6 lakhs. He has no other
income for the P.Y. 2022-23. He has to fly to USA for his treatment of cancer on 31st July, 2023 and his return of
income is not filed before his flying to USA.
Based on the facts of the case scenario given above, choose the most appropriate answer to the following
questions:

Q1) What is the amount of income liable to be taxed she has 4 HPs, all of which are self-occupied.
in the hands of Miss Hetal under the head Accordingly, she can choose any 2 HPs as self
“Income from House Property” for A.Y. 2023-24? occupied and consider NAV as Nil while other 2 HPs
(a) ₹ 7,00,000 shall be considered as Deemed to be let out.
(b) ₹ 10,46,500 In the given case, it is beneficial for Miss Hetal to
(c) ₹ 10,50,000 consider Nagpur HP and Mumbai Property with
(d) ₹ 13,76,500 expected rent of ₹ 10L as Deemed to be Let out.
Solution) Option (B) is correct Acoordingly her Income u/h HP shall be computed as
Since Miss Hetal transferred HP in Nagpur to her under:-
husband without consideration, she will be considered
as Deemed owner of the property as per Sec 27. Now,
 
100 Prepared by : CA Sanchit Grover
  Multiple Choice Questions
  for CA Inter Taxation For May 2023 Exams

Nagpur Mumbai (b) Yes, he is required to furnish return of


GAV (based on expected 5,00,000 10,00,000 income on or before 31st July,2023
rent of HP) (c) Yes, he is required to furnish return of
Less:- Municipal Taxes (5,000) Nil income on or before 30th September,
NAV 4,95,000 10,00,000 2023
Less:- Standard deducton (1,48,500) (3,00,000) (d) Yes, he is required to furnish return of
(30% of NAV) income on or before 31st October, 2023
Income u/h HP 3,46,500 7,00,000 Solution) Option (B) is correct
As per Sec 139(1), an Individual is required to furnish
Q2) What would be tax liability of Miss Hetal for the
Return of income if his Total Income (before
assessment year 2023-24? Compute in a
considering ChapterVI-A deductions and Exemption
manner so that her tax liability is minimum.
u/s 54) is more than exemption limit. In the given case,
(a) ₹ 66,300
although Total Income of Hetal’s Father is less than ₹
(b) ₹ 88,400
2,50,000 but if we do not consider exemption u/s 54
(c) ₹ 87,670
of ₹ 4.98 Crore and Deduction us 80C of ₹ 1.5L, it
(d) ₹ 1,31,510
would surely be more than applicable exemption limit.
Solution) Option (C) is correct
Hence, return filing is mandatory for him
In the given case, since Miss Hetal has not other
Q4) Is Miss Hetal’s grandfather required to pay
income except Income u/h House Property, it would be
advance tax during the previous year 2022-23?
beneficial for her to opt for Sec 115BAC (Note that
(a) No, he is not required to pay advance tax,
she doesn’t have any deduction or exemption that
since he is a senior citizen
would be disallowed by Sec 115BAC)
(b) Yes, he is required to pay advance tax,
Calculation of Tax Liability u/s 115BAC
since his tax liability exceeds ₹ 10,000
On first ₹ 2,50,000 Nil
(c) No, he is not required to pay advance tax,
On Next ₹ 2,50,000 @5% 12,500
since he is a senior citizen and he is not
On Next ₹ 2,50,000 @10% 25,000
having any income under the head
On Next ₹ 2,50,000 @15% 37,500 “Profits and gains from business or
On Remaining ₹ 46,500 @20% 9,300 profession”
Total Tax before HEC 84,300 (d) Yes, he is required to pay advance tax,
Add:- HEC @4% 3,372 since his total income exceeds basic
Final Tax Liability 87,672 exemption limit of ₹ 5,00,000
Rounded Off u/s 288B 87,670 Solution) Option (C) is correct
Q3) Is Hetal’s father required to furnish his return of In the given case, since Hetal’s Grandfather only has
income in India for the A.Y.2023-24? Interest income (and no PGBP Income), he would not
(a) No, he is not required, since his income be required to pay any advance tax
does not exceed basic exemption limit

  
Prepared by : CA Sanchit Grover 101
   CHAPTER 18 : CASE SCENARIO BASED MCQS
Case Study No. 18 :- Income under the head PGBP + Capital Gains + Set-Off & Carry Forward
of Losses + TDS
Ms. Chanchal, aged 45, provides the following data of her gross receipts for the financial year 2021-22 and 2022-
23. She is engaged in agency business along with providing services as tarot card reader. She is generally engaged
in cash payments and cash receipts
F.Y. Receipts from business (₹) Receipts from profession (₹) Total Gross Receipts (₹)
2021-22 1,05,00,000 47,00,000 1,52,00,000
2022-23 98,00,000 49,00,000 1,47,00,000
She paid an amount of ₹ 12,00,000 to a contractor for polishing her old furniture in her self-occupied residential
house property on 12.04.2022. Further on 05.06.2022, she has taken services from renowned interior designer for
the same residential house property for which she paid ₹ 2,50,000.
On 28.05.2022, she sold one commercial property for ₹ 50,00,000. The stamp duty value on the date of registration
is ₹ 58,00,000. The value adopted for stamp duty was ₹ 54,00,000 on the date of agreement (part payment by account
payee cheque was received on the date of agreement). It was purchased for ₹ 40,00,000 on 28.06.2019. (Cost Inflation
Index for F.Y. 2022-23: 331, F.Y. 2019-20: 289).
The brought forward long-term capital loss from unlisted shares of F.Y. 2021-22 is ₹ 5,50,000.
During the year, Ms. Chanchal incurred a loss of ₹ 70,00,000 while trading in the agricultural commodity derivatives
(no CTT paid).
Ms. Chanchal does not want to opt for the new tax regime available under section 115BAC for A.Y. 2023-24.
Based on the facts of the case scenario given above, choose the most appropriate answer to the following
questions: - (RTP Nov 2020)

Q1) Is Ms. Chanchal liable to tax audit under the Q2) With respect to payment made to contractor and
Income-tax Act, 1961 for the P.Y. 2022-23? to the interior designer during the P.Y. 2022-23,
(a) Yes, as the total gross receipts exceeds ₹ Ms. Chanchal consulted various persons and
1,00,00,000 they have the following views -
(b) No, as the gross receipts from business or (i) She is required to deduct tax at source
profession are below the specified under section 194C and 194J, since her
threshold limits. turnover from business for the previous
(c) Yes, as the gross receipts from business year 2021-22 exceeds ₹ 1,00,00,000
exceeds ₹ 50,00,000 (ii) She is required to deduct tax at source
(d) Yes, as the gross receipts from profession under section 194M on both the
exceeds ₹ 25,00,000 payments
Solution) Option (B) is correct (iii) She is not required to deduct tax at source
In the given case, since Chanchal usually carries neither under section 194C nor under
business in cash, applicable threshold limit for Sec section 194J, since such amounts are
44AB shall be ₹ 1 Crore for Business & ₹ 50L for paid for personal purposes
profession. In the given PY 22-23, her Turnover from (iv) She is not required to deduct tax at source
business is less than ₹ 1 Cr. and her Gross receipts under section 194M, since payment to
from profession is less than ₹ 50L and hence no tax each individual does not exceed ₹
audit u./s 44AB shall be required 50,00,000
 
102 Prepared by : CA Sanchit Grover
  Multiple Choice Questions
  for CA Inter Taxation For May 2023 Exams

Which views are correct? consideration, Sec 50C shall not be


(a) (iii) and (iv) views are correct applicable here)
(b) (i) view is correct Less:- Cost of acquisition (40,00,000)
(c) (ii) view is correct (Since period of holding is less than
(d) (i) and (iv) views are correct 24 months, it is a case of STCG)
Solution) Option (A) is correct STCG on sale of building 10,00,000
Sec 194C and Sec 194J provide for deduction of TDS
Q4) What is the amount of losses which can be
only when payment for contract & professional
carried forward to A.Y. 2024-25, assuming that
services are made for business purpose. In the given
business income is ₹ 45,00,000 and income
case, since these payments were made for self-
from profession is ₹ 25,00,000 for the P.Y. 2022-
occupied house of Chanchal, hence no TDS
23?
implications under these sections shall arise
(a) ₹ 5,50,000 under section 74
Further, Sec 194M provides for deduction of TDS on
(b) ₹ 70,00,000 under section 73
Payments made for contracts of work and professional
(c) No loss is required to be carried forward,
services only when aggregate of such payments
since brought forward loss and current
exceeds ₹ 50L during PY. Since such condition is not
year loss are set-off against current year’s
getting satisfied here, no TDS u/s 194M shall be
income
applicable.
(d) ₹5,50,000 under section 74 and ₹
Q3) What is the amount and nature of Capital gain
70,00,000 under section 73
chargeable to tax in the hands of Ms. Chanchal?
Solution) Option (A) is correct
(a) ₹ 14,00,000 and Short-term capital gain.
Trading in Commodity derivatives is not a speculative
(b) ₹ 10,00,000 and Short-term capital gain.
transaction and hence Loss of ₹ 70L is non-
(c) ₹ 11,00,000 and Long-term capital gain.
speculative PGBP loss that can be set off against PGBP
(d) ₹ 7,00,000 and Long-term capital gain.
income of 45L and 25L. However, LTCL of ₹ 5.5L
Solution) Option (B) is correct
cannot be set off against STCG or PGBP income, hence
Full Value of Consideration 50,00,000
it shall have to be carried forward
(Since SDV as on date of agreement
is not more than 110% of Sale

Case Study No. 19:- Clubbing of Income + Income under Capital Gains + Deductions under
Chapter VI-A + Computation of Tax Liability
Mr. Abhishek Seth, aged 42 years, is working as a CEO of Soil Limited. He provides you the following information for
preparation and filing of his income-tax return for the year ended 31st March 2023:
• Salary, allowances and perquisites from Soil Limited – ₹ 1,35,00,000
• Dividend from ABC Ltd. which was declared in February, 2022 and received in April, 2022 - ₹ 4,55,000
• Dividend from PRQ Ltd. declared and received in July, 2022 - ₹ 5,90,000 (Gross)
• Interest income on saving bank account in SBI – ₹ 24,530
• Long term capital gains on transfer of residential house in Mumbai on 15th December, 2022 - ₹ 1,73,540
Short term capital gain on transfer of listed equity shares (STT paid both at the time of transfer and acquisition) of
Ind Ltd. - ₹ 73,00,000
He also furnished the following details of investment/ payments made by him during the P.Y. 2022-23:
  
Prepared by : CA Sanchit Grover 103
   CHAPTER 18 : CASE SCENARIO BASED MCQS
(a) Three-year post office time deposit - ₹ 25,000
(b) Contribution to PPF - ₹ 35,000
(c) Tuition fees of three children in Bharti Sr. Sec. School in Delhi - ₹ 20,000 per annum per children
(d) Subscription to NHAI redeemable bonds after 5 years on 16th March, 2023 - ₹ 2,00,000.
Further, his son Mr. Dhaval, aged 15 years, has also earned the following income:
(a) Income from a quiz competition - ₹ 25,000
(b) Interest on bank fixed deposit - ₹ 9,500
Assuming that the tax has been deducted on time, wherever applicable. Mr. Abhishek does not want to opt for the
provision of section 115BAC.
Based on the facts of the case scenario given above, choose the most appropriate answer to the following
questions:-

Q1) What is the quantum of income of Mr. Dhaval Less:- Exempt u/s 54EC (1,73,540)
which is to be clubbed with the income of Mr. Since NHAI Bonds were purchased
Abhishek, if any, assuming that income of Mr. within 6 months from date of transfer,
Abhishek is greater than the income of his exemption u/s 54EC shall apply
spouse? STCG u/s 111A 73,00,000
(a) ₹ 34,500 Income of minor child clubbed 8,000
(b) ₹ 8,000 GTI 2,13,72,530
(c) ₹ 33,000
Q3) What is the amount of deduction allowable
(d) ₹ 9,500
under section 80C to Mr. Abhishek?
Solution) Option (B) is correct
(a) ₹ 1,00,000
Income from quiz competition is not be clubbed u/s
(b) ₹ 1,20,000
64(1A) since Dhaval has earned this income through
(c) ₹ 95,000
his own skill and talent. However, Interest income of ₹
(d) ₹ 75,000
9,500 shall be clubbed in hands of Mr. Abhishek [after
Solution) Option (D) is correct
allowing exemption of ₹ 1,500 u/s 10(32)]
Investment in 3 yr deposit Not eligible for
Q2) What is the gross total income of Mr. Abhishek
in PO deduction u/s 80C
for A.Y. 2023-24?
Contribution to PPF 35,000
(a) ₹ 2,13,72,530
Tuition fees for children 40,000
(b) ₹ 2,14,22,530
(Deduction allowed for
(c) ₹ 2,13,64,530
max of 2 children)
(d) ₹ 2,15,46,070
Solution) Option (A) is correct Q4) What shall be the tax liability of Mr. Abhishek
Salary [1,35,00,000 – 50,000] 1,34,50,000 for A.Y. 2023-24?
Dividend from ABC Ltd. (taxable in Nil (a) ₹ 62,67,350
year of declaration) (b) ₹ 61,04,100
Dividend from PQR Ltd. (Gross) 5,90,000 (c) ₹ 59,60,050
Interest from saving account (No TDS 24,530 (d) ₹ 61,45,610
is applicable on this) Solution) Option (B) is correct
LTCG u/s 112 1,73,540 Nil

 
104 Prepared by : CA Sanchit Grover
  Multiple Choice Questions
  for CA Inter Taxation For May 2023 Exams

GTI 2,13,72,530 Tax on STCG u/s 111A of ₹ 73L 10,95,000


Less:- Deduction u/s 80C (75,000) @15%
Less:- Deduction u/s 80TTA (10,000) Tax computed before Surcharge 51,03,759
Total Income 2,12,87,530 Add:- Surcharge @15% 765563.85
Tax Computed on Normal income 40,08,759 Tax before HEC 58,69,322.85
of ₹ 1,39,87,530 Add:- HEC@4% 2,34,772.91
(Nil + 12,500 + 1,00,000 + Tax after HEC 61,04,095.76
38,96,259) Tax after rounding off u/s 288B 61,04,100

Case Study No. 20 :- Return of Income + Deductions under Chapter VI-A + Computation of
Tax Liability
M/s Abhinav & sons, a sole proprietorship is engaged in the business of manufacturing pharmaceutical products
and it had started its business on 20th June 2018. Tax head of M/s Abhinav & sons furnishes you the following
particulars for the year ended 31 March 2023:
• Income under the head PGBP – ₹ 5,75,22,750
• Interest on fixed deposits (Gross) - ₹ 12,50,000 [The same was received on 30th April, 2022 after deduction of
tax at source]
• Donation to PM Cares Fund - ₹ 2,50,000
• Turnover during the previous year 2022-23 - ₹ 15,50,00,000
M/s Abhinav & sons does not want to opt for the provisions of section 115BAC. It has employed total 150 employees
during the P.Y. 2021-22 with an annual increment of 10% in their monthly emoluments. Details of the same are as
under:
Date of joining No. of employees Employee category Monthly Participate in
emoluments per recognised
employee (₹) provident fund
1.5.2021 50 Regular 26,500 Yes
1.6.2021 65 Casual 23,000 No
1.7.2021 35 Regular 22,500 Yes

It has employed further 50 employees during the P.Y. 2022-23. Details of the same are as under
Date of joining No. of employees Employee category Monthly Participate in
emoluments per recognised
employee provident fund
1.4.2022 20 Regular 21,000 Yes
1.8.2022 30 Regular 26,000 Yes

Emoluments to all the employees are being paid by way of account payee cheque only. No employees have left the
job during P.Y. 2021-22 as well as during P.Y. 2022-23.
Based on the facts of the case scenario given above, choose the most appropriate answer to the following questions:

  
Prepared by : CA Sanchit Grover 105
   CHAPTER 18 : CASE SCENARIO BASED MCQS
Q1) What is the due date of filing of return of (a) ₹ 5,70,10,750
income of M/s Abhinav & sons for A.Y. 2023- (b) ₹ 5,48,84,500
24? (c) ₹ 5,57,60,750
(a) 31st July, 2023 (d) ₹ 5,52,64,250
(b) 30th November, 2023 Solution) Option (B) is correct
(c) 30th September, 2023 Income u/h PGBP 5,75,22,750
(d) 31st October, 2023 Interest on FD 12,50,000
Solution) Option (D) is correct GTI 5,87,72,750
Since the turnover from the business is more than ₹ Less:- Deduction u/s 80JJAA (36,38,250)
10 Crore, Audit u/s 44AB would become mandatory Less:- Deduction u/s 80G (2,50,000)
and accordingly, due date of filing return u/s 139 shall Less:- Deduction u/s 80TTB Nil
be 31st October (No where mentioned in question
Q2) What shall be the amount of deduction that he is a senior citizen)
available to M/s Abhinav & sons under section Total Income 5,48,84,500
80JJAA for A.Y. 2023-24?
Q4) What would be the tax payable of M/s Abhinav
(a) ₹ 36,38,250
& sons for the A.Y. 2023-24?
(b) ₹ 15,12,000
(a) ₹ 2,47,47,810
(c) ₹ 46,30,500
(b) ₹ 1,94,68,310
(d) ₹ 33,84,000
(c) ₹ 2,31,92,680
Solution) Option (A) is correct
(d) ₹ 2,30,67,680
Deduction in respect of 50 15,12,000
Solution) Option (D) is correct
employees employed during PY
Tax payable on normal income of 1,62,77,850
22-23 (20 employees x 21000 x 12
₹ 5,48,84,500
months x 30%)
(Nil + 12,500 + 1,00,000 +
Note:- Deduction shall not be allowed for 30 regular
1,61,65,350)
workers since their Salary is more than ₹ 25,000
Add:- Surcharge of 37% 60,22,804.50
pm.
Tax after Surcharge 2,23,00,654.50
Deduction allowed in PY 22-23 for 21,26,250
HEC @4% 8,92,026.18
35 regular additional employees
employed on 01-07-2021 Tax after HEC 2,31,92,680.68
[35 employees x ₹ 22,500 x 9 Rounded off 2,31,92,680
months x 30%] Less:- TDS already deducted u/s (1,25,000)
Note:- Deduction is allowed u/s 80JJAA for a period 194A
of 3 years, that’s why deduction has been allowed Tax amount payable 2,30,67,680
in PY 22-23 in respect of eligible additional WN 1) Calculation of Marginal Relief
employees employed during PY 21-22. Further, 50 Tax on ₹ 5,48,84,500 2,23,00,654.50
regular employees who joined on 01-05-2021 Tax computed on ₹ 5,00,00,000 1,85,15,625
would not be eligible since their Salary is more than (Nil + 12,500 + 1,00,000 +
₹ 25,000 pm and remaining 65 employees who 14,70,000) + Surcharge of 25%
joined on 01-06-2021 would also not be eligible Incremental Tax 37,85,029.50
since the don’t participate in RPF Since Incremental tax is not more than Incremental
Q3) What would be the total income of M/s Abhinav income, hence no marginal relief shall be allowed
& sons for the A.Y. 2023-24? in this case

 
106 Prepared by : CA Sanchit Grover
  Multiple Choice Questions
  for CA Inter Taxation For May 2023 Exams

Case Study No. 21 :- Residential Status + Sec 115BAC + Deductions under Chapter VI-A +
TDS + Advance Tax
Mr. X wanted to file his return of income for the previous year 2022- 23. He required assistance for which he has
approached you. He has shared the following details relevant to the P.Y. 2022-23.
Mr. X owned a house property in Mumbai and the same was rented out for ₹ 70,000 p.m. He claims that this was
the only income which he earned during the P.Y. 2022-23. However, when you had sought for his bank statement,
you observed the following information additionally.
There is a credit for ₹ 23,975 towards income-tax refund which includes ₹ 5,775 towards interest on income-tax
refund. On 15th August, 2021, the bank statement showed a credit of ₹ 55,000 which he claimed to have received
as a gift from his grandchildren on his 60th birthday. On further assessment you were able to understand that Mr.
X and his wife had travelled to Australia during the P.Y. 2022-23 to spend some time with their daughter, who is
staying in Australia, since her marriage. On scrutiny of their passport and relevant documents you conclude that they
had left India on 27th September, 2022 and retuned on 30th March, 2023. During the 4 years preceding previous
year 2022-23, both had stayed in India for 320 days. Prior to that, they had been staying only in India.
Based on the facts of the case scenario given above, choose the most appropriate answer to the following
questions:

Q1) What is the residential status of Mr. X for the (8,40,000 – 30%)
P.Y. 2022-23? Interest on Income tax refund 5,775
(a) Resident and ordinarily resident Gift from grandchildren (not taxable Nil
(b) Resident but not ordinarily resident since they are covered in the
(c) Non-resident definition of ‘relative’)
(d) Deemed resident but not ordinarily Total Income (after rounding off) 5,93,770
resident Tax liability (Nil + 10,000 + 18,754) 29,904
Solution) Option (A) is correct + 4% HEC
No. of days stay in India during PY 22-23 is 182 days Calculation of Tax liability if Sec 115BAC is opted
and hence he shall become resident. Further, since he Total Income 5,93,770
had been staying in India only, he must be satisfying (Since Mr. X has not claimed any of
additional conditions. Accordingly, he shall be ROR the restricted deductions or
Q2) Mr. X requests you to compute his tax liability exemptions, hence his Total Income
for the A.Y. 2023-24 in a manner such that his shall remain the same)
tax liability is minimum. Accordingly, his tax Tax Liability (Nil + 12,500 + 9,377) + 22,752
liability would be 4% HEC
(a) ₹ 22,750 Conclusion
(b) ₹ 29,910 In the given case, it is beneficial to opt for Sec
(c) ₹ 32,510 115BAC to ensure minium tax liability
(d) ₹ 20,150
Solution) Option (A) is correct Q3) In continuation to question Q2), what would be
tax liability of Mr. X for the A.Y. 2023-24, if he
Calculation of Tax liability if Sec 115BAC is not
had paid ₹ 1,00,000 towards life insurance
opted
premium for self?
Income u/h House Property 5,88,000
  
Prepared by : CA Sanchit Grover 107
   CHAPTER 18 : CASE SCENARIO BASED MCQS
(a) ₹ 20,150 (c) Yes, Mr. Y is required to deduct tax at
(b) ₹ 29,910 source of ₹ 31,500
(c) ₹ 10,400 (d) No, there is no requirement to deduct tax
(d) Nil at source, since Mr. X is a non-resident
Solution) Option (D) is correct Solution) Option (B) is correct
Calculation of Tax liability if Sec 115BAC is not Since Mr. Y is a salaried employee, he would not be
opted liable to deduct TDS u/s 194I. However, since rent paid
Gross Total Income 5,93,770 is more than ₹ 50,000 p.m., he would be liable to
Less:- Deduction u/s 80C (1,00,000) deduct TDS u/s 194IB. Amount to be deducted shall be
Total Income 4,93,770 5% of (70,000 x 12) = ₹ 42,000. This TDS has to be
Tax liability (Nil + 9,688.50) 9,688.50 deducted from the rent paid of last month of FY or
Less:- Rebate u/s 87-A 9,688.50 month of termination of lease agreement, whichever
Tax liability Nil is earlier.
Calculation of Tax liability if Sec 115BAC is opted Q5) Which of the following statements is correct
Total Income 5,93,770 with respect to advance tax liability of Mr. X for
(Deduction u/s 80C is not allowed if P.Y. 2022-23?
Sec 115BAC has been opted) (a) Advance tax liability shall not arise to Mr.
Tax Liability (Nil + 12,500 + 9,377) + 22,752 X since he is a non-resident
4% HEC (b) Advance tax liability shall not arise, since
Conclusion Mr. X is a resident senior citizen and he
In the given case, it is beneficial not to opt for Sec has no income chargeable under the
115BAC to ensure minium tax liability head “Profits and gains of business or
profession
Q4) Mr. X had given the house property at Mumbai
(c) Advance tax liability shall arise, since he
on rent to Mr. Y, a salaried employee. Is there
is a non-resident
any requirement to deduct tax at source on such
(d) Advance tax liability shall arise, since his
rent by Mr. Y, if yes, what would be the amount
tax liability is not less than ₹ 10,000
of TDS to be deducted? Solution) Option (B) is correct
(a) No, there is no requirement to deduct tax In case of any individual who is atleast 60 years of age
at source, since Mr. Y is a salaried at any point of time during PY + Resident in India, such
employee person is not required to pay any advance tax provided
(b) Yes, Mr. Y is required to deduct tax at he has no income cha
source of ₹ 42,000
rgeable to tax under the head PGBP

Case Study No. 22 :- Residential Status + Income u/h Capital Gains + Agricultural Income
Mr. Zukaro, aged 42 years, a Singapore citizen, visits India for business purpose on a regular basis. He was in India
for the first time in the year 2018-19 for 270 days, in the year 2019-20 for 190 days, in the year 2020-21 for 145
days and in the year 2021-22 for 155 days. In the current financial year 2022-23, he along with his family had come
to India on 10th August, 2022 for a pleasure trip. His family returned to Singapore on 31st August, 2022, however
he stayed back to complete some business commitments and then returned to Singapore on 17th November, 2022.

 
108 Prepared by : CA Sanchit Grover
  Multiple Choice Questions
  for CA Inter Taxation For May 2023 Exams

Mr. Zukaro owns a manufacturing unit in Singapore. He basically comes to India for procurement of raw material.
He has appointed Mr. Manish, as a dependent agent in Mumbai, who procures raw material from India and then
exports it to Singapore to his manufacturing unit and then sells the finished product there. An income of ₹ 8,75,000
was received in Singapore out of this activity in the P.Y. 2022-23. He had purchased a residential property for ₹
17,50,000 in Indore in April 2018. On getting an attractive deal in November, 2022, he sold the property for ₹
26,25,000. He also paid brokerage @2% on sales consideration.
Mr. Zukaro had also purchased an agricultural land in India and leased it out to a tenant. The tenant shares a
portion of his agricultural income with Mr. Zukaro as a consideration for rent of land every year. The share in the
income from the land for the previous year 2022-23 was ₹ 6,50,000.
Cost inflation index (CII) for the Financial Year (F.Y.) 2018-19 is 280; F.Y. 2022-23: 331
Based on the facts of the case scenario given above, choose the most appropriate answer to the following
questions:-

Q1) What is the Residential Status of Mr. Zukaro for an income deemed to accrue or arise in
the assessment year 2023-24? India
(a) Resident and ordinarily resident (c) Yes, as business is controlled from India
(b) Resident but not ordinarily resident (d) No, as income is received outside India
(c) Non-resident Solution) Option (B) is correct
(d) Deemed resident but not ordinarily As per Sec 9, if any person has operations confined
resident to purchase of goods from India for the purpose of
Solution) Option (A) is correct export, it would not lead to business connection in
Since Mr. Zukaro is neither an Indian citizen nor a India. Accordingly, in the given case, ₹ 8,75,000 shall
person of Indian origin, 2 Basic condition shall also
nd
not be deemed as an income accrued in India.
be applicable on him. Since his stay in India during PY Q3) Would income arising from transfer of
22-23 is > 60 Days and aggregate stay in 4 preceding residential property in Indore is chargeable to
PYs is > 365 days, he shall be resident. Further his tax in India in the hands of Mr. Zukaro? If yes,
aggregate stay during 7 preceding PYs = 760 days and compute the amount of capital gains
he is clearly resident in PY 18-19 and PY 19-20 (since chargeable to tax.
his stay is more than 182 days in those PYs). (a) Yes, long term capital gain of ₹ 4,68,286
Accordingly, he shall be ROR is chargeable to tax, since income is
Q2) Assume for the purpose of answering this deemed to accrue or arise in India and
question only, that Mr. Zukaro is a non-resident hence taxable in his hands though he is
in India for the P.Y. 2022-23, would income of ₹ non-resident in India
8,75,000 earned though activity of procuring (b) Yes, long term capital gain of ₹ 5,20,786
raw material for manufacturing unit in is chargeable to tax, since he is resident
Singapore be taxable in India? in India
(a) Yes, since it is deemed to accrue or arise (c) Yes, long term capital gain of ₹ 4,68,286
in India through a business connection in is chargeable to tax, since he is resident
India in India
(b) No, as it is confined to purchase of goods (d) Yes, long term capital gain of ₹ 5,20,786
in India for further export and hence not is chargeable to tax, since income is
deemed to accrue or arise in India and
  
Prepared by : CA Sanchit Grover 109
   CHAPTER 18 : CASE SCENARIO BASED MCQS
hence taxable in his hands though he is Q5) What is the tax liability of Mr. Zukaro for A.Y.
non-resident in India 2023-24 assuming he does not opt to pay tax
Solution) Option (C) is correct under section 115BAC?
Full Value of Consideration 26,25,000 (a) ₹ 1,88,400
Less:- Brokerage @2% (52,500) (b) ₹ 3,78,200
Less:- Indexed COA (21,04,214) (c) ₹ 2,95,750
17,50,000 x 331/280 (d) ₹ 2,82,000
LTCG 4,68,286 Solution) Option (D) is correct
Income from sale of goods purchased 8,75,000
Q4) Would income earned from agricultural land
from India (although it is a foreign
given on lease is taxable in the hands of Mr.
income but since he is ROR it shall be
Zukaro?
taxable)
(a) No, such income is exempt, since it is
LTCG u/s 112 4,68,286
agricultural income
Rent from Agricultural land Exempt
(b) Yes, such income is taxable as income
u/s 10(1)
from house property, since land is given
Total Income (after rounding off) 13,43,290
on lease
Computation of tax liability
(c) Yes, such income is taxable as income
from other sources, since land is given on Tax on Normal income (by applying 1,77,500
lease scheme of partial integration)
(d) Yes, such income is taxable since he is Step 1:- Tax computed on 8,75,000 +
non-resident even though it is an 6,50,000 = 2,70,000
agricultural income. Step 2:- Tax computed on 6,50,000 +
Solution) Option (A) is correct 2,50,000 = 92,500
As per Sec 2(1A), any rent derived from an agricultural Tax computed on Special Income 93,657
land shall be treated as agricultural income provided (4,68,286 x 20%)
2 conditions are satisfied:- Tax liability after HEC of 4% (after 2,82,000
i) Such land should be situated in India rounding off)
ii) Such land should be used for agricultural
purposes

Case Study No. 23 :- Residential Status + Income u/s Salary + Chapter VI-A Deductions +
Computation of Tax Liability + Return of Income
Mr. Rajan, aged 62 years, an Indian citizen, resides in Delhi. His wife Sheetal and daughter Riya also reside with him.
Riya, aged 16 years, is studying in 12th Standard in DAV school at New Delhi. Mr. Rajan left for employment to the
United States of America on 15th September, 2022 but his family did not accompany him. He returned to India on
25th March 2023. Mr. Rajan had gone outside India for the first time in his life. During April, 2022 to September,
2022, he was working with a multinational company in Delhi. He earned salary of ₹ 14,00,000 from his job in India.
He paid Tuition Fee of ₹ 1,80,000 for Riya’s education in DAV school.

 
110 Prepared by : CA Sanchit Grover
  Multiple Choice Questions
  for CA Inter Taxation For May 2023 Exams

Apart from that, Mr. Rajan also earned professional income of ₹ 60,00,000 (Gross Receipts – ₹ 90 lakhs) from India.
During the year, he also earned interest from his Indian savings bank account to the tune of ₹ 12,000 and interest
from fixed deposits with nationalized banks of ₹ 45,000. Mr. Rajan also earned a salary income equivalent to ₹
6,00,000 from USA for his job, on which no tax is paid or payable in USA, which was deposited in his bank account
in USA and later on remitted to India. Mr. Rajan decides not to opt to pay tax under section 115BAC.
Based on the facts of the case scenario given above, choose the most appropriate answer to the following
questions:-

Q1) What is the residential status of Mr. Rajan for received outside India, it wont be
the previous year 2022-23? taxable in hands of RNOR)
(a) Resident and ordinarily in India Gross Salary 14,00,000
(b) Resident but not ordinarily resident in Less:- Deduction u/s 16 (50,000)
India Income u/h Salary 13,50,000
(c) Non-resident in India
Q3) How much deduction is available under Chapter
(d) Deemed resident but not ordinarily
VI-A from the Gross Total Income of Mr. Rajan?
resident in India
(a) ₹ 2,30,000
Solution) Option (D) is correct
(b) ₹ 1,95,000
Since Mr. Rajan is an Indian citizen who is leaving
(c) ₹ 1,60,000
India for the purpose of employment, hence 2nd basic
(d) ₹ 2,00,000
condition shall not be applicable to him. Since his stay
Solution) Option (D) is correct
during PY 22-23 is 175 days (i.e. < 182 days), he shall
Deduction u/s 80C in respect of 1,50,000
be non-resident as per Sec 6(1). However, Since he is
Tuition fees paid for daughter
Indian citizen having Total Income (excluding Income
(allowed maximum to the extent of
from foreign sources) > 15L and he is not subjected to
₹ 1.5L)
tax in any other country (Question clearly specifies that
Deduction u/s 80TTB in respect of 50,000
no tax is payable in US), hence he shall be Deemed
Interest on Saving Bank A/C and
RNOR u/s 6(1A)
Interest on fixed deposits (allowed
Q2) What would be the income chargeable to tax
maximum to the extent of ₹ 50,000)
under the head “Salaries” in the hands of Mr.
Total Deductions under Chapter VI- 2,00,000
Rajan in India for F.Y. 2022-23?
A
(a) ₹ 20,00,000
(b) ₹ 19,50,000 Q4) What shall be the tax liability of Mr. Rajan for
(c) ₹ 13,50,000 the A.Y. 2023-24?
(d) ₹ 19,60,000 (a) ₹ 22,69,810
Solution) Option (C) is correct (b) ₹ 22,58,940
Salary earned in India upto 14,00,000 (c) ₹ 22,56,080
September 2022 (Since services (d) ₹ 22,72,670
have been rendered in India, this Solution) Option (C) is correct
Salary is deemed to accrue in India) Income u/h Salary 13,50,000
Salary earned in US (Since this Nil Income u/h PGBP from India 60,00,000
salary is accrued as well as Interest on Saving Bank Account 12,000

  
Prepared by : CA Sanchit Grover 111
   CHAPTER 18 : CASE SCENARIO BASED MCQS
Interest on FD 45,000 Q5) What would be the due date for filing income-
Gross Total Income 74,07,000 tax return of Mr. Rajan for the P.Y. 2022-23?
Less:- Deductions under Chapter VI- (2,00,000) (a) 31st July, 2023
A (b) 31st October, 2023
Total Income 72,07,000 (c) 30th November, 2023
Tax computed (Nil + 10,000 + 19,72,100 (d) 31st March, 2024
1,00,000 + 18,62,100) Solution) Option (B) is correct
Add:- Surcharge of 10% 1,97,210 Since receipts from profession is more than ₹ 50L,
Tax plus surcharge + 4% HEC 22,56,080 hence Audit u/s 44AB shall be applicable and
accordingly, the due date shall be 31st Ocotber and not
31st July

Case Study No. 24 :- Re


Mr. A (aged 52 years), is a CEO of XYZ Enterprise Limited. During the previous year 2022-23, he earned salary of ₹
1,65,00,000 and long-term capital gain on sale of listed equity shares (STT paid) amounting to ₹ 1,06,500. He earned
interest of ₹ 4,82,778 on saving bank account.
Further, he has provided the following other information for filing his return of income:
He does not receive house rent allowance from his employer. Mr. A took a loan from State Bank of India on 27th
October 2020 for repairing his house (self-occupied) at Delhi and paid interest on such borrowings of ₹ 80,000 and
₹ 1,50,000 towards principal amount during the previous year 2022-23.
Mr. A has made the following payments towards medical insurance premium for health policies taken for his family
members:
- Medical premium for his brother: ₹ 13,500 (by cheque)
- Medical premium for his parents: ₹ 17,670 (by cheque)
- Medical premium for self and his wife: ₹ 21,000 (by cheque).
He also incurred ₹ 6,400 towards preventive health check-up of his wife in cash. He deposited ₹ 1,00,000 towards
PPF. He also deposited ₹ 50,000 and ₹ 2,50,000 towards Tier I and Tier II NPS A/c, respectively.
He has paid ₹ 5,30,000 as advance tax. His employer has deducted tax at source of ₹ 51,89,000. He is of the opinion
that the balance amount of tax, if any, he will pay on 27th July 2023 (i.e. before the due date for filing of return of
income).
Mr. A does not want to opt for section 115BAC.
Based on the facts of the case scenario given above, choose the most appropriate answer to the following
questions:- (RTP Nov 2020)

Q1) What would be the amount of deduction Solution) Option (D) is correct
available to Mr. A under Chapter VI-A for the Deduction u/s 80C in respect of Nil
assessment year 2023-24? Principal repayment of loan meant for
(a) ₹ 2,04,070 repairing HP (Not allowed)
(b) ₹ 2,42,670 Deduction u/s 80C in respect of PPF 1,00,000
(c) ₹ 2,52,670 Deduction u/s 80C in respect of Nil
(d) ₹ 2,02,670 investment in Tier-II account (Not

 
112 Prepared by : CA Sanchit Grover
  Multiple Choice Questions
  for CA Inter Taxation For May 2023 Exams

allowed since Mr. A is not Central Solution) Option (C) is correct


Government employee) Income u/h Salary (after allowed 1,64,50,000
Deduction u/s 80D in respect of Self + 25,000 Standard deduction u/s 16)
Spouse (21000 + 6,400) – maximum LTCG u/s 112A 1,06,500
deduction shall be 25,000 only Interest on Saving A/c 4,82,778
Deduction u/s 80D in respect of 17,670 Loss u/h House Property – Since HP (30,000)
parents is self-occupied and loan is in
Deduction u/s 80CCD(1B) in respect of 50,000 relation to repair of HP, maximum
self contribution to NPS deduction allowed will be ₹ 30,000
Deduction u/s 80TTA in respect of 10,000 Gross Total Income 1,70,09,278
Interest on saving bank account Less:- Deductions under Chapter VI- (2,02,670)
Total Deductions under Chapter VI-A 2,02,670 A
Total Income (after rounding off) 1,68,06,610
Q2) Assume that, for the purpose of answering this
Tax computed on normal income of 48,22,533
question alone, that Mr. A pays rent of ₹ 65,000
₹ 1,57,34,550
per month for his rented house at Mumbai to
Mr. C, a resident individual, is Mr. A liable to (Nil + 12,500 + 1,00,000 + 47,10,033)
deduct TDS on such rent. If so, what would be Tax on LTCG u/s 112A 650
the rate and amount of TDS? 10% of (1,06,500 – 1,00,000)
(a) Yes, Mr. A is liable to deduct TDS @ 3.75% Tax before surcharge 48,23,183
amounting to ₹ 2,438 every month i.e., at Add:- Surcharge @15% 7,23,477
the time of payment of such rent Tax after HEC 57,68,527
(b) Yes, Mr. A is liable to deduct TDS @5% Less:- Advance Tax (5,30,000)
amounting to ₹ 3,250 every month i.e., at Less:- TDS u/s 192 (51,89,000)
the time of payment of such rent Net Tax payable 49,526
(c) Yes, Mr. A is liable to deduct TDS @5% After rounding off 49,530
amounting to ₹ 39,000 in the month of Q4) What would be the amount of interest
March 2022 chargeable under section 234B on account of
(d) No, Mr. A is not liable to deduct TDS, since short payment of advance tax?
he is a salaried person (a) ₹ 1,980
Solution) Option (C) is correct (b) Nil
Since Mr. A doesn’t carry out any business or (c) ₹ 3,130
profession, he is not liable to deduct any TDS u/s 194- (d) ₹ 2,410
I. Further, since the amount of rent exceeds ₹ 50,000 Solution) Option (B) is correct
p.m., he shall be liable to deduct TDS u/s 194IB. Hence Total Tax liability 57,68,527
tax to be deducted = 5% of ₹ 65,000 = ₹ 3,250
Less:- TDS already deducted u/s 192 (51,89,000)
Q3) What would be the amount of net tax payable
Advance Tax to be deposited 5,79,527
for the assessment year 2023-24 in the hands
90% of Advance tax liability 5,21,574
of Mr. A?
Advance tax actually deposited 5,30,000
(a) ₹ 78,230
Since Advance tax deposited is more than 90% of
(b) ₹ 60,290
Amount that should have been deposited, no
(c) ₹ 49,530
interest u/s 234B shall be chargeable in this case
(d) ₹ 67,470

  
Prepared by : CA Sanchit Grover 113
   CHAPTER 18 : CASE SCENARIO BASED MCQS
Case Study No. 25 :- Income u/h Salary + Chapter VI-A Deductions + Sec 115BAC + TDS +
Income u/h Capital Gains

Mr. Sarthak, aged 38 years, working in Nobita Pvt. Limited as Senior Manager- Finance. His yearly pay slip for the
financial year 2022-23 is as follows:
Earnings ₹ Deductions ₹
Basic Pay 6,34,068 Employees’ contribution to Provident 1,14,132
Fund
Dearness Allowance 1,26,814 Professional Tax 2,400
HRA 3,17,040 Income Tax 2,32,830
Transport Allowance 19,200 Net Pay 13,03,848
Personal Allowance 5,09,088
Children Education allowance for 12,000
two children
Medical Allowance 15,000
Bonus 20,000
Total Earnings 16,53,210 16,53,210
(i) His employer also contributes equivalent amount of contribution towards provident fund.
(ii) Dearness allowance forms part of retirement benefits.
(iii) He has intimated to his company that he would opt for 115BAC for the A.Y. 2023-24. Consequently, he has
not submitted any investment proof to company.
(iv) He has paid ₹ 55,212 towards mediclaim premium for his parents (aged above 65 years) by account payee
cheque.
(v) He has purchased a house of ₹ 38,00,000 during the year 2014 and taken a loan of ₹ 28,00,000 from HDFC
to purchase this house. He is paying EMI of ₹ 22,835. Possession of house received on 01/04/2022. He himself
is occupying this house. Total principal and interest paid for full year is ₹ 55,037 and ₹ 2,18,983, respectively,
as per interest certificate received from bank for F.Y. 2022-23.
(vi) He has 3 children, studying in Sandalwood International School. The following are the components of school
fees paid for the Academic Session 2022-23:
School Fees Component Child 1 Child 2 Child 3 Total
Tuition Fees 30,000 37,000 40,000 1,07,000
Admission Fees 20,000 - - 20,000
Books, Stationery and uniform 8,000 12,000 15,000 35,000
Infrastructure Fund 25,000 30,000 35,000 90,000
Commute Cost 8,000 8,000 8,000 24,000
Acitivity Fees 6,000 7,000 8,000 21,000
Total Fees 97,000 94,000 1,06,000 2,97,000
(vii) He has invested ₹ 5000 in HDFC ULIP and taken a LIC policy for his wife for ₹ 10,000.
(viii) He has invested ₹ 12,500 and ₹ 25,000 towards NPS Tier I A/c and Tier II A/c, respectively.
(ix) He has also donated ₹ 50,000 in PM CARES fund.

 
114 Prepared by : CA Sanchit Grover
  Multiple Choice Questions
  for CA Inter Taxation For May 2023 Exams

(x) He has invested ₹ 40,000 in listed equity shares of Shaktimaan Power Solution Limited on 01/03/2022 at ₹
200 per share and sells 100 shares at ₹ 350 per share on 01/11/2022. STT is paid both at the time of sale and
purchase of these shares.
Based on the facts of the case scenario given above, choose the most appropriate answer to the following
questions:- (RTP May 2021)

Q1) What would be the amount of income (c) No, the correct amount of tax to be
chargeable to tax under the head “Salaries” in deducted at source is ₹ 2,42,800.
the hands of Mr. Sarthak for the A.Y. 2023-24? (d) No, the correct amount of tax to be
(a) ₹ 16,53,210 deducted at source is ₹ 2,41,300.
(b) ₹ 16,21,236 Solution) Option (B) is correct
(c) ₹ 16,76,036 Income u/h Salary 16,76,036
(d) ₹ 16,71,236 Loss u/h HP Nil
Solution) Option (C) is correct (No deduction shall be allowed u/s
Basic Pay 6,34,068 24 in respect of Interest, if Sec
DA (Fully taxable) 1,26,814 115BAC is opted)
HRA [No exemption allowed u/s 3,17,040 Gross Total Income 16,76,036
10(13A) if Sec 115BAC opted] Less:- Deductions under Chapter VI- Not allowed
Transport Allowance (It is assumed 19,200 A
that Mr. Sarthak is not Total Income (after rounding off) 16,76,040
handicapped, hence no exemption) Tax Liability on normal income ₹ 2,40,312
Personal Allowance 5,09,088 16,76,040
Children allowance [Exemption u/d 12,000 (Nil + 12,500 + 25,000 + 37,500 +
10(14) not available since Sec 50,000 + 62,500 + 52,812)
115BAC opted] Add:- HEC @4% 9,612
Medical Allowance 15,000 Total tax liability (after rounding off) 2,49,920
Bonus 20,000
Q3) What would be the total income (without
Employer’s Contribution to PF 22,826
rounding off) of Mr. Sarthak for the A.Y. 2023-
1,14,132
24, assume that he does not opt for section
Less:- Exempt upto 12% of Salary
115BAC?
91,306
(a) ₹ 11,73,736
Gross Salary 16,76,036
(b) ₹ 11,76,699
Less:- Deductions u/s 16 Not allowed (c) ₹ 11,61,699
Income u/h Salary 16,76,036 (d) ₹ 11,58,736
Q2) Whether the tax deducted at source by Nobita Solution) Option (A) is correct
Pvt Ltd. on the salary paid to Mr. Sarthak based Income u/h Salary if Sec 115BAC is 16,76,036
on the intimation submitted by him, is correct? opted
(a) Yes, the amount of ₹ 2,32,830 deducted Less:- Exemption u/s 10(14) in respect (2,400)
as tax at source is correct. of Children education allowance (100 x
(b) No, the correct amount of tax to be 12 x 2)
deducted at source is ₹ 2,49,920.
  
Prepared by : CA Sanchit Grover 115
   CHAPTER 18 : CASE SCENARIO BASED MCQS
Less:- Exemption u/s 10(13A) in respect Nil (Max deduction u/s 80C cant be more
of HRA – Since he is staying in his own thn 1.5L)
house, no exemption shall be available Less:- Deduction u/s 80D in respect of (50,000)
in respect of HRA Medical insurance premium (Max
Less:- Standard Deduction u/s 16 (50,000) allowed will be ₹ 50,000 since parents
Less:- Deduction u/s 16 (2,400) are senjor citizen)
Income u/h Salary if Sec 115BAC not 16,21,236 Less:- Deduction u/s 80CC(1B) in (12,500)
opted respect of Contribution to Tier-I of NPS
Loss u/h HP (2,00,000) Less:- Deduction u/s 80G in respect of (50,000)
(Since purchase of HP was done in Donation to PM Cares fund
2014 itself i.e. PY in which loan was Total Income (after rounding off) 11,73,736
taken, hence maximum limit of
Q4) What would be tax liability of Mr. Sarthak for the
deduction u/s 24 shall be ₹ 2L)
A.Y. 2023-24, if he does not opt for section
STCG u/s 111A 15,000
115BAC?
(35,000 – 20,000)
(a) ₹ 1,66,530
Gross Total Income 14,36,236
(b) ₹ 1,68,870
Less:- Deduction u/s 80C in respect of (1,14,132) (c) ₹ 1,71,210
employees contribution to RPF (d) ₹ 1,67,450
Less:- Deduction u/s 80C in respect of Nil Solution) Option (B) is correct
Contribution to Tier-II of NPS – not Tax computed on normal income of ₹ 1,60,121
allowed since Mr. Sarthak is not central 11,58,736
government employee (Nil + 12,500 + 1,00,000 + 47,621)
Less:- Deduction u/s 80C in respect of (35,868) Tax computed on STCG u/s 111A 2,250
principal portion of housing loan (15,000 x 15%)
Tax after HEC 1,68,870

Case Study No. 26 :- Residential Status + Income u/h House Property + Income u/h Capital
Gains + Deductions under Chapter VI-A + Sec 115BAC
Mr. Animesh, an Indian citizen, aged 61 years, has set-up his business in Canada and is residing in Canada since
2010. He owns a house property in Canada, half of which is used by him for his residence and half is given on rent
(converted into INR is ₹ 12,00,000 p.a.).
He purchased a flat in Delhi on 13.10.2020 for ₹ 42,00,000. The stamp duty value of the flat was ₹ 35,00,000. He has
taken a loan from Canara Bank in India of ₹ 34,00,000 for purchase of this flat. The interest on such loan for the F.Y.
2022-23 was ₹ 3,14,000 and principal repayment was ₹ 80,000. Mr. Animesh has given this flat on monthly rent of
₹ 32,500 since April, 2022. The annual property tax of Delhi flat is ₹ 40,000 which is paid by Mr. Animesh, whenever
he comes to India to meet his parents. Mr. Animesh visited India for 124 days during the previous year 2022-23.
Before that he visited India in total for 366 days during the period 1.4.2018 to 31.3.2022.
He had a house in Ranchi which was sold in May 2019. In respect of this house, he received arrears of rent of ₹
2,96,000 in February 2023 (not taxed earlier).
 
116 Prepared by : CA Sanchit Grover
  Multiple Choice Questions
  for CA Inter Taxation For May 2023 Exams

He also derived some other incomes during the F.Y. 2022-23 which are as follows:
(i) Profit from business in Canada ₹ 2,75,000
(ii) Interest on bonds of a Canadian Co. ₹ 6,20,000 out of which 50% was received in India.
(iii) Income from Apple Orchid in Nepal given on contract and the yearly contract fee of ₹ 5,00,000 for F.Y. 2022-
23, was received by Animesh in Nepal.
Mr. Animesh has sold 10,000 listed shares @ ₹ 480 per share of A Ltd., an Indian company, on 15.9.2022, which he
acquired on 05-04-2017 @ ₹ 100 per share. STT was paid both at the time of acquisition as well as at the time of
transfer of such shares.
On 31-01-2018, the shares of A Ltd. were traded on a recognized stock exchange as under:
Highest price - ₹ 300 per share
Average price - ₹ 290 per share
Lowest price - ₹ 280 per share
Based on the facts of the case scenario given above, choose the most appropriate answer to the following
questions:- (RTP Dec 2021)

Q1) What would be the residential status of Mr. (Since he is RNOR, annual value of
Animesh for the A.Y. 2023-24? HP in Canada will not be taxable in
(a) Resident and ordinarily resident in India his hands)
(b) Resident but not ordinarily resident in GAV of HP in Delhi (32,500 x 12) 3,90,000
India Less:- Municipal Taxes (40,000)
(c) Non-resident NAV 3,50,000
(d) Deemed resident u/s 6(1A) Less:- Standard Deduction (30%) (1,05,000)
Solution) Option (B) is correct Less:- Interest on Borrowed capital (3,14,000)
Since Total Income (excluding Income from foreign Loss from HP in Delhi (69,000)
sources) is more than ₹ 15L (It will be computed in Q2 Arrears of Rent in respect of Ranchi 2,07,200
and Q3) and Mr. Animesh is an Indian Citizen who house taxable u/s 25A
usually resides outside India for business and only (2,96,0000 minus 30%)
comes on a visit to India, modified 2nd Basic condition Income u/h House Property 1,38,200
is applicable here. His Stay in PY 22-23 is more than
120 days and aggregate stay in 4 preceding PYs is Q3) What amount of capital gain would arise in the
more than 366 days, hence he shall be RNOR (whether hands of Mr. Animesh on transfer of shares of
or not he satisifies additional conditions) A Ltd ?
Q2) What would be amount of income taxable (a) ₹ 18,00,000
under the head “Income from house property” in (b) ₹ 19,00,000
the hands of Mr. Animesh for the A.Y. 2022-23? (c) ₹ 20,00,000
(a) ₹ 2,52,200 (d) ₹ 38,00,000
(b) ₹ 1,38,200 Solution) Option (A) is correct

(c) ₹ 9,78,200 Full Value of Consideration of 48,00,000


(d) ₹ 10,92,200 shares (Since shares are of Indian
Solution) Option (B) is correct Company, capital gain on their
Rental Income from HP in Canada Not taxable transfer shall be deemed to accrue

  
Prepared by : CA Sanchit Grover 117
   CHAPTER 18 : CASE SCENARIO BASED MCQS
in India and hence taxable in hands Les:- Deduction u/s 80C in respect (80,000)
of RNOR also) of Principal payment of housing
Less:- COA as per Special method (30,00,000) loan
u/s 55(2)(ac) – WN-1 Less:- Deduction u/s 80EEA in Nil
LTCG u/s 112A 18,00,000 respect of Interest (Since entire
Calculation of COA as per Sec 55(2)(ac) interest has already been allowed
i) Full value of Consideration 48,00,000 u/s 24, no deduction shall be
ii) FMV as on 31-01-2018 30,00,000 available u/s 80EEA)
(Highest price) Total Income 21,68,200
iii) Lower of (i) and (ii) 30,00,000 Q5) What would be the tax liability (computed in the
iv) COA as per normal provisions 10,00,000 manner so as to minimise his tax liability) of Mr.
v) COA as per Sec 55(2)(ac) 30,00,000 Animesh for the A.Y. 2023-24?
[higher of (iii) and (iv)] (a) ₹ 1,82,950
Q4) What would be total income of Mr. Animesh for (b) ₹ 1,87,110
the A.Y. 2022-23, if he does not opt to pay tax (c) ₹ 1,80,350
u/s 115BAC? (d) ₹ 1,84,510
(a) ₹ 22,82,200 Solution) Option (C) is correct
(b) ₹ 22,68,200 Calculation of tax liability if Sec 115BAC not
(c) ₹ 22,48,200 opted
(d) ₹ 21,68,200 Tax computed on normal income of ₹ 3,410
Solution) Option (D) is correct 3,68,200
Income u/h House property 1,38,200 (₹ 68,200 x 5% - since exemption limit
LTCG u/s 112A 18,00,000 in this case shall be ₹ 3L)
Profit from business in Canada Nil Tax computed on LTCG u/s 112A 1,70,000
(Since it is accrued as well as (₹ 17L x 10%]
received outside India, it shall not Tax including HEC 1,80,346
be taxable in hands of RNOR) Calculation of tax liability if Sec 115BAC opted
Interest on Canada Gov Bonds 3,10,000 Total Income if Sec 115BAC opted 22,48,200
(only that portion which is received Note:- Deduction u/s 80C wont be
in India, shall be taxable in hands allowed if Sec 115BAC opted
of RNOR) Tax computed on normal income of 9,910
Income from agriculture in Nepal Not taxable 4,48,200 (1,98,200 x 5%)
and also received there Tax computed on LTCG u/s 112A 1,70,000
GTI 22,48,200 Tax after HEC of 4% 1,87,106

Case Study No. 27 :- Income u/h PGBP + Sec 115BAC


Mr. Kashyap, a manufacturer, has disclosed a net profit of ₹ 40 lakhs for the year ended 31st March, 2023. He
claimed depreciation of ₹ 12,20,000 in his books of account. Expenditure in profit and loss account includes interest

 
118 Prepared by : CA Sanchit Grover
  Multiple Choice Questions
  for CA Inter Taxation For May 2023 Exams

payable to Mr. Raj, a resident, without deduction of tax at source ₹ 1,50,000. Such tax was, however, deducted on
15.4.2023 and remitted on 17.5.2023.
Mr. Kashyap is engaged in in-house scientific research and development. He incurred expenditure of ₹ 1,50,000 on
purchase of research equipments and ₹ 1,00,000 as remuneration paid to scientists. The said sums are also debited
in the profit and loss account.
Mr. Kashyap purchased a new plant and machinery for ₹ 45,00,000 on 2nd August, 2022 and put the same to use
on 1st November, 2022. For this purpose, he borrowed ₹ 25,00,000 on 1st August, 2022 and paid interest@10% p.a.
which is debited in profit and loss account. Mr. Kashyap also purchased a motor car for ₹ 8,00,000 on 2nd October,
2019, which was put to use on the same date. Written down value of block of plant and machinery (15%) as on 1st
April, 2022 is ₹ 95,00,000. Turnover for the P.Y. 2021-22 and P.Y. 2022-23 is ₹ 2.5 crores and ₹ 3 crores, respectively.
Based on the above information, choose the most appropriate answer to the following Multiple Choice
Questions –

Q1) What would be the depreciation allowable u/s Depreciation computed on Motor vehicle purchased
32 in respect of block of plant and machinery on 23rd August 2019 to 31st March 2020 –
(15%) and motor car for A.Y.2023-24? Assume Depreciation rate will be 30%
that motor car is the only asset in the block. Actual Cost of Car 8,00,000
(a) ₹ 22,23,438 and ₹ 83,320, respectively Less:- Normal Depreciation for PY (2,40,000)
(b) ₹ 17,67,188 and ₹ 73,695, respectively 19-20 @30%
(c) ₹ 22,12,500 and ₹ 73,695, respectively Note:- Additional depreciation is not
(d) ₹ 22,23,438 and ₹ 2,40,000, respectively allowed on cars
Solution) Option (a) is correct WDV of Block as on 01-04-2020 5,60,000
Depreciation u/s 32 computed on P&M (15%) Less:- Normal Depreciation for PY (1,68,000)
Purchase Price 45,00,000 20-21 @30%
Add:- Interest from date of loan till 62,500 WDV of Block as on 01-04-2021 3,92,000
machine is put to use Less:- Normal Depreciation for PY (1,17,600)
(25,00,000 x 10% x 3/12) 21-22 @30%
Actual Cost of P&M as per Sec 43 45,62,500 WDV of Block as on 01-04-2022 2,74,400
Normal Depreciation @7.5% on 3,42,188 Less:- Normal Depreciation for PY (83,320)
above (Since asset is put to use for 22-23 @30%
< 180 days during PY22-23)
Q2) What is the amount of disallowance, if any,
Normal Depreciation @15% on 14,25,000
attracted for non-deduction of tax at source on
Opening WDV of ₹ 95,00,000
interest payable to Mr. Raj during the P.Y.2022-
Additional Depreciation @10% on ₹ 4,56,250
23?
45,62,500
(a) Nil, since the tax was deducted and
Note:- Additional Depreciation
deposited on or before the due date of
allowed since it is a case of
filing of return of income
manufacturer
(b) ₹ 30,000
Total Depreciation on P&M 22,23,438
(c) ₹ 45,000
(d) ₹ 1,50,000

  
Prepared by : CA Sanchit Grover 119
   CHAPTER 18 : CASE SCENARIO BASED MCQS
Solution) Option (a) is correct added to ‘Actual Cost’ but
Since payment has been made to resident without wrongly debited to P&L
deduction of TDS before end of PY, hence disallowance Income u/h PGBP 30,20,742
u/s 40(ia) shall be 30% of ₹ 1,50,000
Q4) What would be the income chargeable under
Q3) What would be the income under the head
the head “Profits and gains of business and
“Profits and gains of business and profession”
profession” of Mr. Kashyap for A.Y.2023-24, if he
of Mr. Kashyap for A.Y.2023-24 under the
opts for section 115BAC?
normal provisions of the Act?
(a) ₹ 35,84,492
(a) ₹ 30,20,742
(b) ₹ 34,14,492
(b) ₹ 29,58,242
(c) ₹ 34,76,992
(c) ₹ 28,01,562
(d) ₹ 35,39,492
(d) ₹ 34,11,112
Solution) Option (c) is correct
Solution) Option (a) is correct
Income u/h PGBP as per normal 30,20,742
Profit as per P&L A/c 40,00,000
provisions
Add:- Depreciation already 12,20,000
Add:- Additional depreciation on 4,56,250
debited to P&L
P&M not allowable if Sec
Less:- Depreciation as per Sec 32 (23,06,758)
115BAC opted
(22,23,438 + 83,320)
Income u/h PGBP if Sec 115BAC 34,76,992
Add:- Disallowance u/s 40a(ia) 45,000
opted
Add:- Interest for the period 1st 62,500
Aug 22 to 31st Oct 22, being

Case Study No. 28 :- Income u/h House Property + Chapter VI-A Deductions + Clubbing of
Income
Mr. Akash had bought a residential house worth ₹ 2.5 crores at South Extension, Delhi in 2020 and let out the house
on rent to Mr. Riyaz. The property was funded through loan from PNB. The interest due for F.Y. 2022-23 to PNB is ₹
25 lakhs, out of which he paid only ₹ 20 lakhs during the year. Mr. Akash then took a loan of ₹ 1.5 crores from SBI
on 1.7.2022 for construction of first floor in that house for self-occupation. The construction is in progress as on
31.3.2023. Mr. Akash started repaying EMIs due to SBI. During the P.Y. 2022-23, he repaid principal amount of ₹ 25
lakhs and ₹ 5 lakhs to PNB and SBI, respectively. He also paid interest of ₹ 8 lakhs to SBI out of ₹ 10 lakhs, being
interest due for the period from 1.7.2022 to 31.3.2023.
Mr. Akash owns another house in Haryana. He transferred that house to his minor daughter Miss Sia on her birthday
as her birthday gift. Miss Sia gave the said house to the local Panchayat from September, 2022 at a rent of ₹ 5,000
per month. Mrs. Akash’s total income for A.Y.2023-24 is higher than that of Mr. Akash. This is the first year when
Miss Sia has any source of income.
Mr. Akash bought electric vehicle worth ₹ 50 lakhs on loan from BSM Bank which it sanctioned on 1.4.2022. BSM
Bank charged interest of ₹ 7 lakhs on electric vehicle for the P.Y. 2022-23. Mr. Akash has also taken loan from ABC
Bank for his daughter’s higher education. He paid ₹ 50,000 as interest to ABC Bank. He also paid mediclaim of ₹
20,000 to New India Assurance Scheme for insuring his health via cheque

 
120 Prepared by : CA Sanchit Grover
  Multiple Choice Questions
  for CA Inter Taxation For May 2023 Exams

Mrs. Akash owns a shop of 200 square feet area in Gurgaon. She rented it to Mr. Vishal from October, 2022 at ₹
60,000 per month, who gave her an interest-free deposit of ₹ 1,50,000.
From the information given above, choose the most appropriate answer to the following questions –

Q1) What is the amount of interest allowable as Deduction u/s 80E for interest on 50,000
deduction u/s 24(b) to Mr. Akash for A.Y. 2023- education loan
24? Deduction u/s 80D in respect 20,000
(a) ₹ 2 lakhs payment of Mediclaim via cheque
(b) ₹ 25 lakhs Total Permissible deductions under 3,70,000
(c) ₹ 28 lakhs Chapter VI-A
(d) ₹ 35 lakhs
Q3) Is notional interest on interest free deposit
Solution) Option (b) is correct
received in respect of shop let out on rent
First Floor of HP is let out and hence there is no limit
chargeable to income-tax? If so, under which
on the amount of interest that can be allowed as
head of income would the same be taxable?
deduction u/s 24(b). Hence entire ₹ 25L shall be
(a) No, it is not chargeable to tax
allowed (it is allowed on accrual basis). Further, since
(b) Yes, it is chargeable to tax as profits and
2nd Floor has not yet been constructed and hence no
gains from business, since a commercial
deduction shall be allowable in respect of interest
property has been let out.
payable to SBI
(c) Yes, it is chargeable to tax as “Income
Q2) What is the amount of deduction permissible to
from Other Sources”, being the residuary
Mr. Akash under Chapter VI-A of Income-tax Act,
head of income.
1961 for A.Y. 2022-23?
(d) Yes, it is chargeable to tax as “Income
(a) ₹ 1,70,000
from house property”, since section 22
(b) ₹ 2,20,000
does not distinguish between a
(c) ₹ 3,70,000
residential house property and
(d) ₹ 9,20,000
commercial house property.
Solution) Option (c) is correct
Solution) Option (a) is correct
Deduction u/s 80C in respect of 1,50,000
CBDT has clarified that in case of non-refundable
principal repayment of housing loan
deposit, amount shall be chargeable to tax on
(maximum permissible deduction is
proportionate basis. However, in case of refundable
₹ 1.5L)
deposit, neither the amount of deposit nor any
Deduction u/s 80EEA in respect of Nil
notional interest computed on it, shall be taxable
Interest on housing loan (Deduction
under the head house property
is not allowed since value of the HP
Q4) In whose hands would Sia’s rental income from
is more than 45L)
house property at Haryana be taxable and how
much income would be taxable?
Deduction u/s 80EEB in respect of 1,50,000
(a) I n Sia’s hands; ₹ 24,500
Interest of loan taken for electric
(b) In Mr. Akash’s hands; ₹ 24,500
vehicle (maximum permissible
(c) In Mrs. Akash’s hands; ₹ 23,000
deduction is ₹ 1.5L)

  
Prepared by : CA Sanchit Grover 121
   CHAPTER 18 : CASE SCENARIO BASED MCQS
(d) It would change every year depending on shall apply and Mr. Akash will be considered deemed
the parent whose income is higher in that owner. It is to be noted that since provisions of Sec
year 64(1A) are not applicable here, hence Exemption u/s
Solution) Option (b) is correct 10(32) of ₹ 1,500 shall not apply. Income u/h House
Since HP in Haryana has been transferred to minor property will be ₹ 5,000 x 7 months minus 30%
child (other than minor married daughter) without Standard Deduction u/s 24(a)
consideration, provisions of Deemed Owner u/s 27

Case Study No. 29 :- TDS + Return of Income + Income u/h Capital gains + Gift Provisions +
Clubbing of Income
Mr. Alishaan is engaged in the business of clothes trading since 2016 and Mrs. Alishaan is a house wife. Their minor
daughter’s marriage is fixed in October, 2022. Mr. Alishaan planned destination wedding in Udaipur for his minor
daughter. For the wedding, he withdrew ₹ 40,00,000 cash in the month of August, 2022 and ₹ 65,00,000 cash in the
month of September, 2022 from Aapka Paisa Bank.
He booked 30 rooms for 5 days for the accommodation of his relatives in Raho Hotel and paid ₹ 40,000 in cash as
advance and balance by account payee cheque. He took the catering services of Daana Caterers, a sole proprietor,
for the wedding for which he paid ₹ 10,20,000 on 15.9.2022. On her wedding, he gifted his daughter a house property,
purchased from RK Builders on 10.9.2022 by account payee cheque for ₹ 15,00,000. The stamp duty value of the
property on 10.9.2022 is ₹ 16,00,000 and on the date of transfer to minor daughter is ₹ 20,00,000.
Mr. Alishaan paid ₹ 45,000 in cash and balance in cheque to travel agent for the return ticket of some of his relatives
to US. He regularly files his return of income on time but he failed to file his return of income of P.Y. 2020-21. His
daughter let out the house property received from him at a monthly rent of ₹ 40,000 from 01.11.2022.
Based on the above information, choose the most appropriate option of the following Multiple Choice Questions
(MCQs):- (MTP May 2021)

Q1) The amount of tax to be deducted by Aapka Q2) The amount of tax to be deducted by Mr.
Paisa Bank on cash withdrawals by Mr. Alishaan on payment made to Daana Caterers
Alishaan is – is –
(a) ₹ 10,000 (a) ₹ 1,200
(b) ₹ 25,000 (b) ₹ 900
(c) ₹ 1,85,000 (c) ₹ 150
(d) ₹ 4,25,000 (d) Nil
Solution) Option (a) is correct Solution) Option (d) is correct
In the given case, Mr. Alishaan has not filed return Catering is covered under the definition of ‘work’ as
only for PY 20-21 bu the has filed for PY 2019-20, PY per Sec 194C. However, in the given case, TDS u/s
18-19. Hence he shall not be covered by Proviso ro Sec 194C shall not apply since the payment for catering
194N. Accordingly applicable threshold limit will be ₹ has been made for personal purposes and not
1 Crore. and TDS would be 2% of [(40L + 65L) – 1 Cr] business purposes.
= ₹ 10,000 Q3) For which of the following transactions, Mr.
Alishaan is required to quote his PAN?

 
122 Prepared by : CA Sanchit Grover
  Multiple Choice Questions
  for CA Inter Taxation For May 2023 Exams

(a) Purchase of immovable property from RK builder and nothing is taxable u/s
Builders 56(2)(x) in hands of Mr. Alishaan.
(b) Payment to Raho Hotel Minor Since daughter is receiving gift from
(c) Payment to Travel agent Daughter parents (who are covered in the
(d) All of the above definition of ‘relative’) hence nothing
Solution) Option (a) is correct is taxable u/s 56(2)(x) in her hands.
No PAN is required to be quoted in case of Payment
Q5) What shall be the amount taxable under
to hotel or to travel agent since the amount paid in
“Income from House property” with respect to let
cash is less than ₹ 50,000 in each case. However,
out of house property by daughter of Mr.
since SC as well as SDV of HP purchased on 10-09-22
Alishaan and in whose hands it will taxable?
is more than ₹ 10L, hence PAN is required to be
(a) ₹ 1,40,000 taxable in the hands of
quoted in such case
daughter of Mr. Alishaan
Q4) What shall be the amount taxable and in whose
(b) ₹ 1,40,000 taxable in the hands of
hands with respect to purchase of immovable
husband of daughter of Mr. Alishaan
property by Mr. Alishaan from RK Builders and
(c) ₹ 1,38,500 taxable in the hands of Mr.
gift of the same to his daughter?
Alishaan
(a) ₹ 1,00,000 in the hands of Mr. Alishaan,
(d) ₹ 1,40,000 taxable in the hands of Mr.
₹ 1,00,000 in the hands of RK builders
Alishaan
and ₹ 20,00,000 in the hands of minor
Solution) Option (C) is correct
daughter
Rent received from HP 2,00,000
(b) Nothing is taxable in the hands of Mr.
Less:- Standard Deduction u/s (60,000)
Alishaan, RK Builders and minor
24(a) being 30% of NAV
daughter
Income u/h House Property 1,40,000
(c) ₹ 1,00,000 in the hands of Mr. Alishaan,
Since HP has been transferred to minor married
₹ 1,00,000 in the hands of RK builders
daughter, provisions of Deemed owner u/s 27 won’t
and nothing is taxable in the hands of
apply. However, clubbing provisions u/s 64(1A)
minor daughter
shall apply and income will be clubbed in hands of
(d) Nothing is taxable in the hands of Mr.
Mr. ALishaan (Since it is given that his wife is a
Alishaan and RK Builders but ₹ 20,00,000
house-wife, it is reasonable to assume that income
is taxable in the hands of minor daughter
of Mr. Alishaan is more than his wife and hence
Solution) Option (B) is correct
clubbing of income in his hands). Further, he shall
RK Builder Here SDV is not more than 110% of
also be entitled to exemption of ₹ 1,500 u/s 10(32)
Mr. Sale consideration, hence Sec 50C or
Alishaan Sec 43CA won’t apply in hands of RK

Case Study No. 30 :- Income under the head Capital Gains + Gift Provisions + TDS
Mr. Ram, an Indian resident, purchased a residential house property at Gwalior on 28.05.1999 for ₹ 28.5 lakhs. The
fair market value and the stamp duty value of such house property as on 1.4.2001 was ₹ 33.5 lakhs and ₹ 32.4
lakhs, respectively. On 05.02.2012, Mr. Ram entered into an agreement with Mr. Byomkesh for sale of such property
for ₹ 74 lakhs and received an amount of ₹ 3.9 lakhs as advance. However, as Mr. Byomkesh did not pay the balance
amount, Mr. Ram forfeited the advance.

  
Prepared by : CA Sanchit Grover 123
   CHAPTER 18 : CASE SCENARIO BASED MCQS
On 15.04.2022, Mr. Ram sold the house property for ₹ 2.10 crores, when the stamp duty value of the property was ₹
2.33 crores. Further, he purchased two residential house properties at Delhi and Mumbai for ₹ 54 lakhs each on
28.08.2022.
On 28.02.2023, Mr. Ram signed agreement to sale the house property at Mumbai to his nephew, Mr. Vaibhav, for ₹
58 lakhs, from whom ₹ 19,000 was received in cash on 15.01.2023 as advance for signing the agreement to sale.
Sale deed was registered on 30.03.2023 on receipt of the balance amount through account payee cheque from Mr.
Vaibhav. The stamp duty value of house property at Mumbai on 28.02.2023 and 30.03.2023 was ₹ 61 lakhs and ₹
64 lakhs, respectively.
Cost inflation index – P.Y. 2022-23: 331; P.Y. 2011-12: 184; P.Y. 2001-02: 100
Based on the above information, choose the most appropriate option of the following Multiple Choice Questions
(MCQs):-

Q1) What shall be the indexed cost of acquisition of Full value of Consideration 2,33,00,000
residential house property at Gwalior for (Since SDV of ₹ 2.33 Cr. is more
computation of capital gains in the hands of Mr. than 110% of SC of ₹ 2.1 Cr., hence
Ram? Sec 50C shall be applicable here)
(a) ₹ 1,10,88,500 Less:- Indexed COA 94,33,500
(b) ₹ 1,07,24,400 LTCG before exemption 1,38,66,500
(c) ₹ 94,33,500 Less:- Exemption u/s 54 (Since 1,08,00,000
(d) ₹ 97,97,600 amount of LTCG is less than ₹ 2
Solution) Option (c) is correct Cr., hence exemption can be taken
Actual Purchase Price 28,50,000 in respect of purchase of both the
FMV as on 1 April 2001
st
32,40,000 properties)
(can’t be more than SDV as on the Taxable LTCG 30,66,500
same date)
Q3) The amount of capital gains taxable for A.Y.
Less:- Advance money forfeited on (3,90,000)
2023-24 in the hands of Mr. Ram for sale of
05-05-2012, to be deducted from
residential house property at Mumbai is –
COA
(a) ₹ 7 lakhs
Indexed COA 94,33,500
(b) ₹ 10 lakhs
(28,50,000 x 331/100)
(c) ₹ 61 lakhs
Q2) The amount of capital gains taxable for A.Y. (d) ₹ 64 lakhs
2023-24 in the hands of Mr. Ram for sale of Solution) Option (a) is correct
residential house property at Gwalior is – Full value of Consideration 64,00,000
(a) ₹ 30,66,500 (In the given case, consideration
(b) ₹ 84,55,500 received at time of agreement is in
(c) ₹ 27,02,400 cash, hence we will consider SDV
(d) ₹ 14,11,500 of ₹ 64L as on 30.03.2023. Since it
Solution) Option (a) is correct is more than 110% of SC, hence
Sec 50C applicable in this case)
Less:- COA Nil

 
124 Prepared by : CA Sanchit Grover
  Multiple Choice Questions
  for CA Inter Taxation For May 2023 Exams

(₹ 54L minus ₹ 54L in respect of during P.Y. 2022-23 assuming the tax was fully
exemption taken earlier - Since HP deducted by both the buyers at the time of
at Mumbai has been sold witin 3 payment?
years from 28.08.2022) (a) ₹ 2,91,000
STCG 64,00,000 (b) ₹ 2,94,000
(c) ₹ 2,68,000
Q4) The amount taxable under section 56(2)(x) in
(d) ₹ 2,97,000
the hands of Mr. Vaibhav, if any, is –
Solution) Option (d) is correct
(a) ₹ 3 lakhs
As per Sec 194IA, TDS is to be deducted @1% of Sale
(b) Nil
consideration or SDV, whichever is earlier.
(c) ₹ 6 lakhs
TDS on sale of property in Gwalior 2,33,000
(d) ₹ 5.50 lakhs
(1% of ₹ 2.1 Cr or 2.33 Cr
Solution) Option (c) is correct
whichever is earlier)
Since nephew is not covered in the definition of
TDS on sale of property in Gwalior 64,000
‘relative’, hence difference of SDV and Purchase price
(1% of ₹ 58L or 64L whichever is
will be taxable in hands of Mr. Vaibhav. This difference
earlier)
(₹ 64L – 58L) is more than 10% of purchase price and
Total amount of TDS 2,97,000
is also more than ₹ 50,000
Q5) What shall be the total TDS available with Mr.
Ram with respect to sale of two house properties

  
Prepared by : CA Sanchit Grover 125
   CHAPTER 18 : CASE SCENARIO BASED MCQS

 
126 Prepared by : CA Sanchit Grover

You might also like